Sat1 1 PDF

You might also like

Download as pdf or txt
Download as pdf or txt
You are on page 1of 222

e-$pl

*:I
uruerob

MATFI
WORKBooK
sTH EDITIoN
Lawrence S. Leff, M.S.

Hundreds of math questions reflective Extensive coaching to familiarize you


of those that you'll find on the SAT's with both multiple-choice and grid-in
Math section SAT math questions
l All questions come with solutions Detailed advice and tips on how to
l Special math Strategies that will help approach and excel on the SAT Math
you approach and answer unfamiliar Test
types of questions

'sAT is a Egi§teEd tlademark ol the college Boaíd, which Wm nol involv€d in lhe pmduction ol, and d0as not€ndoíse,lhis prcduct,
General Math Strategies
*i§l§] '§§.'

Overview
This lesson deals with math strategies that do not depend on the format of the
test question. Knowing difierent SAT math strategies can help you solve
unfamiliar types of problems. Strategies do not tell you the specific steps to
follow when solving a problem. They merely suggest approaches you can try
when figuring out the answer to a nonroutine problem,

Draw a Diagram
Drawing a diagram can help you visualize a problem situation and organize the
important facts.

EXAMPLE
Amy goes shopping and Spends one-third oí her money on a new dress. She then goes to
another store and spends one-halí oí the money she has leít on Shoes, lf Amy has s56 leít
after these two purchases, how much money did she have When she started shopping?

Solution
. Draw a rectangle to replesent the amount of money Amy had when she
started shopping. Since Amy spends oFher money on a dress, divide the
}
rectangle into J equal parrs:

Dress

Two rectangles remain. Amy spends ] oí the remaining money on shoes.


Hence, the second rectangle represenis the amount of money Amy spends on
shoes. Since $56 remains, fill in the last rectangle with this amount:

I)tess Shoes $56


. Since each ofthe three rectangles is equal, Amy started TIp ó
shoppingwith3 X $56 - $168. Crid in 168. Never
attempt to grid in
special symbols
such as $ and 7o.

];]],li'
::;]],: Look at a Speciíic CaSe
If a problem does not give specific numbers or the actual dimensions of a figure,
make up a simple example using easy numbers.

24
General Math Strategies 25

EXAMPLE
The perimeter of a rectangle is 10 times aS great as the Width oí the rectangle. The length
of the rectangle is how many times as great aS the Width oí the rectangle?

(A) one-half
(B) two
(c) three
(D) íoUí
(E) thé length and Width are equal

SoIution
. Consider a specific rectangle whose width is 1. Iíthe perimeter ofthe
rectangle is 10 times as great as the width, the perimeter of the rectangle is 10.
4

perimeter = 10 width = 1

length = 4

. Because (2 X length) + 2 Xl: l0, 2 X kngth : 8, so tlre lengdl of the


rectangle is 4.
. Since length : 4 and width : 1, the lerrgth is 4 times as great as the width,

Tlre correct choice is (D).

Plug in Numbers to Find a Pattern


'tou can solve some
problems by subsdtuting a few test values for a variable until you
_iiscover a pattern.

:X,AMPLE
When a positive integer k is divided by 5, the remainder is 3, What is the remainder When
3k s divided by 5?

: r Utlon
. ,: a |ew positive integers that, when divided by 5, give 3 as a remainder. Any
.itive integer that is the sum of 5 and 3 or of a multiple of 5 (i.e., 10, 15,etc.) and
,.,,il]
' have this pfoperry For etample, when 8, 13, and 18 are each divided by 5, the
,_::.rinde
r is 3:
b k- )
8 5:1 remainder 3
1,3 1.' - 5:2 remainder 3
18 18 + 5:3 remainder 3
26 Math strategles Yau Need to KnoW
5 and find the remainders,
Now, using the same values for k, dilide 3k|:y


k3b 3k+ 5
24+5 :4 remainder 4
13 39 39+5 remainder 4
18 54 54+5 :10 remainder 4

The correct answer is 4,

Value when None ls Given


,..,',.,,,,'' Choose a Convenient Starting
changes,without knowing its beginning
If you need to figure out how a quantity
;rl'".:;il;;;Jo"ing ,au, ,l,i, ,n,k" ,h, ],irh"ri, easy, Carry out (he com-
that
rr-"" th, fi",l answer with the starting value
;r*,io"n."iil;ái. "a""] ",i""
you chose.

EXAMPLE
purchased, By what
The curíent value oí a stock is 20%
-oi less than its va|ue when it was
its original
uuru" in" r.l"" in order íor the stock io have
percent must the "to"t
"u""nt
ValUe?

(^) 20%
(B\ 25%
(c) 30ol.o
(D) 33T%
(E) 50%

Solution
100
stock, \íhen working with percents,
Choose a convenient starting value ofthe
is usually a good starting value,

Assume the original value ofthe stock


was $100,
' ,i,i",',á Si",e the curlent va]ue is 20% less than
the
' Find the current va]ue
original value, the current value is
$'OO - O,'O($'O0) = $tOO - $20: $80

value ofthe stock must increase in


order
' Find the amount by which the current which is a
;;;g;i;;i".,igi"a,a"", rr,",",1", -"" rise from $80 to $100,
change of $20,
of the stock must rise in order
' To find thepercentby w|,Lich the current value
find what percent $20 is of $80, Since
for the stock to r,"u, *, o,igi""l value,
$20,. too% :!x rcovo : 25o/o
$80 4
in order for the stock to regain its
original
the current value must increase 25o/o

value.

The correct choice is (B),

!..
General Math Strategies 27

EXAMPLE
13
Fred gives T of his DVDS to Andy and then gives 4 of the remaining DVDS to Jerry, Fred
now has What fraction of the original number oí DVDS?

(A) a12

(B) 1
6
(c) 1
3

'12-9
íD]

"2 1
(E)

Solution
Since Fred gives j and then of his DVDs away, pick any number that is divisible by
}
borh 3 and 4 for the original number ol DVDs. Since 12 is the lowest common
multiple of 3 and 4, assume that Fred starts with 12 DVDs.

. After Fred gives 4 (: X 12) discs to Andy, he is left with 8 (: tZ - 4) D\{Ds.


+
. Since J of 8 isi X 8 or 6, Fred gives 6 of the remaining DVDs to Jerry,
This leaves Fred wirh Z (- 8 -
6) of rhe original t2 DVDs.
. Since 2: Fred now has of the original number of D\{Ds.
12 }, }
The correct choice is (B).

:}1
,,a1 Make Organized Lists
Some problems can be solved by making a list and then discovering a pattern.

]üMPLE
People enter a íoom one at a time and are given a name tag in one of íive possible colors.
The colors are given out in this order| red, blue, White, green, and yellow. What is the color
of the name tag,that is given to the g3rd person Who enters the room?

(A) red
(B) blue
(c) White
(D) green
(E) yellow
28 Math stralegles You Need to KnoW

SoIution
Determine the colors of the name tags that the first ten people who enter the room
receiye:
person 1: red person 6: red
person 2: blue person 7: blue
person 3: white person 8: white
person 4: green person 9: green
person 5: yellow person 10: yellow

The pattern ofcolors repeats every five people. This means that if the number ofthe
person is divisible by 5, that person will receive a yellow tag. For example, since 10 is
divisible by 5, the tenth person will receive a yellow tag. Ifthe number of the person
is not divisible by 5, then the remainder tells where the person fits in the cycle of five
colors. For example, since 7 + 5 : 1 remainder 1, the 7th person receives a blue tag
since blue is the second color in the repeating pattern of colors. Since 93 + 5 : 18
remainder f,, the 93rd person will receive a white tag which is the third color in the
repeating cycle of five colors.

The correct choice is (C),

EXAMPLE
What is the units digit oí 335?
(A) 1
(B) 3
(c) 5
(D) 7
(E) 9

Solution
Use your calculator to help make a list of consecutive powers of3.

. List consecutive powers of 3 beginning with an exponent oF 0:

3o__l 34:8t 38:656L


3| :3 35 - 243
31 :2 36 :722
3':27 3- : 248Z
. Do you Notice that as the exponent oí3 increases from 0 to 3,
see a pattern?
the units digits become 1, 3, 9, and then 7. Similarly, when the exponent of3
increases from 4 to 7, the sequence of units digits is once again 1, 3,9, and7,
This repeating pattern of the same four units digits means that the units digit of
an1 power of 3 can be obtained by dividing the exponent by 4 and making the
remainder the new exponent of 3.
. Since 35 + 4: 8 remainder 3, 335 must have the same units digit as 33. The
units digit of33 (: 2D is 7 so the units digit of335 is also 7.

Hence, the correct choice is (D).


General Math sarateg/es 29

Redraw Figures to Scale


If a figure that accompanies a question is labeled "Figure is not drawn to scale,'' then
redrawing the figure to scale may reveal a íact that the test makers are trying to hide.

Note; Figure is not drawn to scale.

EXA|VPLE
ln reclang|e JKLM above, ií JK : KL, What is the ratio ol JH to KM?

solution k
Since the figure is zar drawn to scale, redraw it so that
filooks equal to KL Now it should be clear that since
]K: KL, the figure is a square. More important, ,/11
looks one-half the length of &41so the ratio of JH to
KM appears to be 1 :2.
you could also have arrived at the same conclusion
r-rsing mathematical reasoning: J
. The diagonals of a square bisect each other so ff1- ! rUanala: !]t.
. KM: 8. Since the diagonals ofa square have the same length,
Suppose
KM: ]L - 8, which makes/1: KH: +x 8: 4.
. Thus, the íatlo of JH (: Kí!) to KlV[ is 4 to 8, which can be written as the
fraction j or as 0.5 in decimal form. Grid in as tt2 or .5.

.',..

' : ] lnsert Units of Measurement


\\hen writing a propoftion to solve a problem involving rates, include the units of
:]-]easuíement. You can make sure that the terms of the proportion are placed in their
:orrect positions by verifying that the same units of measurement appear in
,orresponding positions.

.-XA|VPLE
The trip odometer of an automobile improperly displays only 3 miles for every 4 miles
actually driven. lf the trip odometer shows 42 miles, how many miles has the automobile
actua|ly been driven?
30 Math Siraíegles You Need to Know

Solution
Form a proportion in which each side represents the rate at which odometer miles
translate into actual miles driven. if r represents the number of actual miles driven
when the odometer shows 42 miles, then

3 odometer miles 42 odometer miles


4 actual miles r actual miles

Since odometer miles are on top in both fractions and actual miles are on bottom in
both fractions, the terms of the proportions have been placed correctly. To solve for
l6,8
x crosr-mulriply: 3x - 4,q) so x = SC .il.".
3

EXAMPLE
A machine can stamp 72 envelopes in 45 seconds. HoW many envelopes can the same
machine stamp in 2 minutes?

Solution
Form a proportion in which each side represents the rate at which the machine
stamps envelopes. If r represents the number of envelopes that the machine can
stamp in 2 minutes, then

72 envelopes I x envelopes
sec
45 sec 2 min

Although the terms ofthe proportion are in their correct positions, it's easy to see that
the units of time measurement are za' consistent. Since 1 minute is equivalent to 60
seconds, 2 minutes : 2 X 60 - 120 seconds: i

72 envelopes _ x envelopes
45 sec 120 sec

Solve for x by cross-multip|ying: 45x : l20.7 ), x &4


4,
: rlZ envelopes.

.',,,
l Work Backward
',.:,,:1
-When
you only know the end result ofa computation and want to 6nd the beginning
value, reverse the steps that led to that final result.

EXAMPLE
sara'S telephone service cost $21 per month plus $0.25 for each local call, and long-distance
calts are extra. Last month, Sara's bill Was $36.64, and it included $6.14 in long-distance
charges, HoW many local calls did she make?
General Math straíegles 3,|
Solution
I
§íork back from the áct that Sara's final bill was $36.64.
. Since the final bill included $6.14 in long-distance charges, the part ofthe bill

.
that did not include any long-distance charges is $36.64
-
$6.14 $30.50. -
The final bill included a fixed monthly charge of $21. Thus, the remaining part
of the bill that includes only charges for local calls is $30.50 $21 -$9,50. -
. Since each local call cost $0.25, the number ofloca] calls i, $9'50 : ;S.
$0.2 5

use Reíerence lnformation as Needed


As you work through a test section; you may find a problem that you know how to
solve but realize you've forgotten a particulaf formula needed for the solution. If this
happens, look back at the beginning of the test section and see if the formula is
included in the reference information box, which looks very similar to the one below:

eű,Afl:ú,,N *,|r.[x
A=ü2
C=2tr
A
0

_l,w
t\

9
A=+bh
(
V
,.,--""?-_

u ,5'
=l,wh V_rPh é=a2+ÉSpJáiat
l, /r'| \

s
night Triangles
The number oí degrees oí arc in a circle is 360,
The sum oí the measures in degrees of the angles oí a triangle is 180,

=XA|VPLE
The diameter of the base oí a right circular cylindeí is 2, and the distance from the center
of one base to a point on the circumíerence of the other base iS 4. What is the Volume of
the cylinder in terms of 1T?

(A) 3n
(B) 5tr
(c)
(D) ]i\ lc
(E) t,17

S l]ution
o solve the problem, you need to know the formula for the volume, I of a right
, .]inder. The reference section gives this formula as V : trlh.
\bu need to know rand / before you can use the volume formula.
32 Math Stralegles You Need to KnoW

. Since the diameter of the base of the cylinder is 2, the radius, r, of the cylinder
.1
ls _x2=1.
2
' The distance from the center ofone base to a point on the cifcumfefence ofthe
other base is the hypotenuse of a right triangle. One leg of the right triangle is
the height, /r, of the rylinder. The other leg is the radius drawn from the given
point on the circumference of a base to the center of that base, as shown in the
accompanying figure. Use the Pythagorean theorem (see the reference section, if
needed) to find /r:

b' +I' = 4'


h' +1--16
h' =I5
h=JB

. Use the volume formula:

V= rr'h
=-(t')(Jls)
=oJB
The correct choice is (D).

Account íor All possible cases


Solving a problem may depend on breaking it down so that all possible cases
considered.

EXAMPLE
ln a certain homeroom class, 21 students are enrolled in math, 17 students are enrolled in
biology, 9 are enrolled in both math and biology, and 3 students are not enrolled in either
course. HoW many students are in the homeroom class?

í^) 22
(B) 25
(c) 28
(D) 32
(E) 50
General Math stralegles 33

Solution
Account for all possible nonoverlapping sets ofstudents that comprise the homeroom
class. Students can be enrolled in both courses, in exactly one of the two courses, oí
in neither of the two courses:

students in both courses 9


Students in math but not biology :2I-9: 12

Students in biology but not math 8


students zo, in either course 3
Total students in homeroom class

By accounting for all possible cases, you know that the total number of students in
the homeroom class is 9+12+8+3:32.
The correct choice is (D),
j.§* * ü}á

K-ffi
Overview
This section looks at a íew special math strategies that involve algebra,

§.i Write an Algebraic Equation


using an algebraic
Sometimes the most efficient way of solving a problem is by
solutions,
;;;;". Problems that compare quantities iend themselves, to algebraic
first iáenrif} the "base" quantity.to which one or more other
i"'rhi.
q,r"rtiti.. are being compared. Úter assigning a _variable to the base
quantity,
th. othe. i rurrtiiies using th, ,^,e variable, Then translate the condition
'p.;problem,
.'.p....n,
of'th. problerrr that iells how these quantities are related into an equarion,

EXAMPLE
times as many tickets Weíe
Ticket Sales receipts for a music concert totaled s2,160, Three
sold íor the sunday afternoon concert, TWo
sold for the saturday night concert as Were
night conced as Were sold for the sunday
times as many tickets Were SoId for the Friday
Sold for $2,oo each, Find the number of
atternoon concert. Tickets for all three concerts
tickets sold for the saturday night concen,

(A) 90
(B) 180
(c\ 27o
(D) 540
(E) 600

Solution
. pick out the quantiry to which the others are being compared. The numbers
of tickets ,olj fo, F.id"y Saturday night concerts are being compared to
".,d
the number of tickets sold íor the Sunday afternoon concert,
. Árig; u".i"st.s. Since the bxe quantiry is the number of tickets sold for the
Sunday afternoon concert:
Let]í:thenumberofticketssoldfortheSundayafternoonconcert.
Then 3x: the number of tickets sold for the saturday night concert,
and 2x: the number of tickets sold for the Friday night
concert,

34
Special Algebra Based slralegies 35

Identi{} the condition that relates the ticket sales for the three concerts. Since
each ofthe tickets costs $2 and the ticket sales receipts totaled $2,160:

Total ticket sa/es rereiprs

x. $2 + 3x. $2 + 2x. $2 = $2,160


$2x+$6x +$4x=$2,160
$I2x = $2,160
-:' j' :
í: $2,160 ]80

. Answer the question that is asked. The number of tickets sold for the Saturda1
night concert : 3, - 3{tSO; : 54O.

The correct choice is (D).

:, : Creatively Manipulate Systems of Equations


SAT problems involving sets of equations with more than one variable may seem
cornplicated at 6rst glance. However, they can often be solved by making a
straightForward substitution or by performing a simple operation on the equations.

EXAMPLE
|'í 2d e = 32 and 2e d : ,í0,
what is the average of d and e?

Solution
1-he average of d and , i" d !r' , Rather than solving the system of equations for each
ofthe individual variables, think ofan easy way of combining the two equations to
:et a simpler expression. Tly adding corresponding sides of the two equations:

2d-e =32
-d +2e = I0
d+e =42
, his is very close to what you need to find. Now divide each síde of t! -l e: 42
:r,2:
d +e 42

_ he average of d and eis2I.


36 ',la!h strategies You Need to KnoW

EXA|VPLE
|f x y = y+ 3: Z What istheValueof x?
(A) -4
(B) 8
(c) 10
(D) 11
(E) 13

SoIution
The extended equation x, ! - J+3 - 7 means í- ! : 7 and ! +3 : 7.
. Begin by solving the second equation since it contains only one variable:

j+3) 3:7-3so7:4.
. Replace 7 with 4 in the first equation, which gives x- 4 : 7. Solve for x
(x-4)+4-7sox-11,
. The correct choice is (D).

EXAMPLE
É !]8 - w and gv - 2w. what is the value of 9?
V

SoIution
Variables u and, u are both related to variable za. Solve for zu in the second equation
and then substitute the result in the first equation.

. Solve for z in rhe second equation:


! ! - , 3,
. Substitute j u for win the first equation:

":T,
l8 2"
' Solve For
lby interchanging the positions of 18 and z in the proportion:

!:!.,r:+
u2l k 27

The value of ! is 27 .

l1l:._l] Rewrite Equations to Help Evaluate Algebraic Expressions


Given an equation with more than one unknown and an algebraic expression
involving the same variables, rewrite the equation in a way that allows you to find the
value of the algebraic expression through a substitution using that equation.
Special Algebra-Based Síraíegles37
EXAMPLE
|Í X - 2y : 1, What is the Value oí 23, , 6n

Solution
Do not try to solve the given equation. Instead, think of a simple relationship that
connects the equation and the exponent of 23. - ur. Multiplying both sides of the
equation x - 2t - 1 by 3 gives 3x -
@: 3. Now that you know the exponent is
3, make the substitution and do the math:

23x 6]:23:2X2x2:8
The answer is 8.

EXAMPLE
rt : wnat is the value oí 39*?
{,
fr

SoLUTIoN
Solve the proportion for ! simp|y by inverting borh sides. Since you now know
that ? is make the substitution:
|,

?=+(L\
Jm )\m )
3

=9
7
^|
=XAMPLE
|Í (x y)2 : 50 and Xy = ,10,
What is the Value of X, + y,?

SoIution
Find a connection between (, -
!)' - 50 and x2 -l yr. Expanding the square ofa
rinomial produces an expression that includes the squares of im t*o t..-.,

@- l)' = (, _ ),)(, _
),)=,. _2x7 + 7,
This means that x2 - 2r! -| !': 50. You can now find the value of x2 -l 71 by
.ubstituting 10 for xy,

x2-2(l0)t!,:50
x2+1z:50+20
x2 + 7':79
]he correct answer is 70.
38 Math sírategles You Need to KnaW

EXAMPLE
lfl:Zana9= |jj :
sr 3. then

,-. 2
J
7
(B) :
(c)
^:

o
(D)
;
ll:\ l

SoIution
^. r _
slnce
J
: 2s and, s : 3r. Make these substitutions
=2and-:3, r in the fraction:
st
r+s 2s+s 3s_ 3/s\ _ l

s+í 3t+t- +t- -l4V )


Using the given information,
substitute 3 for l, =1(3)
1

t 4
:2
4
The correct choice is (D),

- ,::.:::].:

:i,,: :: Account for All Solutions oí a Higher-Degree Equation


Dont forget that a second-degree equdtion has two solutions, a third-degree equation
has three solutions, and so Forth.

EXAMPLE
lí X2 : Ax,lhen X :
(A) 0 only
(B) 2 only
(C) 4 only
(D) 0or2
(E) 0or4

Solution
Since the greatest exponent ofx in the given equation , l : 4x, is 2, the equation has
/za solutions:
.If§_ x,theni,_4x:0.
. Factor out r : x(,r - 4) : 0. If the product
of two quantities is equal to 0, then
either quantity may be equal to 0. Hence, x : 0 or x - 4 : 0, so x - 4.
. The two roots ofthe equation are r: 0 or x: 4.

The correct choice is (E).


- Multiple-Choice-Question§
Special Math Strategies íor Reqular l§l § *!.

Overview
The correct answef to a regular multiple-choice question is given to you, along
with four other choices that are designed to distract you. Somerimes you can
use a "backdoor" strategy that will allow you to identify the correct answer
without going through a standard mathematical solution.

Test Numerical Answer Choices in the Question


When each of the answer choices for a regular multiple-choice question is an easy
number, you may be able to find the correct answer by "baclaolving." To baclsolve,
plug each of the possible choices into the question until you find the number
that works.

EXA|VPLE
When 5 is divided by a number, the result is 3 more than 7 divided by twice the number.
what is the number?

(A)

(B)

(c)

(D)

(E)

]:iution
.':ther than thinking ofan equation and then solving it algebraically, plug in each of
' .. answer choices as the possible correct number until you find the one that works
ihe problem.
l
,,,,ice (A): ,4
fry
",
the unknown number:

A -:,L
51
'Z* 3t7'l ?,

39
40 Math sírategles You Need to Know

-1
Choice (B): Tiy ,2: as the unknown number:

A _}I_
5*f2)= 7+7+?
The correct choice is (B).

EMl\4PLE
|í 22' 1 : 32, then X :
(^) 2
(B) 3
(c) 4
(D) 5
(E) 6

SoIution
Since rhe answer choices give the possible values of r,
replace r with each of these
values untll you hnd the one that_makes 22' r
equal to 32. As the value ofx increases,
so does the value of 22-1. Th-:!:., yo,,
-"y t. able to save time by starting with
choice (C), the middle x-value. If this valu. oi"m"k
.2r-' ,oo l".gá th.., th'..o.-
rect answer must be smaller than the middle ,r-value.
In this case, you can eliminate
choices (C), (D), and (E). Similarly, you can eliminate.t
oi..r-(a), iB), and (C) if the
middle x-value makes 22í-1 too small
. Plug in choice (C) for x. If x - 4, then
)2,, - ]]1.1l l

11'r,rx2x2x2x2x2
:128
Since the correct value ofx must be smaller than 4,
try choice (B). If choice (B)
doesnt work, the correct answer must be choice (A). '
. Plug in choice (B) for x.If x: 3, then
22tl:22|3)l
_)'
:2X2X2X2X2
-12
Hence, choice (B) is correct.

Change Variable Answer Choices into Numbers


If you don't know how to find.the answer to a regular
multiple-choice question in
which the answer choices contain letters, substitute simple
for the letters.
"rLU*.
Special Math strategles for Regular Multiple-Choice Queslions
41
EXAMPLE
lí m + n js an odd number Wh en m and n are positive integers, Which expression always
represents an even number?
(^) m n
(B) m2 + nz
(c) (m + n)'
(D) m' - nz
(E) (mnf

Solution
Pick easy to work with numbers for m and n such
that m n is an odd number. Since l
_]+ 2 -_ 5 is an odd number, substit ute for
3 m and,2 for z in each of the answer choices:
' (A) m- n:3 - 2:1
' (B) m' l n2 = 32 -| 22 : : 13 9+4
' (C) (n + r)' _ (3 + 2)2 : 5z _ r,

' (D) m' - n2 = 32 _ 22 = 9 _ 4 :


5

' (E) (mn)' : (3 x 2), : 62 : 36


:ince the o:ly..u= number produced by the substitutio
ns for m and nis 36, the
;orrect choice is (E).

aü|VPLE
lf t ties cost d dollars, how many dollars Would t + .1
ties cost?
(A) d+ 1

dt
t+1
d +1
(c)
t+
- 1

dít + 1)
(D) ------
t

t@+1\
(E)
d

:: ution TlP
Avoid picking 0
. :k easy numbers for tand l such as t: 2 and, d: I0,If t^ro ties cost $10, then and 1, as these
:e. tie costs $5 and three ties (r '| 1)cost$lJ.Substitute2forrandlOfor numbers tend to
din each produce more than
: :he answer choices until you find
the one that.u"tu",.. a ti.
answer choice. lf
: correct choice is (D), since more than one
answer choice gives
d (t +t) the same correct
_ 10(2+ 1)
_ 30 answer, then start
= 15.
2 over with different
n u m bers.
42 Math slrategles You Need to KnoW

Guess after Ruling Out Answer Choices


Ifyou get stuck on a multiple_choice problem but can eliminate some ofthe answer
choices, guess rather than omit the question. Ifyou skip multiple-choice questions,
you have no chance of gaining points lor those questions. Each time you guess
correctly, however, you add l point to your law scofe; ilyou guess incorrectly, you
lose only ] ofa point for the wrong answer. You can improve your chances ofguessing
the correct answer by eliminaring answer choices that you know are unlikely or
imposiible.

Rule out an answer choice if it stands out as looking very diflerent in form
From the other four choices. For example, suppose the five answer choices of a
multiple-choice problem are these:
]
(1t)
;rr
(B) lrt-JT
(!C) rQ+Ji
(D) r(r'+2)

(E) To1 -z)

Choice (A) is the only choice that does not include the product ofrr and a
parenthesized expression. Because this choice looks very dilferent from the other
iour choices, it is probably not the correct answer, so you can eliminate it.

Rule out this answer choice:


(E) It cannot be determined from the information given

when it belongs to a multiple,choice question that appears late in a test secrion.


This answer choice may be correct when the question appears early in a test
section, but it is rarely the correct answer when the question appears near the
end of a test section.

Rule out an answef choice that does not make sense based on the facts of the
problem. For instance, consider the example that appears under Strategy i0 on
page 32. Since there are 21 students enrolled in Math and 3 students are not
enrolled in eithcr Math or Biology, the homeroom class must have at least
2l + 3 - 24 strrdents. This means that choice (A) can be eliminated. Since it is
not reasonable that the answer would simply be the sum 21 + 17 + 9 + 3 -
50, choice (E) can also be eliminated. This leaves only three possible answer
choices. You might also suspect that because the numbers used in the problem
are somewhat scatteted (2l, 17, 9, and 3), it is unlikely that the correct answer
Special Math síralegies far Regular Muttiple-Choice Quesllons 43
is only 1 more than the minimum of 24. This eliminates choice (B) and leaves
only two possible arrswer choices from which you can guess.

' Rule out an answer choice that contradicts arithmetic reasoning, as illustrated in
the next example.

EXAMPLE
ln June, the price of a cD player that sel|s for $150 is increased by 1o%. ln July, the prjce
of the Same cD player js decreased óy 10%o ot its current selling price. What is the new
selling price oí the cD player?
(A) $140
(B) $148.50
(c) $150
(D) s]52,50
(E) $160

AnaIysis
ls rhe new selling price equa1 to the original price of $ 150, less than the original
price,
,lr greater than the original price? Rule out choice (C) since "obvious" answers that
do
ror require any work are rarely correct. In June the price oF a CD player is increased
:v 10% of$ 150. In July the price ofthe CD player is decreased by iooÁ of"r, rrnou.,.
,:r tar tlJan $150 (the June selling price). Since the amount oftheprice decrease was
:reitter than the amount of the price increase, the
July price muit be /esr tban the
-:arring price oí$150. Yoú can, therefore, eliminate choices (C), (D),
and (E).
- his analysis improves your chances ofguessing the correct answer.
Of course, ifyou
..rorv how to solve the problem without guessing, do so:

1 Juneprice: $150 + (10%X$150): $150 + $15: $165


STEP
STEP2 Julyprice: $165 - (10% x $165) _ $165.00 _ $16.50: $148.50
sTEp 3 The correct choice is (B).
41 uaü slraieg/es YoU Need to Know

TPS Fü Boetií€ Your Score


r- B€foG $afting your solution, look at the five answer choices. The types
ofnumbers or the forms ofthe answer choices may help you decide on a
soluúon strategy. For example, ifthere are whole numbers in the question,
and decimal or fractional answers appear as choices, the solution may
involve division.
2. Ifyou think you have solved a problem correctly, but do not find your
answef among the five choices, try wriqing your answer in an equivalent
form. For example, ifyou donr find ],look for 1.5; if you dont find
2a -l 4, |ook for 2(a + 2); ifyou dont find 3'; :' , look for a - { since

3x- y-3x t :3- lX


; X

3. Avoid random guessing since one-fourth ofa point is deducted for each
incorrect answer to a multiple-choice question. Ifyou cannot rule out at
least two answer choices as definitely wrong, guessing wil1 probably not
help your score. If you can eliminate three ansurer choices as definitely
wrong, always guess between the remaining two choices rather than omit
the question.
4. Guess smartly. Cross out in your test booklet those choices that you
know are impossible or unlikely. ]fi/hen deciding which answer choices
can be ruled out, ask yourself questions such as these: Should the answer
be positive or negative? Greater or less than 1? A whole number? Is there
a number that the answer mrrst be greater than or less than? Can an
accompanying figure be used to estimate the answer?
5. Before guessing, think through a problem and attempt to solve it
mathematically. Guessing should be your last resort.
\,"]-.,,.;-'-, ".-,:.,
PAHT 2
!-.,, :.

BEVIEWINE I{EY MATH SIflLLS


-,,,.-.,i]:.]__.

AND IONIEPTS
,s§ {r§
Numbers, Symbols, and Variables
§

aa
-3* ü
Overview
A]l numbers that appear on the SÁT are real numbers. Real numbers include
the.different types of numbers, both positive and negative, you encountered
a.ll
in arit}rmetic and beginning algebra classes.
A variable is a symbol that serves as a placeholder for an unknown member of
a given set of numbers. Letters of the alphabet such as a, á, and x
are used as
variables to help make general statements about how numbers behave.

Types of Numbers
The set of real numbers is comprised of these sets of numbers:
. |Ybole numbers - 0, l. Z, J, . , .

Posi!ive int€8€§

. Integers 4,-3,-2,_ 1,0.IŰZ:.

I
Fractions .u.h 2 th"t have integers above and
|. ",
. Rational numbers : O.'o* ,h. f.".tio.,3b*
]
Terminating decimals such as 0.75
|.
[. Nonending, repeating decimals such as 0.33333. , ,

. [rrational numb"r, : {' Numbers such asn "nd 16 that do not have exact
l decima] equivalen Is

Comparison Symbo|s
Table 3.1 summarizes the symbols used to compare two numbers.

50
Numbers, Symbols, and Variables 51
TABLE 3.1

6,and_8, you can write 6 < 7 < 8. The


]]::"T_b:*:.n inequaliry 6 < 7 < 8
means ülat O ls.less than 7 and, at the same time,
7 is less than 8. In general, if xis
cetween aand bwitha1 b, then a .-x{
b.The inequaliry )= r= á means that
,i is between a and b, or may be
equal to either n o. á i, ,r,]*lr in Figure
3.1.

l- x_|
b

Figure 3.1 aa x< b

rldicating Multiplication
\\hen two or more numbers are multiplied
together, the answer is called the
Each ofthe numbers that are U.irg
|..odo.1.
lc product. 5lnce ) X 6 - 30. 30 is rhe -rl,iiii.J ,"i.,rr.. i. called a factor of
producr whi]e 5 ánd 6 are Factors of30.
Multiplication can be represented in ways,h".
symbol X;
"uoú ".i"g,h.
' two quanrities indicates mulriplication. For example,
f::::,:i:,11beyeen
+./means4tlmes,y.
' placing parenther.,
,.ourrd quantities written next to each other
means
exam,lle, Q)Q) is 2 times 3, 4() is 4 times
Tl1l,|1,iilll_jlr.
r tlmes trre sum ot x and
y.
n and 5(r + y) is
' Vriting a number and one or more yariables next
to each other means
multiplication. For,example, ab means n
ti^r, b, ;;;;"", 5 times a. If
,/
- +J tnen )n: ) X 4 = 20. The prodtsct 2ab ""dmeans 2 times a times b.If
1: 5 and ú= 8, then 2ab: 2X 5 X 8 : 10 X 8 : 80.
52 Aríthmetic Skills and Concepts

Like Terms
If a number and a variable are written next to each other, the number is called the
coefficient of the variable. For 5n, 5 is the coefficient of z. Since 5n meaas 5 times z,
5n: nl ni- n* nt z. rü/hen products such as 3z and 2n differ only in their
numerical coefficients, they are called like terms, Like terms may include more than one
variable. For example,4ab aná 7 ab are like terms, 6ut2ab and 3ac are notlike terms.

Laws of Arithmetic
Real numbers behave in predictable ways.
. Commutatiue latu: The order in which tao numberc are added or multiplied
does not matter. For example:

3+4:4-|3 and 3x4:4x3


. Associatiue law: The order in which three nlsmbers are added or multiplied does
not matter. For example:
2x(3x4)_(2x3)X4
where the parentheses group the pair of numbers that are multiplied first.
. Distributiue law: To multiply a number by a sum oírwo values, you multiply each
value in the sum by that number and then add the two products. For example:

3x(2+a): lx2+3x4
:6+12
: 18
Applying the Laws oí Arithmetic to Algebraic Expressions
The commutative, associative, and distributive laws can be applied to expressions that
contain variables since these expressions represent real numbers. For example:
. By the commutative law, x .l 3 and 3 l x are equivalent expressions.

. By the distributive law,


3(2x+ 5) - 3(2x) + 3(5):6x+ ],5

. By the reverse of the distributive law,


4x * 5x: (! + J)x: 9x
Thus, like tefms are combined by combining their numerical coefficients. Here are
three more examples:

EXAMPLE
3y+4y=7y

EXAMPLE
9p+3p+2p:l4p

EXAMPLE
5x - x: 4x since 5X - X: 5X - 1X: 4X
/Vurnbe/s, symbois, and Variables 53

Lesson 3-1 Tune-Up Exercises


Multiple-Choice
l. If a: 9 X 23 and b: 9 X 124, what is 5. In store l a scarf costs $12, and in store ,B
the value oF b , a? the same scarf is on sale for $8. How
(A) sot many scarfs can be bought in store B with
(B) 903 the amount of money, excluding sales ta-x,
(C) 906 needed to buy 10 scarfs in store,4?
(D) 909 (A) 4
(E) st l (B) 12
(C) 15
2. By how much does the product of8 and (D) 18
25 exceed the product of 15 and 10? (E) 21
(A) 25
(B) 50 6. Let * represent one of the forrr basic
(C) 75 arithmetic operations such that, for any
(D) 100 nonzero real number r,
(E) I25 r*0_ r and r*r:0
'§íhich
aiithmetic operation(s) does the
3. How many containers, each holding 16
symbol * represent?
íluid ounces of milk, are needed to hold
5 quarts of milk? (1 quart: 32 fuid (A) + only
ounces) (B) - only
(C) + and -
(A) 6
(B) 8
(D) X only
(E) +, -, or X
(C) l0
(D) 12
7. Kurt has saved $160 to buy a stereo
(E) 14
system that costs $400 including taxes.
If he earns $8 an hour after all payroll
+. If the current odometer reading oFa car is
deductions have been made, how many
31,983 miles, what is the LEAST number
hours will he have to work in order to
of miles that the car must travel before the
have exactly enough money to buy the
odometer displays four digits that are the
stereo system?
same?
(A) 20
(A) 17
(B) 24
(B) 239
(C) 25
(C) 350
(D) 30
(D) 650
(E) 40
(E) 1,350
ArithmetiC Skílls and ConCepts

8. If the present time is exactly i:00 p.M., 1 1. If r and 7 are positive integers,
what was the time exactly 39 hours agoi 2x * 1< 29, and 1> 4, what is the
(A) greatest possible value of ;r - .1?
4:00 p,u.
(B) 4:00 a.u. (A) 5
(C) 9:00 p.l,t. (B) 6
(D) 9:00 ,t.1,1. (C) 7
(E) t 0:00 p.v. (D) 8
(E) 9
9. Let # represent one of the four basic
arithmetic opelations such that, for any 12. r: 4?u
nonzero teal numbers r and s, r
r#O:r and r#s:s#r S: e+2
Vhich arithmetic operation(s) does the lf r q are defined by rhe equarions
and,
symbol # represent? above, what is the value of q when p -
aná u: 5?
(A) + only
(B) X only (A) 4
(C) - only (B) 6
(D) - and x (C) 8
(E) + and = (D) I2
(E) 15
10, If w - (6)(6)(6), x: (5)6)0), and
l- (q(q(S), which inequality statement
is true?

(A) x< 7< ru


(B) w<x<y
(C) 1,< w< x
(D)7<x<a
(E) w<l<x

Grid-ln
l . The houses in a cerrain communiry are 4. If 13'< h'< 21,9 < p < 19,2 < m <
numbered consecutively from 2079 to 2176.
6, and É.p, and m are inregers, what is the
How many houses are in the community? k-
l? b
larsest oossible value oF
m
2, lf l kilob; re of compurer memory is
equivalent to 1024 bytes and 1 byte is 5. For some fixed value 6f;r, 9 (x + )) : !.
equivalent to 8 bits, how many kilobytes After the value of r is increased by 3,
are equivalent to 40,960 bits? 9(x + 2) : za. V/hat is the value of
w-]?
3, A television will cost, including taxes
set
and finance charges, $495 if the buyer puts 6. If x and positive integers, and
1 are
$129 down and then pays offthe balance 3x 1- 27 : 21, what is the sum of all
in eight equal monthly payments. Under possible values of x?
this purchase plan, what will each
of the monthly payments bei
powers and Roots ]l§§§§§

ryr
#-d
il,n

Overview
The product of identical factors may be indicated by writing the number of times
the factor is repeated a half-line above and to the right ofthe factor. For example:

5 Exponent
2x2x2X2x2 _2,
2 is used as a factor 5 times L B"r.

Meaning of Exponent
Repeated multiplication of the same number may be ind.icated in a more compact
:orm by using an exponent that tells the number of times the number, called the
base, appears as a factor in the product. The notation b'' is read as "á raised to the
,;rh power." In b", the number á is the base and z is
the exponent.
When a number or variable appears without an exponent, the exponent is under-
.lood to be 1. For example, 3 : 3' and 1 : !',

_-XAMPLE
What iS the Value of 3a 432

ScIution
:lnce3a : 3X 3X 3 x 3 : : 4X4x4:
81 and43 64,
31 _43:87_64:17

-1ÁMPLE
'f 2' : 8, what is the Value of x3?

- =S - 2X 2X 2 : 23,tbenx: 3. Hence,
f:33:3x3x3:27

55
56 Aithmetic Skills and Concepts

Laws of Exponents
You should know these rules for working with exponents:
. To multiply powers with the same nonzero base, add their exponents. For
example:

24 X.23 : 2a+3 : 27 and br. b3 _ b2+3 : b5

Dont try to multiply powers that have different bases. For example:

25X34+(2x3ssta
. To divide powers with íl-Le same nonzero base, subtract their exponents. For
example:

:2' _ 2' ' :2" and 1h' :b':


23 b o

. Don't tfy to divide powers that have different bases. For example:
, . 5-4
2, + 3" +(Z|
\3/
. To raise a power to another power, multiply the exponents. For example:
(23)7 : 23x7 : 221 and (ba)3 : b4x3 _ b12

These rules are summarized in Table 3.2, where bases xand.7are not 0 and exponents
m and, n are positive integers.

TABLE 3.2

Here are some additional examples:


. (2a;l _ 23 , a3 : *al

, ,'
(;)' :"':23 8

. tt_ - x .l : x !
,'!
Powers and ,?oots 57

Order oí Operations
Arithmetic operations are not necessarily performed in the order in which they
are encountered. Instead,

P: First evaluate expressions within Parentheses.


E: Then evaluate Exponents,
M: Next, Multiply and
D; Divide, working From left to right.
A: Finally, ldd and
S: Subtract, again working from left to right.
The first letters of the keywords in the sequence of opefations spell ''PEMDAS,"
which can b-.e by memorizing the sentence '' Piease Exalse My
_remembered
Dear lunt Sally."

a(AÍ\/PLE
Find the value of 5 + 18 + (4 - 1)r.

So ution

5+18+(4-1)'-5+18+32
= 5 + 18 :9
5-* 2
-/
Square Roots
'Squaring
a number" means raising that number to the second power, The square of
< is 52 or 25. Finding the square foot ofa number reverses the p.o..r. oF rqrr".i.rg
:umber. The square root of 25 is 5 since 5 X 5 : 25, and thesquare root őr rc.ri l"
.:rce 4 X 4: 16. The square root ofa nonnegative number ,A/is one of two
lentical numbers whose product is /V
' Every positive number has two squafe roots. The two square roots of 9 are *3
and -3, since
ft3)(+3):9 and ( 3)(-3):9
The two square roots of 9 taken together may be represented using the shorthand
notation t3.

' In .,6, read as the "the square root of 9," the symbol

l radical or squafe root symbol, and


is called a Marx R§F§Rrilc§ FA§T.
the number underneath it, 9, is called the radicand.
Every positive number N has two square roots:

' The symbol .f indicates the positive or


the principal or positiue rqu".e root l, ,lN, th"
negative square root is_ VN, and the two square
principal square"l*"y,
foot of the number undernearh iL. roots together are {N.
Thus, J9 : 3, not _l3.
]_
58 Arithmetic Skílls and Concepts

Cube Roots
The cube root ofa number l/is one ofthree identical numbers whose product is _A[
The symbol f,_..p..r.rr,, ine ruba root oF the number that is written underneath it,
For example. fg :2 since 2X )v 2= 8,

Perfect Squares
The square root of a whole number may not be a whole number. For example, ,i7
do.,notequalawholenumbersinceitisnotpossibletofindtwoidenticalwhole
,rrr_b..s *iro." product is 7. A whole number is a perfect square if its square root is
, *hol. .,uÁber. The numbers \,4,9,16,25,36,49,64,8r,anő 100 are perfect
"lso
squares.

Properties of Square Root Radicals


Here are some facts about square root radicals that you should know:
. Because the product of the same two numbers cannot be less rhan 0, the sfT
does not ha.,e questions about square roots of negative numbers, sl,ch as J-64,
§íhenever you iee a square root radical over a variable, assume that the quantity
underneath the radical is greater than or equal to 0. Keep in mind, however,
that cube roots of negative numbers can be evaluated. For example:

V-r,q - 4 since (-4) x( 4) yí-4) .-64


. Ű X J' : J, b The product ofthe square roots ol two numbers is the
"
square foot of the product. For example:

Jí ",r5 : ,ti "l : la


ű -
- ,lL *hrr- (b The quotient ofthe square roots of two numbers
G \ + O\,
\b ,/

is the square root of the quotient. For example:

Ji
-'I-
61
_ 1ll-:
_ ^lz
-
\J / |/
. rJlll' : ,M The square ofa square foot radical is the radicand, For example:

tó)':Jl""B-s
. lf N is nonnegarive, th.n ,,ff : ly'. For example,

.If a: b, rhrn|i-,[ a,provided a and b are nonnegative. For example,

if x:5,,h.'f-6
. oJi + ,Ji - Q + öJi , To combine square root radicals with the same
radicands, combine their rational coefficients and keep the same radical factor, as in

sJi+zn6=lJ1
Powers and ,?oots 59
To simplify a square root radical, write the radicand MATH R§rERE!{E§ pAíT
as the pfoduct of two numbers, one of which is the
highest perfect square factor ofthe radicand, Then +,li + 16. Radicals cannot be distributed
"|ii
over the operations ofaddition and subtraction.
write the radical over each áctor and simplify. For example:
For example:
Jq+g+Ja+"|ó
JÁ:J4x5_űxJ1 :zJj
To eliminate a radical from the denominator of a fraction, multiply the
numerator and the denominator ofthe fraction by that radical, as in

5 5 .,
.:2(-:- Jj 5J'
G J:J:3
60 Aithmetic skills and Concepts

Lesson 3-2 Tune-Up Exercises


Multiple-Choice
1, ]f 5: a-,then j: If b3 : 4, then b6 :
(A) a't' (42
(B) a*' (B) 8
(C) a,_, (C) 12
(D) aI (D) 16
5 (E) 64
(E) a'
X 7. If z is a positive number and z2 : 2,
2. lű/hat is the greatest number of positive then u3 :
integer values ofx for whió í 1 xz < 50? (^J J2
(A) Five (B) 2J'
(B) Six rc)4
(C) Seven (D 3J;
(D) Eight (E) 6
(E) Nine
If 2- :
!2, which of the followirig must be
3. If his a positive integer, which of the equal to 2"+l?
following is NOT equivale nt to (23r)2? (A) l'
(A) (zh)6 (B)/+t
(B) 64* (C) 2l'
(C) 4k(24l,) (D) 4l'
E\'
.,1
(D) (893
(E) 23k(23h)
c}
Ifx is a positive integer such íhat x9 -r
: and x' =\ il,, which of the Following must
4. t L: :
, _23

x'5 and (ro)' *'o, then mn


X be equal to xl3?

(A) 13 (A) ru- I


(B) 24 (B) r+ w- 1
f2
(C) 28 (C)
(D) 32 -
1l)

(E) 40 (D)rz-w
El
,3 I+zw
5. If z : p3, then 8p must equal
(^)
(B) /u
f
(C) alo
iol'sú
(E) 16
Powers and ,qools 61

10. Given 7 :
ux2 and 1 ís not 0. If the values 14. If x is a positive integer, which of the
of x and, a are each doubled, then the following statements must be true?
value of y is multiplied by
í{l : llt
. .
tl
l0lr

(A) 1 i. '9

(B) 2 \"/ \x + 1/

(C) 4 II. (r()': xJ


(D) 6
(E) 8 I II. /,'ou )

l1. If J;is a positive integer, how many


l ",l- '

values of z are in the interval (A) I only


I00 < n< l99? (B) II only
(C) I and III
(A) Three (D) II and III
(B) Four (E) I, II, and III
(C) Five
(D) Six
15. If J/: 25 - x2 and 1
(E) Seven = Jí = 5, what is the
smallest possible value of .7?

12. If (23)' : 4?, then 3? : (A) 0


(B) 1
(A) 3 (C) 5
(B) 6 (D) 10
(C) 9 (E) 15
(D) 27
(E) 81
l (1. TÍ x
-r4- V6 and7' : 12. rhen _ :
', 3. If a and á are positive integers, 3"
then 3d+ b . 6d :
2]2
(A) 18"*, ,l:
(B) 18a,*a (B) n'
(C) 3slz,* t1 3
(,D) 9"tt , ,,
(,E) 32"+
b , 2, (C) 3-
./2
1-li
-,-
(D)
3
l.
(E) ]9
3

Grid-ln
]i2 \ 4': 1ó', rhen x 3, If (7 - 1)3 : 8, what is the value of
(7 + 1)'?
,a6 :
- ll !1 :7 a- and, , Il,whar isthe
,, alle of ab? b 4. If?+?+P:12aná? ) 0, what
p,P,P
is the value of7?
;§§ *};
Divisibility and Factors
§

Overview
]When 12 is divided by
3, the remainder is 0. Thus, 12 is űvisible by 3, and,
as a result, 3 is called a factor of 12. The numbers I, 2, 4, 6, and 12 are also
áctors of 12 since 12 is divisible bv each ofthese numbers.

The Parts of a Division Example


The parts ofa division example have special names. For example, 7 + 3 can be written
in fractional Fo.-
", l or in long_division
3, form as

2 <- Quotient
Divisor -+ 3F <- Dividend
-6
1 <- Remainder
Thus, 7 ,: e :1: z + ]. In general,
33
Dividend
: ouorient * Remainder
Divisor ' Diuirn,

If the remainder is 0 when x is divided by7, then xis diuisibleby


1. For example, 12
is divisible by 3, but it is zordivisible by 7.

Dividing by 0 is not a]lowed,,.We always assume the number we afe dividing by is not
equal to 0, For example, when a number is divided by x 2, - 2'
asin2,
x
rr cannot be
equalto2 since 2 - 2:0.
Odd and Even Numbers
An even number is 0,2, 4, 6, and 8
a number that is divisible by 2. The numbers
are even numbers. An odd number is a number that is not divisible by 2. The
numbers l, 3, 5, 7, and 9 are odd numbers. Consecutive even numbers and
consecutive odd numbers always differ by 2.
You can predict whether the sum or product of two numbers will be even or odd
by using these rules:

even i : even and


even even x even: evcn
odd + odd - even and odd x odd : odd
odd i even : odd and even x odd : even

62
Dívisibility and Factors 63
Factors and Multiples
Here are some terms you should know:

' The factors of a number i/are the numbers that can be divicled into i/with
a
remainder ofO. For example, the factors of 32 are

l, 2, 4, 8, 16, and,32
since 32 is divisible by each of these numbers.

' The common factors of two numbers are the factors that the numbers have in
common. For example:

Factors of l2: Í'l-.l 2.//^ 4, 6,


l ,./ -,/ 12

2I:
U (r/r, )l
Facrorsof

T'he common factors of 12 and 2l are 1 and 3.


' .{ny number that can be obtained by multiplying a number l/by
a positive
,::.q.. ir called a multiple of r\l For example, some multiples oÍ 6 ire 12, t8,
],1, 30, and 3ó The multiples of a number are also multip-les
of any lactor of
:hat number. Two factors of 6 arc2 and 3. The.rr.mb..s12, 18,24,30,aná36
.r re multiples of 6, and they
are also multiples of 2 and 3.

:, me and Composite Numbers


- :ositive integer greater than 1 is either prime or composite, but not both,

' :. prime number is a number that has exactly two different üctors, itself and 1.
_ he set of prime numbers includes

2, 3, 5, 7, 11, 13, 17, 19, 23, . . .

. lc only even number that is prime is 2.


' :. composite number is a number that has more than two different factors. For
,,,:mple:

4, 6,8,9, 10, 12, 14, 15, 16, . . ,

.,. composite numbers.


' ,, number l ir neirher prime nor composite.

:- ..e Factorization
. ::ocess of breaking down a number into the product of two or more
other
.' _.rs is called factoring. Factoring reverses multiplication:
Multiplying:7X3:21
Factoring: 2l :7 X 3

_', : 7 X 3,7 and 3 are factors of2l.


64 Arithmetic Skils and Concepts

The prime factorization of a number breal.<s down the number into the product of
prime numbers. For example:
. The prime factorization of 18 is

18:3X3X2
. The prime factorization of 30 is

30=5X.3X2

EXAMPLE

What is the greatest possible integer Value of n for which 3,


divides gos?
(A) 8 (B) 9 (c) 10 (D) 16 (E) 30

Solutlon
Rewrite 90 so that it contains a power of3 as a factor:

96s: (10 x 3)8 - 10s x 316.

By representing 908 as 10s X 316, yott know that 316 divides Thus, 16 is the
90s.
greatest integer value of z for which 3' divides
908,
The correct choice is (D).
Divisibility and Factors 65

Lesson 3-3 Tune-Up Exercises


i

Multiple-Choice
1. lW4rich number has the most factors? -ü4rich
5. number is divisible by 2 and by 3?
(A) 12 (ö 112
(B) 18 (B) 4,308
(C) 25 (C) 6,122
(D) 70 (D) 23,451
(E) 100 (E) 701,456

]. Y/hen a whole number -n/is divided by 5, All numbers that are divisible by both
the quotient is 13 and the remainder is 4. 3 and 10 are also divisible by
\ü/hat is the value of 1\/?
(^) 4
(A) 55 (B) 9
(B) 59 (C) 12
(C) 65 (D) 15
(D) 69 (E) 20
(E) 79
7. If x represents any even number and
_]. If1 is divisible by 3 and qis divisible by / represents any odd number, which of the
4, then 74 must be divisible by each of the following numbers is even?
following EXCEPT
(A) l+ 2
(A) 3 (B) r- 1
(B) 4 (C) (x + i)(l - t)
(C) 6 (D) y(1+ Z)
(D) 9 (E) x+7
(E) 12
8. For how many differenr posirive inregers
-, Ifthe sum of the áctors of 18 is Sand the p is 105 also an integer?
sum of prime numbers less than 18 is 4 ?
then P exceeds S by what number? (A) Five
(B) Six
(A) 19
(C) Seven
(B) 17
(D) Eight
(C) 15
(E) Nine
(D) 13
(E) 11
66 AríihmetiC Skilts and Concepts

9. lf nis an odd integer, which expression 13. If nis an integer and r? + 5 is an odd
always represents an odd integei?
integer, then which statement(s) must be
(A) (2n - 1)' true?
(B) i+2n+1 I. z2 - 1 is even.
(C) (z - t)' II. z is even.
III. 5z is even.
(D) '+ 1

(A) I only
2
(E)3,+t (B) I and II on.ly
(C) I and III only
(D) II and iII only
10. If b - 1 is a multiple of 4, what is the
(E) I, II, and III
next larger multiple of 4?
(A) É+1 14. ]X4ren the number
ofpeople who
(B) 4L contribute equally to a gift decreases from
(C) k- 5 four to three, each person must pay an
(D)k+3 additiona] $ 10. \iX/hat is the cost ofthe gift?
(E) 4(k - l)
(A) $:o
11. After m (B) '$60
marbles are put into njars, each
jar contains the same number of marbles, (C) $90
with two marbles remaining. In terms of (D) $120
m a1d n, how many marbles were put into
(E) $tao
each jari
15. If n is any even integer, what is the
(r,) ?+z remainder when (n + 1)2 is divided by 4?
n
(A) 0
B)?-z
n
(B) 1
(C) 2
(D) 3
mi2 (E) 4
(L]
-- n
m-) 16. Ajar contains between 40 and,50 marbles.
(D) If the marbles are taken out of the jar
n
three at a time, two marbles will be left in

(E)
- mn the jar. If the marbles are taken out oF the
--
n*2 jar five at a time, four marbles will be left
in the jar. How many marbles are in
the jari
12. \fi4.enpis divided by4, the remainder is 3;
and when p is divided by 3, the remainder (A) 41
is 0. \W&at is a possible value ofp? (B) 43
(C) 44
(Á) 8
(D) 47
(B) 11
(E) 49
(C) 15
(D) 18
(E) 21
Divisibility and Factors 67
Grid-ln
l. -ü/hen
a is divided by 7, the remainder is 3. 'ifhen a positive integer Éis divided
:
5; and when á is divided by7,the by 6, the remainder is 1, \Xi]rat is the
remainder is 4. \X/hat is the remainder remainder when 5á is divided by 3?
when a * bís divided,by 7?
4. As computer circuit boards move along an
2. How many integers from -3,000 to assembly production line in single file, a
3,000, inclusive, are divisible bv 3i qualiry-control inspector checks every
third circuit board beginning with the
third board. A second qualiry-control
inspector checks every fifth circuit board
beginning with the fifth board. If
100 computer circuit boards were pfo-
duced on the assembly line while both
inspectors were working, how many of
these boards were NoT checked bv either
inspector?
i.::: ; }J

Number Lines and Signed Numbers


|]

§*{&
Overview
This lesson reviews the rules for working with positive
and negative numbers.

Ordering Numbers on a Number Line


The size order of real numbers can be pictured by
drawing a rulerlike line called a
Figure 3.2, positi.,e numbers i.. lo.at.d i" i";;;;
::*::]::§
slze :|,":i,:
order ro the right of 0, and their opposites appear in descending size
order tí
the lefr of 0.

-ffi+
_54-3-2_1 0123 45
Figure 3,2 The Real Number Line
on the number line:

' Each number is less than the number that is located to its right. For example,
-3< -2.

' The larger of two negative numbers is the number


that is closer to 0. ,h-or
example, -1 >
-100, since -1 is closer to 0 than is -100.
' If a > b, then -b } -a. For example, since 7 ) 5, -5 > -7.

Multiplying and Dividing Signed Numbers


The, product or quotient of two numbers
with the same sign is positive, and the
product or quotient ol rwo number, wirh drfferenr
,;gn. i" n."ga,iu..
(+) X (+) :+ (+)+(+):+
(-) X (-) : + (-) + (-): +
(+) X (-): - \,/
/L\
\ ./--
! \ /
-
(-)x(+)=_ (-) + (+): -
Thus. rhe square ofeither a positive or a negarive
number is posirive. lFx2 : !,
x may be equal to eirher 3 or - J sjn6g

(3)r:(-3)r:9

68
Number Lines and Signed /Vumóers 69
Multiplying More Than Two Signed Numbers
'üZhen more than two signed numbers are multiplied together, the sign of the product
can be determined by using these rules:

' The product of an euen number of negative factors 1s positiue. For example:

(-z)o : (-2)x (-2) x (-2) x |-2):16


4 áctors

' The product of an odd number of negative factors is negatiue. For example ;

(-z)' : (-z) x (-z)x (-z) :8


3 factors

Adding and Subtracting Signed Numbers


The rules for adding and subtracting signed numbers are summarized in Table
3.3.

TABLE 3.3
7o ArithmetiC Skills and Concepts

Lesson 3-4 Tune-Up Exercises


Multiple-Choice
1. IF 2b : -3, what is the value of l, - 4b? 5. p,(2-5)+(-i,:
(^) -7 (Á) 4?'
(B) -5 (B) *1'
(C) 5 (C) 2P'
(D) 6 (D) 2p'
(E) 7 (E) 4?'

If a is a negative integer and á is a positive 6. If n + 5 is an odd integer, then z could


integer, which of the following statements be which of the following?
must be true?
(A) 1
l. b+ a>0 (B) -1
..._<0
lba (C) -2
a (D) -3
IIL ab <0 (E) -7
(A) None 7. \íhich of the following statements must
(B) I only be true when a<Oandb>0?
(C) II only
(D) IIi only I. a+ b> 0
(E) I and II only II. b- a>0

lf rhe product of five number. is


tll. ,(!\r, o
3.
positive, then, at most, how many of
lb)
(A) I only
the five numbers could be negativei (B) II only
(A) One (C) I and III only
(B) Two (D) II and III only
(C) Three (E) I, II, and III
(D) Four
(E) Five 8. (-2)' + (_3), :
(A) -l2
4. For which value of ,á is the value of (B) -1
k(k - 2) (É-l 1) negative? (C) 0
(^) -2 (D) 1
(B) -1 (E) 2
(C) 0
(D) 2
(E) 3
Number Lines and Signed Numbers 71

12. IfXrepresents the sum ofthe 10 greatest


negative integers and rrepresents the sum
ofthe 10 least positive integers, which of
the following must be true?
Questions 9 and 10 refer to the diagram L X+ Y<o
above. lI. Y- X:2Y
y/hich of the following statements must
III. X2: Y2
9.
be true?
(ö None
(B) I only
I. c2{c (C) III only
II- a2} c (D) I and 11 only
I]].,,1 (E) iI and III only
b
(§ I only
13, If a2b3r } 0. which of the Following
(B) I and II only
statements must be true?
(C) I and III only
(D) I, II, and III L bc>O
(E) None II. ac> 0
III. ab> 0
10. \W&ich of the following statements
must (A) I only
be true? (B) I and II only
I. ad> b (C) I and III only
lI. ab> ad (D) II and III only
(E) I, II, and III
II1.]>1
ab 14. If a2 bj c5 is negative, which product is
(A) II only
always negative?
(B) I and II only
(C) II and III only (A) bc
(D) I, II, and III (B) bzc
(E) None (C) ac
(D) ab
l l. If b' - 3)' :
16, what is the smallest (E) bc2
possible value of72?
15. If a * 0, which of the following
(A) -4
statements must be true?
(B) 1
(C) 7 I. (-a)': al - 2a2
(D) 16 I]. a- b: -(b a
(E) 49 III. a> -a
(A) I only
(B) II only
(C) III only
(D) I and III only
(E) II and III only
72 Arithmetic Skílls and Concepts

Grid-ln
|. If ?2
:
16 and q2 : 36, what is the If -4 '< x'< 2 and,1= 1 - .xí, what
largest possible value o| q - p? number is obtained when the smallest
possible value of7 is subtracted from the
largest possible value of j
]

I
Fractions and Decimals i l:!i!rl§

-,{
§!§l{
, S"*l
*]§ -"'*.§

Overview
A fraction represents a specific number of the equa1 parts in a whole. Thus,
since there are 24 egual hours in a day, 3 hours represents f ofa day.

The number above the fraction bar is the numerator ofthe fraction, and the
number below the fraction bar is the denominator. In the Fraction {,
3 is the numerator and 24 is the denominator. since the fraction bar meáis
division,
3 :z+24:O.\25
24
The decimal number 0.125 is another name for the fraction f,.

Types of Fractions
fhe numerator of a fraction may be less than, greater than, or equa1 to the
de nominator.

' A proper fraction is a fraction such as ]. in which rhe numerator is less than
the denominator. The value of a proper l'raction is always less than 1.
. An improper fraction is a Fraction such as ], in which the numerator is
greater than or equal to the denominator. TÁe value oF an improper fraction is
always greater than or equal to 1.

-quivalent Fractions
Eqűvalent fiactions are fractions that have the same value, The íiactions ;
:quivalent because they name the same part ofa whole:
-d *

of whole rectangle 5 shaded boxes


} 1o bo,1es

.lultiplying or dividing the numerator and denominator of a fraction bv the same


, ,nzero number always
produces an equivalent fraction:

]: 1X5 5 5_ 5--5 : ]
2 2x5 10
and
10 10 + 5 2

73
74 Arithmetic Skills and ConCepts

Reducing Fractions to Lowest Terms


A fraction is in lowest terms when its numerator and denominator do not have any
common factors other than 1. To write a fraction in lowest terms, divide the
numefator and denominator by their largest common áctor.

EXA|VPLE
Write ]9 in lowest terms,
24

SoIution
The largest number by which 1,6 and 24 are both divisible is 8.
16_16+8 _2
24 24+8 3
You may find it easier to perform the division mentally by thinking "How many
times does 8 go into 16?" and "How many times does 8 go into 24?"'§í'rite the
answers above the numerator and denominator ofthe fraction as shown below:

z =?
'ló
3
3

Mixed Numbers
A mixed number is a number such as 2f , which represents the sum of a whole
number and a proper fraction. Thus, 2f means 2 t
f.
. To change an improper fraction into a mixed number, divide the denominator
ofthe improper fraction into the numerator. §(/rite the quotient with the
remainder expressed as a fraction in lowest terms. For example:
2
l9 :79_7:7| \--19
7 '
-14
._ Remainder
-
Since the quotient is 2 and the remainder S,| - Z}
"
. To rewrite a mixed number such as 3] as an improper Fracrion, multiply 3 by 9,
add rhe producr to 5. and then write the sum over 9:

,5 (3xq) +5 2- 5 32
99
place value jn Decimal Numbers
The place value of each digit of a decimal number is 10 times as great as the place
value ofthe digit to its right.
Fractions and Decimals 75

8253 769

_rl
-1l1 t t L
Thousands Thousandths
Hundreds l L H Lrnr]rer]ths
Tens l T.r,,h,
Ones

Thus, the decimal number 8253.7 69 is equivalent to

8253.769:8 X 1000+2 X 100+5 X 10+3 X 1 +7 X0.1 +


6 X0.01 +9 X0,001

Comparing Decimals
To compare two decimal numbers:

' \flrite one decimal underneath the other so that the decimal points and digits
rvith the same place value are aligned.

' Ifone decimal is shorter than the other, add zeros to the right ofthe last digit of
the shorter decimal until the decimal numbers have the saÁe number of dilits.

' Start from the left and compare the digits that are in the same column. Stop
rvhen you find unlike digits. The greater digit indicates the decimal with thi
gleatef value.

-XAMPLE

n Which of the fo||owing lists are the numbers Written in order from least to grea.test?
A) 0,0361, 0.306,í, 0,306
B) 0,3061, 0,306, 0,0361
C) 0.0361, 0.306, 0.3061
D) 0,306, 0.3061, 0.0301
0.306, 0.0361, 0,3061
=l
:_ lton
::te the three decimal numbers that are being compared, one underneath the other,
. ::,:at digits with the same place value are aligned in the same vertical columns:

0.0361
0.3061
0,3060
T_aaa o.
_ compare the digits ln the same columns. The least decimal is 0.0361. Álso.
'!| < 0.3061 since 0 1 in the last decimal position.
76 Arithmetic skills and Concepts

The correct afrangement, from least to gleatest, of the decimal numbers is as


follows:
0,036i,0.306,0.3061
Thus, the correct choice is (C).

Rewriting Fractions as Decimals


To rewrite a íraction as a decimal number, use a calculator to divide the denominator
of the fraction into the numerator of the fraction.

EXAMPLE
Flewrite f1 as a decimal-
4o

SoIution
Using a calculator, divide 40 into 11:

]1 :r, + 40 _ 0.275
40

Comparing Fractions
To compare two fractions, use these facts:
. If two fractions have the same denominator but different numerators, then the
larger fraction is the fraction with the greater numerator. Fot.r"mple, ] > f.
. If two Fractions have the same numerator but different denominátofs, then the larger
fraction is the fraction with the smaller denominator. For exampilr,+ +,
'
. If two fractions have different numeratoís and different denominators, then the
larger fraction can be determined by comparing the decimal values ofthe two
fractions. For example, .]-! } a since

11
0,47826 ano
,7 o-46666
23 r5

Multiplying and Dividing Decimals by Powers of 10


Multiplying or dividing a decimal number by a power of 10 (10, 100, 1,000, etc.)
affects only the position oF the decimal point of the number.
. To multiply a decimal number by a power of 10, move the decimal point one
place to the right for each 0 in the power of 10. For example,

6.25 X 1,000 : 6.Ux 1,000 : 6,250.


. To divide a decimal number by a power of 10, move the decimal point one place
to the lqft fot each 0 in the power of 10. For example,
8.24 + 100 :08.24 + l00 - 0.0824.
Fractions and DeCimals 77

Multiple-Choice
] , \üíhat part
of an hour elapses from 4. The elapsed time from 1 1:00 ,c.^a. to
4:56 r.1,1. to 5:32 r,.v.?
3:00 p.l.r. on'Wednesday of the same

i day is what fraction of the elapsed time


1
(^)
from 1 l:00 Á.M. on \Wednesday to
3:00 p.nt. on Friday of the same week?
(B) 1 1
2 (A)
D
(C) 1 1
5 (B)
ú
(D) ? 1
3 (C)
1

(E) 1 4
4 lo) li
If each olthe fractions j. f. f ;, ;n lo*.r. (E)
1

terms. r,r hich of the following couid be rhe 12


value of É?

(ö 48 5. In which number is the digit 3 in the


(B) 49 hundredths place?
(C) 50 (A) 300.000
(D) 51 (B) 30.000
(E) 52 (C) 0.300
(D) 0.030
3. \X4rich number has the greatest value? (E) 0.003
(A) 0.2093
(B) 0.2908 6. In which árrangemenI are the lractions
(C) 0.2893 listed from least to greatest?
(D) 0.2938 918
(E) 0.2909 (A)
19' 2'15
18 9
(B)
2'I5'19
9 81
(C)
1,9 1,5 2
19 8
(D)
2'19'15
819
(E)
l5'2'19
Aríthmetic Skilts and Concepts

4 nr.
After the formtla
v = has been lO. IrN \Z:Z" j,,h.oa_
evaluated lor some positive value
3
12 12 14 N
of.r, rhe
rormula ts.again evaluated using
one-half (A) 8
ot rhe original vaiue of r. The
oI, / js whal fractional parr
new value
of the original
(B) :1Á
va]ue of |? 3

!
; (C)
1

{a) 7

(B)
1
(D) i
l4
'
1
(E) ]
(C)
8 6
l
(D) ll, IF n: 2.5 y |0r<, rhen fi -
4
,

(A) 0.5 X 10,


l (B) 9.5 y 16t:
íF.) -
) (C) 5 x tofi
(1J) ) Y l0\
8. (E) § a 19I:
Each inch on ruler l is marked in equal
l
--inch unirs. and each inch on ruler B 12. If 1isa real number Y : ' - 2
is marked in equal a-inch ^nd
' 1)-"' units. \X/hen rh_en x CANNOa.o"rr *rl'.nir,r-ri
ruler l is ured, a side-ofa rriangle following numbers?
(A) -3
measures 12 ofthe ]-inch units. §fhen
8
(B) -z
ruler B is used, how many 1-i.,.h unit, (C) 0
l2 (D) 2
will the same side measurei (E) 3
(A) 8
(B) tz 13. If eiglrt pencils cost $0.42, how
many
(C) la pencils can be purchased with $2.
10i
(D) 20 (A) 16
(E) 24 (B) 24
(C) 30
9, 6o+z+!+ 3 : (D) 36
8 500 (E) 40
(A) 60.256
(B) 62.43
(C) 62.506
(D) 62,53
(E) 62.560
Fractions and Decimals 79
14. A store sells 8-ounce containers
oforange 15. In the repeating decimal
juice at $0.69 each and 12-ounce
containers oforange juice at
$0.95 each. 0.31752:0,3175231752 ,. .
How much money will be saved by rh_e set o_f digirs 3l75Z repeats endlessly.
purchasing a total of48 ou.r... of o.".rg. ]Vhich digir is in rhe
juice in l2-ounce rather than 8-tlunce 968th place to the
right of the decimal poinr?
containersi
(A) 1
(A) $0.24
(B) 2
(B) $0.32
(C) 3
(C) $0.34
(D) 5
(D) $0.48
(E) 7
(E) $0.56

Grid-ln
.l. -^4^
l| *
5
= what digit does Á
- ; ',
If the charge for a taxi ride is $2.50 for
represent?
the first ]1

mile and $0.75 for each


2
2. 01 a certa_in map, l,5 inches repfesens
distance of 45 miles, If two points on
a
addirional ]
1

mile, how many miles did


the tJ
map are 0.8 inch apart, how many miles
the taxi travel for a ride that cost $10,75?
apart are these two pointsi

3. Four lemons cost $0.68. Át the same


5 One cubic foot of a certain metal weighs
rate, 8 pounds and costs $4,2O per pound. [f
1 pound oflemons costs
$1.19. How ,l cubic foor is equivalenr ro l
many lemons
rypically weigh i poundi
}28 cubic
inches, what is the cost of 288 cubic
inches of the same metal?
§§§rls
Operations with Fractions
tr

í§ #t
*§-m
Overview
Fractions are multiplied, divided, and combined (added or subtracted) according
to the following rules, where a, b, c, and /stand for real numbers with b, d + ü

a ,. ( aY
.-,\ -
c
._a ]- _c : aXc
b d bxd b- b b
a t a..d a<d .!*! ad!bc
b d b c bxc bdbd

Multiplying Fractions
To multiply fractions, write the product of the numerators over the product of the
denominatofs. Then write the result in lowest terms. For example:

4.5 20 20+4
-x__
9 8 72 72+4
5
18

Sometimes it is easier to divide out pairs of common factors of any numerator and
denominator before multiplying. For example:

!
2 1

\ 2
X
3
ts 5
15

Finding a Fraction of a Number


To find a fraction of another number, replace the word a/with the times symbol.
Then multiply.

EXAMPLE

What is of 20?
f
Solution
1of20: 1x20 :3x4:12
5 5

80
operations Wíth Fractions 81
EXAMPLE
After John Spends oí his salary on food and ? of what remains on clothes, What part of
;
his original salary does he have left?

Solution-]
. j
After John spends
remalns.
of his original salary on food,
](: 1 - }) of his salary

. John spends ] of what remains on clothes. Since


)1)
i,rf
)4J
_1 _a1 1:!
42
John spends ] of his original salary on clothes.

' ,{fter John spends


} of his original salary on food and } of his original salary on
clothes, he r,* r(:1- + - j) ofhis original salari left.

3c ution 2
. :;k a convenient amount as
Johnt original salary. Choose a number that is divisible
,,
+. for example, $100.

' lolrn spends } of his original


_salary
on food. Since
}X $100 : $25, he spends
S]5 on food, so he has $75 left.

' 'ohn
spends i
or $75 on clothes. Since X $75 ] : $50, he spends $50 on
:iothes, so $25 remains from his original salary.

' :ince fft : i,Joh"has } ofhis original salary left,

- . ding Fractions
livide a fraction by another fraction, invert the second fraction by interchanging
:.Jmeratof and denominator. Then multiply. For example:

4 2
123\i _- Á 4x2
35 ],4 .r5
5
^
T1
5X1
, ,Dl E

,l2
-
7

-'1n
_123
J+--_X
71124
82 Arithmetic Skills and Concepts

Reciprocals
The reciprocal olany nonzero numberlris 1. For example, the reciprocal of3 is
+.
of any nonzero fraction (b + qk !. For example, the reciprocaí
T f
or5 :..T-*'
ts j,

The product of any nonzero number and its reciprocal is 1,

Powers, Square Roots, and Reciprocals of Proper Fractions


Ifa number is berween 0 and 1, then:
. The square of the number is lts than the original number. For example:

í1)'.l,i.,..]<f
\3/393
In general, as the exponent of a proper fraction increases, the value of that
expression decreases.
. The square root ofthe number is greater than the original number;

JO25 > 0.25 since .625 :0.5 and0.5 > 0.25


. The reciprocal of,the number is greater thanthe original number. For example,
.-
the reciprocal orl i, Z, which is greater th",l 1.
2-2
In each of the above cases, taking powers, foots, of reciprocals of numbers that
are
greater tban 1 gives results that are exactly opposite, ,ho*., in Táble 3.4.
".

Table 3.4
Rules of Powers, Square Roots, and Reciproca|s
x is between 0 and 1. ffi
x is greater than 1. X2>x ,]x < x

Adding and Subtracting Fractions


To add or subtract Fractions with like denominators,
. \üZrite
the sum or difference of the numerators over the common denominator.
. Express the result in lowest terms, if possible.
operations with Fractions 89
EfiMPLE
AOa:Z+1
,l2 12

Solution
7 1 7+I
_-|_ 82
12 12 12 723

Write$ - as a single fraction.


f,

3, _, 3x-x
88 8
2x
.8
x
4

combine fractions with unlike denominators:


Find the Zeast Common Denominator (LCD) of the fractions. For example, the
LCD of 2 and á is 24 sjnce,24 is the smallest number that is divisible by
l2
both 8 and 12.
Change each fraction into an equivalent fraction that has the LCD as its
denominator.

vrite the sum or difference of the numerators oyer the common denominator-

Express the result in lowest terms, ifpossible.

íAMPLE
aOO: 1+?
35

LCD is 15. Charrge each fraction into an equivalent fraction that has 15 as its
minator. Multiply the first &action by 1 in the form of
2, and multiply the
fraction by 1 in the form of 3. 5

:-::;(;)- ;[;)
56
|5 15
: 11
I5
84 i,nthmetíC Skills and Conceots

Operations with Mixed Numbers


Mixed numbers have whole number and a fractional part.
a

' To multiply or divide mixed.numbers,


firsr change the mixed numbers to
improper fractions. Then foljow
,rr. .rl.,-ro.
-ui lö;'il dividing fractions.
EXAMPLE

Multipry: rj x s1

Solution

tZxe! :-1l5 X I0 _25


83 8 34
EXAMPLE

Djvide; 51 + 19

SolUtion

;1= t? = 11 . 11 11,,-:zl8
282 8 2 il
' To add or subtract mixed numbers,
write the second mixed number under
mixed number. IFthe denominato.. the first
rrr. r.".,i.", find the LCD
and change the fractions into "r
equivalent Fru.,ion. *i,n"..;;fr;r,,
,rr" i'CD
Add or subtract the fractions. Then d.nomi.r"to..
add ".
,ri,.-,','rr. *nlle numbers.
".

EXAMPLE

43 s?
loo: z1+

Solutíon

1_.3
-,

412
, -2 ,8
312
_ 11
12
operations With Fractíons 85

EXAMPLE
SuOtract o? - a1
99 ]

Solution

,2 -9
)-
+ 2: ,11
)_
9 99
.4 ,4
- 4-
9 9
.7
l-
-
9

Simplifying Complex Fractions


A fraction that has another fraction in its numerator or denominator is called a
complex fraction. To simplify a complex fraction:

' Find the LcD ofthe denominators ofthe fractions that are contained in the
numerator or denominator of the complex fraction.

' Multiply the numerator and denominator of the complex fraction by the LCD.

' Simpli$, the resulting fraction.

E(AMPLE
Simplify: 4
ó
,
Solution 1

Multiply the numerator and the denomin ator by 2:

2,,4 2x 4 9
2 3 2x3. 3
22
Solution 2
Divide the numerator by the denominator:

!:n*1:lx?:!
3233
2
86 Arithmetic Skills and Concepts

Multiple-Choice
l. John compleres a race in 91 minures. 4. \Xil,"t i, l of ? of D?
33 43
and Sreve finishes the.r.. ,r.. in 7f,
(^) 2
minutes. How many seconds after Steve (B) 3
finishes the race does John complete the (C) 4
race? (D) 6
(E) 8
(A) 72
(B) 90 )
(C) sz 5. \{/hatisfof3 x5Y7?
(D) 95
(E) 100 (A) 6
(B) 9
2.
1
(C)
Multiplying a number by j and then
(D)
10
12
]
dividing rhe resuIt by a is equivalent to (E) 15
4
performing which of the following
operations on the number?
6. For which value of z is ! a,a l
_?
4 3

(A) Multiplving bv a
1
rA)
' 1 5

J ),4

(B) Dividins bv ? '24


íB,)
6
-
(C) Multip|vins bv i
tJ rc)L
,24
(D ) Dividins bv ?
8
(E) Multiplying by 3 ,
(D)
-)-4
3. A conyenience store buys keychains (E),)49
wholesale for $0.75 apiece and then sells -
them for $1.25. Assuming no othet costs,
how many keychains must be sold in
order to make a profit of $250?
(A) 50
(B) 100
(C) 200
(D) 500
(E) 600
operations With Fractions 87
7 . If 0 < x { 1, then which of the following
statements must be tfue? 9. An item is on sale on Mond,ay at j orr,l.
list price of / dollars. On Friday, the sale
I.--x10
]

.rc
price of the item is reduced by of th.
}
II.12+x'>x current price. §7hat is the number of

III. -5 > x' dollars in the final sale price of the item?

(A) d
(A) I only '2 -
(B) II only d
(C) III only íB)
'3 -
(D) I and II only
(E) I and III only
G\+4
8. If h:!a , which expressior, .qu"ls 1? (D);d
o
b d
(E)
'8 -
(.\) o'
b
b
10. If a and b areintegers Í : ?3 and,
(B)
-ac 5 < b <
are there for z?
13, how many possible values

(C) a
(A) One
bc (B) Two
(C) Three
(D) b,
a
(D) Four
(E) Five
(E) +
abt If
11.
f is a positive fraction less than 1,

which of the following fractions MUST


be greater than li

íl)'
rel
lt)
B illl
lb)
a
(c)
*
(D) a,|2
,
b+2
@)?a
i
Arithmetic Skills and Concepts

z 13. The stat€ment a3 1 a2 1 a is true when


12. Juan gives - ofhis p pencils to Roger and
I. a>l
then gives ] oFthe pencils thar he has II.0<a<1
4
III. -1< a< 0
left to Maria, In terms of p, how many
pencils does Juan now have? (§ I and III only
(B) Il and III only
tA) f p (C) II only
ö (D) III only
(E) None
(Bl ]p
4,

tcl f p
3,

tD) ]p
2,
§
(E) 1p
6,

Grid-ln
r,-.l-
, r:(t,]]ír,]lÍ, *') í l)
l, í ] :
,/ ,
whar is rhe value of7 o
l. 2)\ 3/\ l--I n)
when x:
}? If each factor in the above product has the

r..- (r + *rr*. ,á represents all the


V/hat fraction of ]-0 i, 1 l })
96 consecutive integers from 2 to 17, what is
the value ofp?

, -"(:) : 4, what is the value of


5, If c- I ++dand a: t +|+ }+ }
+

*rr", is the value of c , d?


f,
",[1),
Fraction word problems "i' . a: ;.'

w
',l
*}1}
|4i.
*s(t '{

Overview
Many rypes of fraction problems can be solved either by:
. forming a fraction in which the number in the numerator is being
compared with the number in the denominator or
. finding the whole when a fractional part of it is given.

Finding What Fraction One Number ls of Another


To find what fraction a given ,r.;.-b.. "i" another number, form the fraction

Part zj
,/\íTlOLE
Irr the fraction above, the'part" is the given number and the "whole" is the rrumber
ro which the part is being compared.

=XAMPLE
There are 15 boys and 12 gir|S in a class. The number of boys is What fraction of the total
number oí students in the class?

Solution
:ince the number oF boys is being compared to the total number of students in the
_:.tss, form the fraction in which the number of boys is the numerator and the total

'rmber of students in the class, boys plus girls, is the denominator:

Boys 15 15 5
\Whole class 15+12 279
: rding the Whole When
a Part of lt ls Known
:. of some unknown number is 12, then multiplying 12 (the 'part") by the reciprocal
: :he given fraction gives the unknown number (the'whole"). Thus,

vthol: lÍ x 7. x7:21

89
í :,-::a.Cepts

\ou can a]so find the unknown number by following these steps:
. Find r ofthe number. Since 1 ofthe unknown number
is 12, ] ofthe
unknown number is \
1
of 12, which is 3.
. Multiply to find the whole number. Since -l of the unknown number
the unknown number is 7 x 3 : 21. /
is 3,
I of

Hence, the unknown number is 21.


The above procedure can be quickly applied using the following
shorthand notation:
4 .-
-
1)
-. told that ! of thrunknown ,r,rmbe. is rZ.]
7 [".u
1
!*n :3 unknown nu-b..l, a.l
[;**.
-7
4
1
-7
7X 3 : 21 rn,unknown .,umb.. i. Zt.l
L+*

EXAMPLE
Of the books that are on a shelf,
] are math books,
] are science books, and the remaining
books are history books. lf the shelf contains 1o history
books, how many books are on the
shelí?

So|utíon 1

. Math and science books account for 7 of the books on the


shelfsince
l2
1+ 1437 +_:-
3 4 12 ],2 12

History bools make up


;(: - {) of the total number of books on the shelt,
Since there are 10 historv books,
'
1h6 pl6§|grn reduces to answering the questiol:
"} of what number is 1'0?"

-=l0 unknown numb.. i. ro.]


12 [;*rn.
a=]xl0:) unknown .,u-b.. i, z.]
125- [;*,n.
1)
-=12>,2:24 *hol.,,,-b.. i. 24.]
[;*.n.
The shelf contains a total of 24 books.
Fraction Word Problems 91
SoIution 2
l

Use algebra by letting x represent the total number of boola on the shelf

an equation that states that the sum ofthe numbers of math, science, and
}Vrite
history books is .,c
xx :
-+-+
34 10 x

To eliminate the fractions from t}ris equation, multiply each member of the
equation by 12, which is the LCD of 3 and 4:

:
"(:)-"(1)-
4x *3x
12(10)

:
12,
+ 120 12x
l20 :12x-7x
120 :5x
120
5
92 ArithmetiC Skils and Concepts

Lesson 3-7 Tune-Up Exercises


MultipIe-Choice
l. tr j or, number is 6. whar is of rhe 5. What is the sum of all two-digit whole
$
same number?
numbers in which one digit is
J
of the
(A) 8 other digit? 4
(B) tz
(A) 77
(C) 14
(B) 102
(D) 16
(C) 129
(E) 24
(D) 154
(E) 23l
2. AFrer Claire has read rhe 6rsr ola
f book.
there are pages left to read. How many 6. Boris ha, Ddollars. lfh. lcnds j- oirhis
]20
pages of the book has Claire readi
amount of money and then spends
(A) 160 { of
(B) 200 the money that he has left, ho-
-".,f
(C) 240 dollars, in terms of D, does Boris now have?
(D) 300
(E) zzo (E) D
2
.r. ! ,, .!,/
-- .
ls 6 oi whar numberi
oI
, (B)'j D
9
-
(A) 5
(B) 10 (C) 2-]
(C) 12 4.)
(D) 15
(E) 20 rniio-]
43
4. 1 of
If aclass that includes 10 girls ar.e tEl 1]o
1)
boys, how many students are in the class?

(A) 15 7. The value obtained by increasing a by _l


(B) 20
of its value is numerically equal to the
(C) 2l
(D) 25 va]ue obtained by decreasing b6y ! of
(E) 30 its ralue. Which equarion
taCti ""or.rr".2,hi,

(A) 1 ,2a : 0.j b


(B) 0,2a: O.Jb
(C) 0.8a: l.Jb
(D) 12a: |.Jb
(E) a+0.2-b-0.5
Fractíon Word Problems 93

many tlmes must a ,o8gef run 2


1. Of
1_1\\,
1 the 7 5 people in a room, are college
rround a circular l_mile track in order to )
4
,9
graduares. lf4 ofrhe stuclents who are
have run 3lmiles?
2 not college graduates are seniors in high
(A) 9 school, how many people in the room are
(B) 12 neither college graduates nor high school
(C) 14 seniors?
(D) 15
(E) 16
(A) 15
(B) 20
(C) 25
A chocolate bar rhar weish" ,16a ofa (D) 36
(E) 40
pound is cut into seven equal parts. How
much do three parts weigh?
12. If 2 of J of a number is 24. what i, 1
(A) l 2
oo.,r,d 344
1t2' of the same numbef?
)7 (A) 8
(B) pound
11), (B) 12
- (C) 16
,^.
(L) 16 (D) 20
Pound
-oJ (E) 24

(D) a1-7 pound \3. A man painrs ,4 f of" hour. in 2 days. Ifhe
ó3^
continues to work at the same rate, how
R5
(E) -:-' pound much more time will he need to paint the
11), rest of the house?

At a high school basketball g"-e, 3 of '4 1


(A) d^y
,
tlle students who arrended were seniors,
G) ] d^y
] oFrhe other- srudents who atrended 2
3
lvere juniors, and the remaining
80 students who attended were all
G)?3 dry
sophomores. How many seniors attended (D) 1 á^y
this game?
(E) j
Z,
(A) 120 days
3
(B) 175
(C) 180
(D) 210
(E) 300
94 :, :nmetic Skitls and Concepts

1,4. A water rank is J rull. AILer l2 gallons are 17. At the beginning ofthe <1ay, the prices
r of
stocks l and B are the same. At the
poured out. rhe tank is end of
]) Fuli. When the the day, the price of stock l has increased
rank is lull. hor.l many gallons ,]
r,y j oI tts original price and rhe price
of water of
does it hold?
stock B has decreased b",
, f of its original
(^) 25 ,
íB) 3) price. The new price of stock l is what
(C) 35 fraction of the new price of stock B?
(D) 42
(E) 45 (,\) z
10

|5. ln a school eJection. Susan received 2.


o[ (B.) 1
l1
rhe bailor> casr. Mary ,...iu.d L oF,Í"
(C)
remaining balIors. and Bill rece5iued
the other votes. If Bill received
aIl of -l0
how many votes did Susan receive?
48 votes,
(D)
' !l0
(A) 75
(B) 90 (E) 1-!
(C) 120 9
(D) 150
(E) l80
l8. 1
Aírer -- oFrhe people in a room leavc,
l6. AIrer Arlene pumps gas into
the gas rank
three people enter the same room.
The
ol hcr car. the gai gauge moves From number ofpeople who are now in the
room, assuming no other people enter
e*u.Ll} j Full ro exrcrly j FuIt. tf,h. g", or

costs 1.50 per gallon and Arlene is


leaue, is j ofthe original number of
$
charged $ I 8.0_0 For rhe ga,. what people who were in the room. How
is rhe many
capJclry. tn gallons. ofthe gas people leit the roon?
rank?
(A) 24 (A) 9
(B) 20 (B) l8
(C) 18 (C) 24
(D) 16 (D) 27
(E) I5 (E) 36
Fractíon Word Problems 95

Grid-ln
l . After a number is increased by
{ oF irs 2, In an election. '21 ofthe -"l. uot.r, 2
".rd 3
value, rhe result is 24. \X/hat wal the
ofthe female voters cast theil ballots for
original number?
candidate l. If the number of |emale

voters was 11times the number of male


2
voters, what fraction of the total number
ofvotes cast did candidate l receive?
Percent

Overview
Since there are 100 cents in 1 dollar, 15 cents represents 15 percent ofa dollar.
Percent means the number of hundredths or the number of parts out oí 100.
Instead of writing the word percent, you can use the symbol o/o. Thus,

Po/o : _P
100

Rewriting Fractions and Decimals as Percents


Since fractions, decimals, and percents all represent parts of a whole, you can change
from one of these types of numbers to another. Fractions and decimals greater than
1 are equivalent to pefcents greater than 100.

. To rewrite a decimal as a percentJ move the decimal point two places to the
right and add the percent sign (%). For example:

0.45- 45o/o 0.08 - 8% 1.5 - 1.50 : l50o/o


UlJlJ
. To rewrite a fraction as a percent, first change the fraction to a decimal. Then
change the decimal to a percent. For example:
!1 - 0.75 2.5o :250o/o 4: 4,00 - 40Oo/o
4Ll
Rewriting Percents as Decima|s and as Fractions
You may need to express a percent as a decimal or as a fraction.
. To rewrite a percent as a decimal, drop the percent symbol and divide the
number that remains by 100. For example:

85 ]
"' :0.85 1o/o_ )
1:100- 0.75 ' :0.0075
85olo - 100 4 4 100
. To rewrite a pelcent as a fraction, drop the percent symbol and make the
numbel that remains the numerator of a fraction whose denominator is 100.
For example:

,l
1

27 l "/o 7.5 7,5 X l0 '75


l/"/o - /
100 2 100 100 100 X 10 1,000

96
_ Percent 97
Finding the Percent of a Number
To find the percent oí a given number:
. Rewrite the percent as a decimal.
. Use your calculator to multiply the given number by the
decimal form of the
pelcent.

EXAMPLE
What is 15% oí 80?

SoIution
Rewrite 15olo as 0.i5. Then multiply:
l5o/o of 80 : 0.15 X 80: 12

Finding an Amount After a


Percent Change Y!p
Iia 20% tip islefton a restaurant bill.of$s0, then to find
li,l";:lil^, resu|t of increasing or decreasing a
rhe total amount of the bill including the tip, do
th. "r,iu*uya percent in one step by multiplying the
tbllowing: number by th,e total p".."n,ug",' '

. STEI 1 : Find the amount of the tip:


. lí 8O is increased by 2Oó/o, the total percentage is
10070 + 2090 - 12Oo/ó, so thefina| amountis
$80 x 0.20: $16 80 >< -|.20 : 96.

' sl'EP 2: Add the tip to the bill: $ao + 616 : 696 , lf 50 is decreased by 30%, the total percenlage
is 100% - 3Oo/ó : 7Oo/ó, so the flnal amount is
:MMPLE 50 X 0.70 : 35.

lf the
length of a rectangle js
increased by 30% and its Width is increased by 1o%, by
What percentage Will the area oí the rectangle be increased?

(A) 33%
(B) 37o/a
|cJ 40%
(D) 43%
(E) 45%

Soiution
'rck,easy nttmbers for the length and width of the rectangle. Assume
the length and
,,lotn of the rectangle are
each 10 units, so the area of the original rectangle is
.0 X 10 - 100 square units.
'fhe total percent
' increase ofthe length is 100% + 30o/o : ],3Oo/o.The length
of the new rectangle is 10 X 1,3 : 13.
. The total percent increase of the width is 100% + :
I0o/o ll}O/o.The width
ofthe new rectangle is 10 X 1.1 : 11.
. The.area of the new rectangle is 13 X 11 = 143 square units.
Compared to the
origina1 area of 100 square units, this is a 43% increase.

_ he correct choice is (D).


98 Arithmeti1 Skills and Concepts

Finding the Percent of lncrease or Decrease


'ü/hen a quantiry goes up or down in value, the percent of change can be calculated
by comparing the amount of the change to the original amount,
Amountofchange
percentofchans.
- - \ l00oó
Original amount

EXAMPLE
lf the price of an item increases fíom $70 to $84, What is the percent oí increase in price?

Solution
The original amount is $70, and the amount of increase is $84 - $70 : $14.

Ámount of increase
Percent of change : X 100%
Original amount
:14 X 100%
70
:1 X 100o/o
5
: 20o/o

Finding an Original Amount After a Percent Change


If you know the number that results after a given number is increased by Po/o, you
can find the original number by dividing the new amount by the total percentage.

New amount after an increase of P%o


original amount - 10oo/o l Po/o

Similarly, if you know the number that results after a given number is decreased by
Po/o, yott can find the original number by dividing the new amount by the tota1
pefcentage.

New amount after a decrease of po/o


Original amount :
I0Oo/o - Po/o
Percent 99
EXAMPLE
A pair of tennis shoes cost $48.60 including sales tax. lf the sales tax rate is 8%, What is the
cost of the tennis shoes before the tax is added?

Solution
The total percentage is 100o/o + 8o/o : I08o/o.

cost with tax included


cost oftennis shoes
Total percentage

48.60
1080/o

48.60
1.08

Use a calculator to divide: : 45


The tennis shoes cost $45 without tax.
1oo Arithmetic Skills and Conoepls

Lesson 3-8 Tune-Up Exercises


Multiple-Choice
1 5, After 2 months on a diet, Johns weight
1. \What is 20olo of 1 of 15? dropped from 168 pounds to 147 pounds.
3
(A) 1 By what percent did ]ohn's weight drop?
(B) 2
(A) I2-o/o
]
(C) 3
)
(D) 4
(E) Z
5 (B) l4-0k
7
2. \Which expression is equivalent ,o Zoo' (C) 2lo/o
5
(A) 0.40 (D) 25o/o

(B) 0.04 4
(E\ 28-o/o
(C) 0.004 7
(D) 0.0004
If 1 cup of milk is added to a 3,cup
(E) 0.00004 )1
milk. *har
i
mixrure that is flour and
{
3. In a movie theater, 480 of the 500 seats
percent of the 4-cup mixture is milk?
were occupied. §íhat percent oF the seats
were NOT occupied? (ö 80%
(B) 75o/o
(A) 0,4%o
(C) 7ovo
(B) 2%o
(D) 650/o
(C) 4a/o
(E) 600lo
(D) 20o/o
(E) 40%
7. Ifthe result of increasing aby 3000/o of a
is ú, then a is what percent oí /?
4. In a certain mathematics class the part
of the class that are members ol the math (A) 20o/o
club is 50% of the rest of that class. (B) 25o/o
The total number of math club members
,333
1
(C,) o/o
in this class is what percent of the entire
class? (D) 40o/o

(A) 20o/o
E) ee?yo
(B) 25%o 3

Cl BlVo
j 8. Nter a 2Oo/o increase, the new price of a
(D) 50% radio is $78.00. 'ü7hat was the original
(E) 75o/o price of the radio?
(A) $15.60
(B) $60.00
(C) $62.40
(D) $65.00
(E) $97,50
Percent 101

9. After a discount of 15o/o, the price of a 13, A used-car lot has 4-door sedans, 2-door
l shirt is $5 1. -ü/hat was the original price sedans, sports cafs, vansJ and jeeps. Of
of the shirt? these vehicles, 40o/o are 4_door sedans,
(A) $44.35 25o/o are 2-door sedans, 207o are sports
(B) $58.65 cars,100/o are vans, and 20 of the vehicles
(C) $60.00 are jeeps. If this car lot has no other
(D) $64.00 vehicles, how many vehicles are on the
(E) $65.00 used-car lot?

(ö eoo
10. Three students use a computer for a total (B) 400
of 3 hours. If the first student uses the (C) 480
computer 28o/o of the total time, and the (D) 600
second student uses the computer 52o/o of (E) 1,000
the total time, how many minutes does
the third student use the computer? I4. Jack's weight first increased by 2Oo/o and
(A) 24 then his new weight decreased |ly 25o/o.
(B) 30 His final weight is what percent of his
(C) 36 beginning weight?
(D) 42 (A) 95o/,,
(E) 50 (B) 9z.5oto
4 (C) gOvo
l1. \X/har is 20olo of 25oto of :) ? (D) 88.5%
(A) 0.0025 (E) 85%o
(B) 0.004
(C) 0.005 15. If ais 40o/o of b, then ú exceeds a by what
(D) 0.04 percent of a?
(E) 0.05 (A) 60Vo
(B) 100%
]2. In a factory that manufactures light bulbs, (C) 140yo
0.04o/o of aII light bulbs manufactured are (D) 150%
defective. On the average, there will be (E) 250o/o
three defective light bulbs out ofhow
many manufactured?
(A) 2,500
(B) 5,000
(C) 7,500
(D) 10,000
(E) 12,500
1o2 Arithmetic skílls and Concepts

Grid-ln
1. A store offers a 4o/o discount if a consumer During course registration, 28 students
pays cash rather than paying by credit enroll in a certain college class. Afier three
card. llthe cash price ofan item is boys are dropped from the c\ass, 44o/o of
$84.00, what is the credit-card purchase the class consists ofboys.'ff/hat percent of
price of the same item? the original class did girls comprise?
Answers to Chapter 3 Tune-lJp Fxerclses í03
i

Answers to Chapter 3 Tune-Up Exercises


Lesson 3-1 Earning $8 an hour, he will lreed to work
MULTlPLE-CHoICE # - 30 hours to have enough money to
buy the stereo system.
1. (D) ifa :9 X 23
and b: 9 X 124. then 8. (E) Since 24 hours -| 12 horrrs f 3 hours :
\ b a ,)y 12+ 9 r2J 39 hours, break down the problem by figuring
\-] You ,, an u\e your calculátor to do the ,uirhme- out the dme 24 hours ago, 12 hours before
tic on tlre liiht side of the equation, but it's that time, and then 3 hours earlier, Ifthe prev
easier to use the reverse ofthe distributive law: ent dme is exactly 1:00 p.M., then 24 hours
b a: 9024 - 23) ago it was a]so 1:00 p.]\,í., and 12 horrrs before
: 9 (101) that time it was 1:00 a.v. Three hours before
- 909 1:00 e.tl,t. the time was 10:00 r,.lt.
íB) fo figure out by what amount quanriry 9. (A) If r # 0 : z, then the symbol # can
l exceeds quantity 4 calculate l - B (8 X represent either addition or subtraction, but
25) - (15 X l0) : 296 150 : 50 not multiplication or division. Sir-rce it is
(C) Since l quart : 32 fluid ounces, also given that r# s: s# r, the operation #
quarts : 5 x 3z _ 160 fluid ounces. If
5
is commutative. since addition is commu_
each container holds ] 6 íluid ounces, then rarive and subtra.tion i5 not commulativeJ
# - tO containers are needecl to holrl 5 the symbol # represents only addition.
quarts oí milk. 10. (D) Each product contains 6, so 6 can be
1. (B) Solution 1: If the current odometer read_ ignored, Then u: 36, r: 35, and 7:
ing of a car is 31,983 miles, then the next 32, so 1- x <, w,
', mileage reading in which at least iour digits I I. (B) ll 2x,t 1 <, 29. the expression x !
are the same will be 32,222, Hence, the lást will have its greatest possible value when
nunrbel of miles that the car must travel
7 has its least possible value and x has its
before the odometer displays four digits that maximum value.
are the same ts 32,222 - 31 ,983 : 239. o Since is an integer and ) 4, the least
7 1
,9olution 2: Find the sum of 31,983 and the possible value lor;rl is 5.
number given in each answer choice begin,
' If ),:5, then 2l + 1 < 29 becomes
rring with (A), Stop when you
Bet a sum 2x i 5 129 so 2x 124 and, x 1 12,
(31,983 + 239) in wlrich at least four of . Since y is an integer and x ( 12, the
the digits are the same. Hence, the correct maximum value of r is 1 1.
choice is (B). . Hence, 11 - 5 - 6 is the greatest pos_
i. (C) In store l 10 scarfs cost 10 X $12 : sible value of x - y
S 120. Since tlre same scarf costs
$8 in store 12. (D) If r and q are defrned by r: 4pu
4 + - 15 scarfs can be bought in store ,B ,r
- t ith $ 120. and q - rhen when p 3 nd u
v, 2. 5:
., (B) If r * 0 : r, then * represents either r-4pu:4x3x5-60and
aclclition or subtraction or both. It is also
qiven that r* r:0. Since r
- r:0 and
l,] r * 0 when ris not 0, x can fepresent 60
only subtraction.
- (D) Since Kurt has saved 3+2
$160 to buy a 60
steleo system that costs $400, he needs to
5
_ carn an additional $400 - $160 : §24o. _12
'l04 ArithmetiC Skills and Concepts

GRID-|N zI - )X
1. (158) In general, ifland Bare positive inte- 2
gefs, then the number oí integers from Ato 21 3 18
,B is (B ,4) + 1. Il the number of houses 1
2 -:9
2
in a certain communiry are numbered con- 21 3(2) : 15
secutiyely ftom 2,019 to 2,176, there are 2 2
(2,176 2,019) + I : 157 + I : 2l - 3(3) l2
l58 house. in rhe communiry. 2 2
2. (5) Since 1 kilobyte is equivalent to 1,024 X 21 - 3(4) _2
\"\ 8 or 8,]92 birs.40,q60 bits are equivalent 2 2
B ,o
49''t'0 o, 5 kilobr Les.
21 - 3(5) 6
8.192
- 3(6) :1
2 2
3. (45,8) The balance that needs to be paid 21
off is $495 - $129 or $366. Since eight 2 2
equal monthly payments will be made, each 21 3(7) _0
.. $366
!
monrhl; paymenr is or $45,75. Since Stop!
*
45.75 wl|I not fit the grid, glid in 45.8. Any value of r greater than 7 will pro-
duce a negative value for 7. Since x: 1,
4. (4) The fraction will have its largest value
3, and 5 produce positive integer values
when " í has irs grearest value and z has for 1 the sum of all possible values of x is
_
its least value. The largest value of É ? 1" 1+3+5:9.
21 ,9 or 12. The inequality 2 < m< 6
means that m is greater than 2 but less than Lesson 3-2
6. Since ru is an integer, the least value of z is
MULT|PLE-CHolCE
J. Hcncc. rhe largesr po,sible value oF !-J
.12 , - 1. (B) To divide powers with the same base,
ls-of4- keep the base and subtract the exponents.
3
If 5: é, then
5. (27) For some fixed va.lue of x, 9(x + 2)
- 7. lí the value of r is increased by 3, then
the value of7 is increased 6y 9 X 3 : 27. aa
Since after xis increased by 3,9(x + 2) : u, :a
the value o{ w - 1 ts 27.
6. (9) To find the sum of all possible values 2. (B) If 1 < x2 1 J7 <
50,.r,.,, ",Á <
of x given 3x -l 21 : 21, where x and 7 n6o,*!ryh can be written 1 < < JÍ.
". "
are positive inregers. plug succe.sive integer Since J56 is between 7 and 8, there are six
values for í stafting with i into the given integer values ofr that are greater than 1 but
equation and note which values of x pro- less than ,{Á, Z, l, 4, 5,6, and,7,
duce positive integer values for .,1. 3. (D) The given expression, (23})2, is equiva-
lent to 23*'2 or 261. Using the laws ofexpo-
nents for positive integers, rewrite each of
the answer choices as a power of 2:

' (A) (29o __


26k ,/
. (B) 64h : (26)k : 26I |/
Answers to Chapter 3 Tune-Up Fxerclses
105
. (C) : :
: !:(2'9 (22)h. (21r) (2,r) . (24h)
l. (B) Ir \r^is a positive inreger.
22L+4* : 26r / l
then ir must
be a pellecl square inreger. The perGcr
' (D) (39: = (2jb)3 : 23k.3 = 29I X square integers in the interval
100 i z <
199 are as follows:
' (E) 2', (2lL) : - \"
= 2"l (: l44(: -l
23(
"/
121 1 1z1, 12z',, 69(: 13z',,
196(_ 14z:1
4.(E)Since |=*,,.r, =t
x = x,,,,. Hence, there.are lour perfect square integers
í]\ " ln the given interval.
: tc', .o m = 8, If
(l)' : r,0, then 12. (D) If Q). 4p, you can find by express-
,= 7
4n:20,ror=4=5.Thus, ing each side olr the equarion ,,
,'po*., oF
4 the same base and then serring .i,.
.*po-
mn:8x5=40. nents of the two bases equal:
5. (C) If 2 : f, then (23)' : 4r
(2)' : (pu)' 26 : 22p
8-- p" 6:2p
:?o 3: ?
Hence.8p:fxp=rl,,. Since p = 3, then
6. (D) Ir a 3P:33:3X3x3:27
-- 4. Lhen
b- = (F).
= (4). -- t6 ,l3. (E] Rewrire 6'as (J . 2)'.
-. Then ur. ,h. lr*,
(B\ If w is a posirive
number and ul : 2, of exponents:
then ,=Ji,so 3"*b .6. : 3"*b , 0.2)"
uP: u?..:2Ji = 3,+b .3d .2r
8. (C) Break down 2. |: 2' | )' . ), .),) :
=
9. íC). Tesr each answer .hoi.. = Q"*b .3').2,
in ,urnli : 32d+b .2a
replacing r with x9 and zz with
*r. Onty 14. (Á) Determine whether each
choice (C) is true: Roman numeral
statement is true or fálse when r is
a positive
'(A)ru-|=xn.x.-l= -q-q - t _ integer.
x|a -1 í x''
, I. (;)" (t)" : : l
'(B)r+ a-7: x9 l x5* 1a rn _since ".,d
(i#)'u"
-(I)l00 = l. sratement Iis rrue.
,
^, P {xq)2 ,I8
a X' x. ' II. Since (x)' : xr, and x,'is not
equal to
x2' for all positive integer u"lu., of r, .t"."-
' (D) ,' - u: (ys)l _ jí5: í.]8 _ x5 + x|j ment II is false.
r-q .
'(E) i t2w:\+2x,*x|J tll. Since #
x'n'
= }l00 -
,jr
= }.l = xaid l. =
, ü. (E) Give_n
7= ul
and 7 is not 0. Since the 1, statement III is false.
values o1 xand uare each
doubled. replace Since only Roman numeral statement
u with 2w and x with Zr ln th. I is
o.iginai true, the correct choice is (A).
equation:
15. (A) If 7 : 25
- l, the smallest possible
1,.* : (2w)(2x)' value oF7 is.obrained by .rb,.r.,;ng
,i.
: (2u)(4x') largesr possible value of ,/ from
25. iince
:S(ux') ]Is i * = 5. rhe largest possible value o["#
:81 )' = 25. \(/hen _c : 25. rhen = 25 _ ]
25=0.
Hence, rhe original value ofy is multiplied
bv8
106 Arithmetic Skills and Concepts

16. (B) If x - ,,E and 7' : 12, then Lesson 3-3


7: Ji, so MULT|PLE-CHoICE

a
4_ 4 4 1. (E) \X/hen figuring out how many factors a
XJ V6 v12 number has, be sure to include the number
irselfand l. Try each choice in turn:
4 . (A) There are 5 factors of 12: 1,3,4,6,
vZ and 12.
1,/JD
. (B) There are 6 factors of 18: 1, 2, 3, 6,
4
9, and 18.
eE . (C) There are 3 factors of25: l,,5,and,25.
2J' . (D) There are 8 factors of 70: t, 2, 5,7,
I0, 14,35, aná70.
3\l2 -l 2 . (E) There are 9 factors of 1O0: l, 2, 4, 5,
z.l, I0. 20. 25, 50, and l00.
3.z Hence, 100 has the most factors. The cor-
rect choice is (E).
:15 2. (D) In any division example, the divi-
3 sof times the quotient plus the remainder
should equal the dividend. If the quotient
GRlD-|N of tVdivided by 5 is 13 and the remainder
N4:
1. (2) Given 21 X 42 -
16', find the value of ]s 4, then
-55 13 ]- -. so
x by expressing each side of the equation as
,^/- (5X13) + 4: 65 + 4: 69
a power of the same base.
3. (D) Solution 1: If p ís divisible by 3 and q
2o x(21)' : (21),
is divisible by 4, then ?q must be divisible
]4 V, ].1 _ -.,rl
by any combination of prime factors of 3
_
and4. Since 3 - 3 X 1 and4 : 2 X 2, Pq
]4+1 -r{.

28 : 24' is divisible by each of the following: 3, 4,


8: 4x, so x: 2 3X2oí 6, and 3 X 2X2or 12. Sinceno
product oí prime factors of 3 and 4 equals
2. (7O7l Since1= ll,a :1lá.Thus, 9,pq cannot be divisible by 9.
b
Solution 2 Pick numbels for p and 4. Then
a7 : aXa6: 7777 test each choice until you find a number
d x (llb) : 7777 that does not divide pq evenly. For example,
l|ab: 7777 if p : 6 q : 8, then ?q - 48. Testing
1lab 7777 ^nO
each answer choice, you find that 48 is
11 11 divisible by 3, 4, and 6, but not by 9,
ab : 707 4. (A) Find the value of l S:
3. (16) Since2J : 2 X 2X 2 - 8 and (/- 1)] . If .l represents the sum of the factors of
: 8, then7 - : 1 2, so 7 - 3. Hence, 18. then
(l+ 1)2 : (3 + t;' 42 : : 4 x 4 : 16 S:1+2+3+6+9+18:39
4. (1i2) Since . If Prepresents the sum of the prime num-
D+ D - D \h } bers less than 18, then

?,?,? P,?,? ?' P:2+3+5+7+1].+13+17


:58
.b2 l ]l . Hence, P- S: 58 - 39 : 19.
then
T: l),so l: ir: i. Hence.
- 1since 1X 1 1_Grid in as l/2.
P , 1 , t.
Answers ta Chapter 3 Tune-Up Exerclses 1o7

5. (B) Solution 1: A number is divisible by 3 9. (A) Since z is an odd integer, let z : 3.


if the sum of its digits is divisible by 3. In Evaluate each answer choice in turn until
choice (B), the sum of the digits of 4308 you find an odd number. For choice (A),
is 4 + 3 + 0 + 8 : 15, which is divisible (2n - 1)' : (2(3) , r)2 : (6-I)z :25
by 3. In choice (E), the sum of the digits of :2(3) 1 : 5
23,45\ is2 + 3 + 4 + 5 + 1 - 15,so (E) There is no need to go further. The correct
is also divisible by 3. A number is divisible choice is (A),
by 2 only if its last digit is even. Since the 10. (D) Consecutive multiples of 4, such as 4,
last digit of4308 is even but the last digit 8, and 12, always difíer by 4.If h 1 is a
of 23,45l is odd, the correct choice must multiple of 4, then the next larger multiple
be (B). of4 is obtained by adding 4 to k 1, which
Solution 2 Using a calculator, test each givesÉ- l 4ork-l 3,
choice in turn to find a number that gives a 1 1 . (D) You are told that, after z marbles
0 remainder when divided by 2 and by 3. afe put into z jars, each jar contains the
6. (D) Solution 1: Since 3 and 10 do not have same number of marbles, with two marbles
any common factors other than 1, any remaining. If x represents the number of
number that is divisible by both 3 and 10 marbles put into each jar, m divided by n
must be divisible by their product, 30. Any equals r with a remainder of 2. This state-
number that is divisible by 30 must also be ment can be written as
divisible by any factor of30. Since 15 is the
only answer choice that is a factor of30, any
?-,+2
number that is divisible by both 3 and 10
nn
Since
must also be divisible by 15.
Solution 2: Pick an easy number that is X:
m 2 m-2
divisible by both 3 and 10, say 30. Then nnn
divide 30 by each of the answer choices in m-2 marble. were pul into each jar.
turn. Stop when you find a number (1J)
that divides evenly into 30. The correct 12. --
(C) You are given that, when p is divided by
choice is (D). 4, the remainder is 3, and when p is divided
(C) Since í fepresents any even number and by 3, the remainder is 0. Identify the answer
_/ represents any odd number, let x
: 2 and choices that are divisible by 3. Then sub-
_l:3. Evaluate the expression in each of sritute each oFrhese choices lror7 until you
the answer choices until you find one that find the choice that produces a remainder of
produces an even number. 3 when divided by 4.
. (A) Choices (C), (D), and (E) are each divisible
"y+2:3+2:5 by 3. Fbr choice (C), Iet p : 15. \When 15
'(B)" 1:2-1_1
. (C) ("+ l)0 - 1) - (2 + 1)(3 - 1) : is divided by 4, the remainder is 3.
(3)(2) : 6. There is no need to go fur- There is no need to go further. The correct
ther. The correct choice is (C). choice is (C).
!, (D) Break down 105 into its prime factors: 13. (D) Determine whether each Roman numeral
105 1X5X21 1X3x5x7 statemel]t is true or íalse.
. I. Subtracting any multiple of2 from an
??? odd integer always produces anothel odd
Thus, when p equals any ofthe eight positive inreger. Forcxample, - - 4 = 3. Since
integers 1, 3, 5,7, t5 (3 X 5), 21 (3 X n, ,' + 5 is an odd integer and (n2 + J) -
37
l05
í5 / -),o. l0S. p is an inreger.
6 - n2 - I, n2 , 1is also an odd inte-
ger. Hence, statement I is false.
108 Arithmetic skills and Concepts
. II. The sum of an even integer and an GRlD-lN
odd integer is an odd integer. Since z2 -F
1. (2) \íhen a is divided by 7, the remainder is
5 is an odd integer, z2 must be even, so z
5, so let a: 12. ]Vhen /isdividedby7, the
must also be even. Hence, statement II is
remainder is 4, so let } : Il.Then a 1 b:
true.
. 23. ]*i}ren 23 is divided by 7, the remainder
1II. The product ofan odd integer and an
is 2.
even integer is an even integer. Since you
2. (2,0Ol) All multiples of 3 from -3000
have determined that z is even, 5 z is also
to 3000 are divisible by 3. Since
even. Hence, statement III is true.
Since only Roman numeral statements 1I
3 - lY 3,6- )x J,9 : J x J,..,,
and III are true, the correct choice is (D).
3000:1000X3
there are 1000 multiples oF 3 from 3 to
14. (D) You are told that, if the number of people
3000, inclusive. Similarly, there are 1000
who contribute equally to a gift decreases
multiples of 3 from -3000 to -3. Since 0
from four to three, each pelson must pay an
is also divisible by 3, there are 1000 -]- 1000
additional $10. You can eliminate choices
-t, 1 or 2001 integers Írom -3000 to 3000,
(C and (C), which are not divisible by both
inclusive, that are divisible by 3.
3 and 4. Check each ofthe remaining choices
3. (2) \X4ren a positive integer h is divided by
6, the remainder is 1, so let b : 7. Th,en
until you find the right one.
. (B) 60 divided by 3 is 20, and 60 divided
5b - 35, \,X4ren 35 is divided by 3, the
by 4 is 15. The difference between 20 and
remainder is 2.
15 is
. (D) i20 4. (5Z) lt-r:' problem is equivalent to asking,
'. divided by 3 is 40, and 120 "How many integers from I to 100 are not
divided by 4 is 30. The difference berween
divisible by either 3 or 5?" Make an orga-
40 and 30 is 10.
nized list while being careful not to count
There is no need to test the last choice. The
any number divisible by both 3 and 5, rwice.
correct choice is (D). . There are 20 integers from 1 to 100 that
lJ. (B) Since z is any even integer, pick a sim-
ple number for n.If n: 4, then (n+ l)2 :
are divisible by 5:

(4+l)' : 25. ]ff4ren 25 is divided by 4, the 5, 10, 15, 20,25,..., 100.


. There are 27 integers from 1 to 100 that
remainder is 1.
are divisible by 3 but not by 5:
16. (C) You need to find the answer choice
3,6,9, 12,8,...,99.
. Hence, 20 + 27 : 47 integers from 1 to
that produces remainders of 2 and 4 when
divided by 3 and 5, respectively.
. (A) The remainder when 41 is divided by 100 are divisible by either 3 or 5. Of the
100 circuit boards, 100 - 47 : 53 were
3 is 2, and the remainderwhen 41 is divided
not checkedby either ofthe two inspectors,
by 5 is l.
. (B) The remainder when 43 is divided by
3 is 1.
Lesson 3-4
. (C) The remainder when 44 is divided MULT|PLE-CHolCE
by 3 is 2, and the remainder when 44 is
divided by 5 is 4. 1. (E) Multiplying both sides of the give n

There is no need to continue. The correct equation, 2b - -3, by -2:


choice is (C).
2l2b: -2|-3)
Since -4b : 6, then I-4b : l+ 6 : 7.
2. (C) Using the given facts that a is a negatir,<
integer and á is a positive integer, determine
whether each Roman numeral statement ii
true or false.
Answers to Chapter 3 Tune,IJp Exerolses .l09
. l: o - b mq 6e negative. 0. or positive. 3: 5. Since b- a > 0. statement II
depending on the particular values of a is tnte.
and ó. For example, if a : -2 and, b : . II1. Since the square ofa negative number
2, a ,t b : 0. If, however, a :
-3, and is posirive. ,(q\ -- q' > 0, 50 §tlle-
b:4, a+ b> 0. Statement I is false. \ bl h
ment II] is true.
. 11.b - a : L_ n : L l.Sincea Only Roman numeral statements II and III
aa[la
are true,
positir e in teger dir ided by a ncga rive integer
8. (D) Since ( 2)]: (-2)X(-2)X(-2) :
is negative, á i. ,-,.g"ti r., making l -8 and ( 3)': (-3)X(_3) : 9, then
! (-2)]+( 3),:_8+9:1
.l^,
negallve. Hence "/ < 0. Sratemenr II
ls trlre_ Questions 9 and l0.
. iII. \X/hen á is an even integer, a' ) 0. From the diagram, a4 -l, -l < b < 0, 0 { r
Statement III, al -- 0, is not always true.
{ 1, and /> 1.
Hen,,e. only 5tlícment ll murr be rrue. 9. (B) Determine whether eaclr Roman numera]

.). (D) The product of five negative numbers is statement is true or false.
negative. Four negative numbers yield . I. Pick a number lror r. tf
1._ l
, -1
I ,.
then
í-)"í-] ()\{.rx(|) _() ,, 1 ^. l
c' <.
(+]
!----,J
),2:.1.slnce-<- z,
Hence, statement I is true.
Thus, four numbers in the product, at . II. Since a 1 -1, then a2 ) 1. For
most, could be negative. example,ila : 3, then a) - 9. Since 0
(A) if ,á equals 0, 2, or.
- l, k( l? - 2)(k + 1) ( c ( 1, a2 } c, Hence, statement ]I is
evaluates to 0. Hence, you can eliminate tfue.
answer choices (B), (C), and (D). Plug each . lII. Pick a number lor b. ll b: rhen
of the two remaining answer choices into 1 _2 ] c:,, _ L -- _.,'].
/<(k-2)(k+ l) to see which results in a nega- b -1 -l.Jlnce - j, -z,
tive value. Fot answer choice (A), h :
-
1

-2: rhen á Hence. statement lll i.


blk )l\k+]) - _2(_) _2)(_2 r-J; ,. fal.e.

: -2(-4)(,l) Only Roman numeral statements I and I]


--8 are true.
The correct choice is (A). 10. (C) Determine whether each Roman numeral
). (C) p'(2-5) + (-?), : -3?. -| ?. _
statement is true or false.
. I. On the basis of the diagram, pick num-
-2?'
(,. (C) If n + 5 is an odd integer, then z could bers for a and d. If a : -2 and d: 2,
be -2 since -2 + 5 _ 3. then ad - -4 { ú, so statement I is false.
(D) Determine whether each Roman numera.l . IL Since a/is the product oftwo negative
statement is true or false when a ( 0 and numbers, aá ) 0. Since a/is the product
b> 0- of numbers with opposite signs, ad 1 O.
.l. a r b may 6e negarire. 0. or positive. Since ab } ad, statement II is true.
depending on the pafticular values of . III. Since a <1 b, the reciprocaIs of a and b
a and b. For example, if a: -2 and have the opposite size relationship. Since
b - 2, then a + 1, : 11. If a: l1
-3 and, - > j. sraremenr lll is rrue.
b : 2, then a 1 b .-0. Hcnce, state,
ment I is false. Hence, only Roman numeral statements 1I
. II. Pick numbers for a and b. If a: and III are true,
-3
andá= 2.rhenb - a- ) .(-3)= 2 r
1'lo Arithmetic Skills and Concepts

1 1 . (B) If (rl- 3)' -


16, then the number inside GRlD-lN
the parentheses must be either 4 or -4. If
3:4,thenlt- 1. (10) Since 16, p' : 4 or -4. ? :
l 7 and12:49.If 1 Also, 42 :
q 6 :
ot -6. Hence,
3_4 : -1,so ),2:
36, so
3: _4, then 7: the largest possible value of q - p ís
12.
The smallest possible value of_1] is 1.
(E)
1.
Itisgiventhat r- 1+ 2+ 3 + ... + 10
6-(-4)_6+4:10.
2. (l6) Follow
(
andx: 1) + (-2) + 3) +.., + ( .
these steps:
Find the largest possible value of.1. Since l
-
(-10) -(1 + 2+ 3... + 10) : -r is always nonnegative, the largest possible
X: -14
Since
value of 7: 1 - t' occurs when ,t' has
. X+ Y: -Y+ ]': 0, which makes
its smallest value. Since -4 <
Roman numeral choice i false. = x 2, the
. Y- X: Y-(-Y)- Y-f Y-2Yso
smallest value of .t' is 0. The krgest posslb|e
valueof7is7 : I - x2 - 1 _ 0 : 1.
Roman numeral choice II must be true. . Find the smallest possible value of y
' (X)' : (- )Z)' or, equivalently, X2 : Y2,
The smallest possible value of )/ : 1 -
so Roman numeral choice I]I must be
true.
.l occurs when .t' has its largest value.
. The largest value of lis(-4)2 = 16.The
Since only Roman numeral choices II and
smallest possi5le value of 7 is 1- ! - *
IIl must be trueJ the correct choice is (E). :1 1,6_ -I5.
13. (A) Rewrite 0. Then
a2b3c as (a2b2) bc ) . subtract. The number obtained when the
determine whether each Roman numeral
smallest posslble value of 7 is subtracted
statement is true or false.
. I. Since (a2 b') bc> 0, a2 } 0, and b2 ) 0, from the largex posslbIe value of7 is 1 -
it must be the case that ác ) 0. Hence,
( 15) l * 15: ]6.

statement I is true.
. II. Using (a'b2) bc ) 0, you cannot tell Lesson 3-5
whether ac is positive or negative. Hence, MULT|PLE,CHolCE
statement II is false.
. IlI. In the product (a2b2)bc ) 0, there is 1 . (C) To find the number of minutes that
elapse from 4:56 p.l,,l.
to 5:32 t,.l,t., subtract
no restriction on the signs of a and b, so
their product can be positive or negative. 4 hours and 56 minutes from 5 hours and
Hence, statement I]I is false. 32 minutes:
Only Roman numeral statement I must be 5 hours 32 min 4 hours 92 min
true. 4 hours 56 min = 4 hours 56 min
l4. (A) Since azb3c5 : (a2bzca)bc ( 0 and 36 min
a2 b2 ca is always positive, it must be the case
Since
that bc is negative.
15. (B) Determine whether each Roman numeral 36minutes _36 _l2X3 3
statement is true or false when a * 0.
. I. (- a)' _ (_a) x (._ a) : a. and a2 - 1hour 60 12X5 5
the number of minutes that elapse from
2a2 : -a2. Since ( a)2 * -a2, state-
ment I is false.
4:56 p.tsl, to 5:32 p.xtt. is i
5
of an hour.

' II. Since -(b a): -b (-a) b+ 2. (B) lleach of the fractions f '!') i, i" to*-
a : A - á, statement I] is truc. est terms, then / cannot be divisible by 3, 4.
' IIL If a ) 0, then a ) a. However, if and 5. Of the answer choices, only 49 is not
a ( 0, then d < - a, so statement III is divisible by 3, 4, and 5.
false. 3. (D) To find the number that has the great-
Hence, only Roman numeral statement ]I is est value, compare each digit, reading from
alwavs true. left to right. The numbers in choices (A
Answers to Chapter 3 Tune-LJp Exercises 1.11

and (C) can be eliminated since they are


less rhan any ofthe orher choices. Since rhe ! i: o, . 0.5 .r 0.000
9. (C) 60 + zl
thousandths digit of 0.2938 is greater than 8 500 :
the thousandths digit ofeach ofihe remain- 62.506.
ing answer choices, 0.2938 is the largest 10. (B) SinceN -
" + iX fr,youcancancel
}:
number_
4. (C) Four hours elapse from 11;00 a.lt. to $ on each side to get N = ?e,so irr.

3:00 p.l,t. on §7ednesday of the same day, 11. (E) If n - 2.5 X 1025, then
and 52(: 24 + 24 f 4) hours elapse From
l1:00 a.v. on 'Wednesday to 3;00 p.M. on J;:rlr.5"10"
Friday of the same week. To compare these Jr5 10"
elapsed times, form the fraction "x.,l|É
^[zs
4 Á1
1,
5X\fiO".10'
5 X 10L
52 / xtl 13
5. (D) In 0.030, the digit 3 is located two 12. (A) Since division by 0 is not allowed,
places to the right of the decimal point, so a variable in the denominator of a fraction
it is in the hundredths place.
cannot be equal to a number that makes the
6. : j, then < Similarly,
(A) Since 2,1 denominator evaluare to 0. IF / : x__2.
$ ].
:
;; j ,o > 1, Hence, you cán then xcannor be equal ro l,;"* ]#:
,8i
eliminate answer choices (B), (C), (D), and
:0.

R
(E). Choice (A) is the corfect answer. 1,3. (E) Eight pencils cost $0.42, and $Z.tO +
0.42 - 5. Hence, 8 X 5 : 4O pencils can
7. (C) Since (+)' : , the new value of Zis be purchased.
* 14. (C) For a roral of48 ounces oí:orange juice.
j of rhe original value of /. six 8-ounce containers must be purchased
8. (C) Since each inch on ruler ,4 is marked in since 6 X 8 ounces : 48 ounces. If each
S-ounce container costs $0.69, six of these
equal ! -inch units, a side that measures 12
containers will cost 6 X $0.69 : $4.14.
ofthese -inch units is For a total of 48 ounces of orange juice.
{
four l2-ounce containers must be pur-
12 x ]:1: i]i.,.h.. long chased since 4 X 12 ounces : 48 ounces-
822 Ifeach 12-ounce container costs $0.95, four
If the same side is measured with ruler fl of these containers will cost 4 X $0.95 :
which is marked in equal $3.80. The amount of money that will be
}-inch units, the saued by purchasing rhe l2-ounce contain-
ers is $4.14 - $3.80 : $0.34.
side will measure 18 of t}rese l-inch units since
1,2

t!.-12+6
2 ],2 12
and 12 -| 6: 18.
1'l2 A,|ihmetiC Skills and Concepts

1i. (E)
B
There are 6ve digits that repeat in 2. (24) Since 1.5 inches replesents 45 miles,
order. The remainder obtained by dividing 1 inch represents S or 30 miles. Hence,
the number ol positions to the right of the 0.8 inch represents 0.8 X 30 or 24 miles.

R
decimal point by 5 produces the following 3, (7) If four lemons cost $0.68, one lemon
cyclical pattern. $0.68 $ 1.19
costs ------- or bU.1 /. 5lnce .. - - /.
4 b|).l /
Nth position Remainder Repeating there are typically seven lemons in 1 pound
to right of when N digit
decimal is divided of lemons.
in Nth
point by5 position 4. (l5l8 or 1.88) Since $2.50 was charged for

1 1 3 the first j mile, $10.75 $2.50 or $8.25


2 2 1 represents the charge for the total number of
1
3 7 additional miles. ar the rJte oF $0.-5 per
i
4 4 5 l
miie. Dividing $8.25 b) $Q.-5 giver rhe
5 0 2 i
6 1 3 nu-b.r" of 1miles traueled. Since $8.25
lJ
:
7 2 ,
: l I. rhc l ]l'*'.
l- mile:.
1
$0.-5 rari trip was
8 3 7 ,
l 11
9 4 5
\Wrire
j l i' as eirher a fiacrion or a decimal.
10 0 2 and then grid in the result:

. 1 11 4 II
. since7-e-,,.', 15
sridl5/8:
Hence, the repeating digit (3, 1, 7,5, or 2) or
that is in the l/th decimal position is the . l
. since __r 11
_-: : 0.5 ].3-5 1.8-5.
repeating digit from the above table that 7
corresponds to the lemainder when l,/ is grid 1.88,
divided by 5. For example, a remainder of
1 corresponds to the fepeating digit 3, a
5. (5.6) Since
ffi +
},
ZSS cubic inches are

remainder of 2 corresponds to the repeating eottivalent ,o I oln cLrbic loor, tt 1 cubic


D
digit 1, and so forth.
Because 968 - 5 : I93 remainder 3, the
Foor o[ rhe meral weighs 8 poundr, then
(l ' ]
of a cubic loor * ill weigh
repeating digit 7, obtained from the above
4
table, is in the 968th place to the right of
the decimal point.
]./or1oou,,d.
h.)
3
GRID-|N Since each pound of the metal costs $4.20,
l. t8) lf'f ,' - |.rhen *f : ]j since the cosr ofl
l
pounds equals
12r : i. !, : il.4o:
5
17 17 $+.ro 4x $5.60
Since : 4^ must be multiple of
3
ff ,U,
a Omit the dollar sign and grid 5.6.
12. Hence, 4^:48, so A : 8.
Answers to Chapter 3 Tune Up Exerolses .'
í3
Lesson 3-6
7. (C) It is given that 0 <r { 1.
MULTlPLE-CHolcE .lfx-:,';-- = r-jro,.o
l. (D) To subtract 7f minutes from j -inut
9 r, Roman numeral choice I is false.
.}^"a!.$, .
].*' - *'
fi.,t.hang.f (j)'- (j)
IfJr = -_ = ] -
1_1_
n4 _.,9 _ ^tc, _9 / r 2 ^ . Koman numeral choice l[
is false.

Since 1 minute : 60 seconds, ' uR* , Ú,


lf, mi.r,,,t..
Since i Ron.''rn
= 60 + 35 : 95 seconds.
numeral choice III is equivalent to
2. tA) sincej -
i= :, := f. , which, after squaring both
sides'
imulriplying :'
the inequaliry, ii .qriual.nt
tox).of54.Vhen 0 {r{
a number ay
} ^nathen dividing the result
1, as the
by exponent of x increases, the power
f is equivalent to multiplying of
the origina1
l decreases. Since it is true that x ) xa,
.,,rmb.r by Roman numeral choice III
]. -u.t b. t.,r",
which makes (C) the correct choice.
3 Jh. profit for each keychain
sold is
!?)_
$1?5 - :
I9l: $0.50. To make a profit
8. (C) if É:
},rhen
oF$250.
!5O = ZSO = )50 ./ 2 b
$0.50 l
=
k-- e -c'=4="^b -bc
1
abo;
500 keychains must be sold. b
I_
4. tDl l soi-i' a
41a]ortz-_í,',&
,t

_. q (A) Ifan item is on


3 \ /' ]-') sale ar , oFf rhe lisr orice
of 1dollars, its sale price k d -
2
:d - ' :d.
). ((]) l ' <,,z _ lF the sale price oi ., 1 is reduced
-'"
-nÉf
21 oF that price. rhen rhe final ,ale
by ;
p.ice ii
/ +,t - +(+d) Simpli§.:
,.7/l>,5\/--l0
"í 2, - r/;,\ -,),
3" ;\i'): i, - i;l
6, (C) You. are rold rhar f < z <
Since j. :la-t,1
66
each ot lhe answer choices
has a
tor of 24.,change 1 and -| into
denomina- :tr1
equivalent 6
lractions that have 24 as their
tors. Since
denomina- :d
2
6_6 10. tC) For rhe lracrion ,o ,rdrr, ,o )5
4 624 | .b
and must be
.a.
mulriple of J. fhere .r. only
1r rhree multiples ofJ
] o berween 5 and i J: 6. 9,
3 3 6 18
and l2. For each ofth..e uulu.. of
á th... i,
one integer^value for a such that
rhen f <
"
< *. A possible value of z is reduces to f:
the fraction

i, 3

::t:9:2:!
b()912]
'l14 Arithmetic Skills and Concepts

ll. |E) Solution l:lf ! i, a fracrion less rhan l, GRlD-lN


b
then its reciprocal is greater than 1. For 1. (3/2)'§íhen x: },
example, ] ( 1 and j > 1. Sincethe Ei |(1 -
)t)
reclProcal or
a
b may be Wrl(ten as '\i 2

l.a>t
a'a
ii 2
:Étl]- \]ZF 1
2
ru o time, thi. value. or 1
b Grid in as 3/2.
must also be greater than 1. 2. (3l4) To find what fraction of a is á divide
solution 2: pick numbers for a and b that bby a, as in
make f, { 1. For example, let a:3 and 5 l0 5._9 ]
á : 4. Plug these numbers into each answer 6 9 6 10 4
choice ur-rtil you find the one (E) that pro-
Grid in as 3/4.
duces a number greater than 1.

3 (625),,r,[í) :4,;:
12. (B) After Juan gives ] of his p pencils to
Roger, Juan is leit with { pencils. 'When
he gives I ol the pencils that he has left
**i: i.
to Maria. Juan has :. _ _Í:) pencils lefr.
/\
t \ Hence. ]00|{ |: roo1f,1 )5(25|
Simplií}: /
05. (* /

p ríp) p p 4. (9iExpress the sum insi<le each set ofparen-


---|-|-
3 4\3) 3 ],2 theses as an improper fraction. Then mul-
p tiply by matching pairs of numerators and
=4p
12 12 denominators that cancel out to 1, except
3p for the denominator ofthe first fraction and
l2 rhe nttmerator of the last fraction:

- 4.P
13. (C) Determine whether each Roman
1

, |,
- j][ rI i] ,:)( il [,
numeral inequaliry makes the statement _ [,]1,]t:l |." lí ,
IrJlr.il'-J
]
true or false. Ilr,.J|r./
. I and II. Ifa number that is greatef than 1 :t;)t+]t;)
is raised to a power, then, as the exponent
(+)(+]
gets larger, the value of that expression : 18
also becomes larger. Thus a3 1 a2 1 a 2
is false when a) \, and true when 0 {
a ( 1 . Hence, inequality I is ülse, and
inequaliry II is true.
. III. If 1<a<O,thena2)0anda1
0, so a2 ) a. Therefore, a3 1 d 1 ais
not true. Thus, inequality III is false.
Only Roman numeral inequality Il is true.
Answers ta Chapter 3 TUne-Up Exercises 'l 15

5. (ll32) Replace /with its equivalent:


6. (A) After Boris lends ] of D dollars, he
lt l1 11 h". jD doll"r. left. After he spends j of
, 2 [' +
,+4+ *-*] the money that he has left, the number
11
I

++ l of dollars he now has is iD - j(+D).


l +
1

2+ 4 ,8 1

16 32 Simpli$,:
d j l(3 \ l
-l l) -| - l) l
3|r =
.'-';o
^_ ) L
l /
32 2
,1 =-D
4
'-o-12
Grid in answer as l/32. - lO o,2
22
7. (A) The value obtained by increasing a by I
Lesson 3-7 of its value is
N/ULT|PLE-CHoICE l
a+ a: a+0.2a:1.2a
1. (C) If f of a number is 6, then j of the 5

same number is { ol6, which is 2. Since I The value obtained by decreasing á by ] of


of the number is 2, { of the same number its value is
is7 X 2 or 14.
2. (B) If Claire has read ] ofa book, f ofthe l ll-b
2
0.5b:0.5b
book remains to be read. since there are
Since it is given that these rwo values are
120 pages left to read, f oFthe total num-
eqlaI, :
ber oípages in the book equals 120. Hence, '-) 1 ,2a 0,5 b.
I ofthe total number ofpages in the book l _ \.4
is ] of 120 or 40. Since there are 40 pages s. tCt J]: : v = ]+
in r 6flfie book. rhere are 5 ' +0 or 200
24\1 -
pages in rhe 6rsr ; ofrhe book rhat Claire 9. (B) If a chocolate bar that weighs f of a
has read. §3 pound i, cut into :evett equaí parrs. rhree
\J parrs weigh - ., |" = , pound,
J_
r,tBl 1oF27-_1, L_ll 10. (C) At the basketball ga-e, { of the stu-
9\1 dents who attended were seniors, so ] were
If i ofa number is 12, then ] ofthat num- not seniors. of the students who were not
.l
bel is I of 12 or 2. Since j ofa number is senlofs, ; were lunlors.
. The number of juniors who attended the
2, the number must be 5 X 2 or 10.
,. rDl If ] oFa cl;,., are Llo1s. rhen ; are girl.,. game comprised j of ] or f oFthe total

Since rhere are )0 3irls in the class. i of rhe


number oí students.
. The number of seniors and juniors who
class represents l0 students, so in I ofthe
attended represented
class there are 1 olr 10 or 5 students. Since
there are 5 students in { ofthe class, there 3 z 9 ) ll
is a total oí 5 X 5 or 25 students in the 515 15 15 15
whole class. of the total number of students.
S. (E) The nvo-digit whole numbers in which . The remaining 80 students were aIl soph-
cither digit is ] of the other digit are 34, omores. 5ince l ]i {. ; ol the
43,68, and 86. Use a calculator to find the students in attendance were sophomores.
sum ofthese four nrrmbels: 34 + 43 + 68 Thus.
+ 86 _ 23l. 411 ... 80 )0
15 15 -}
----..80.so- 4
1'16 Arithmetic Skills and Concepts
. Since * of the total number of students Since F ofa fu]l tarrk holds J gallons. i or
is 20 and 15 X 20 - 300, a total of a full tank holds 15 X 3 or 45 gallons.
300 students attended the game, 15. (C) Since Susan received ] of the ballots
. Since the number of seniors comprised I cast, + of the ballots were cast for the other
ofthe total number ofstudents, the num- candidates. Bill received i - _t : 4 of the
ber olseniors who attended the game was remaining ballots, *hiJh ,o
"Lo,rrl..d
48 votes. Bill's votes represent 1 X j or *
3
of300:1*:oo:rso of the total number of ballots. Hence,
5 5
Hence, 180 seniors attended the basketball jl,,|]--; 48.so] -> -' \ 48 - ]2
game. 15í54
11. (C) Il of rhe 75 people in a room, ! are Since 12 votes afe } of the total number
college graduates, then ! are not col\ege cast, the total number of votes cast was
gracluates. If f ofthe students who are zol 12 X 15 : 180. Since Susan received
college graduates are seniors in high school,
] oF the 180 ballots cast, she received
then ] of the students who are not college
+x180:120votes.
graduates are za, seniors in high school. 16. (D) You can find the capaciry of the gas
Thus, } X { or { of the number of people tank by reasoning as follows:
in the room are neither college graduates . If a gas Bauge moves from exactly } full
nor high school seniors. to exaclly * full. rhen the amount ofgas
Since there are 75 people in the room and pur into the rank l,epre.ents i(- ; ;)
+ x 75 : 25,25 people are neither college of the capaciry of the tank.
graduates nor high school seniors. . If the gas costs $t.50 per gallon and
12. (B) You are given that ] oí j ofa number Arlene is charged $ 18.00 for the gas, then
is 24, Since the number of gallons put into the tank
11 eouals 18 + I.5 - 12.
/í1 S;n; + -+ l2.; -+ | \ ]2 - +.
-r<::-
íÁ) 4(full
2 ,rnkt -> 4X4 - ]6 8a]lons
4
J of the number is 24, so the number is The capaciry ofthe gas tank is 16 gallons.
2 X 24 or 48. Hence, } of the number is 17. (E) Suppose the prices of stocks A and B
f, x48or12. were each $ 100 at the beginning of the day.
13. (C) Ifa man paints j ola
house in 2 days, Then:
then he painted } of the house in ] X 2 . If the price of stock l increases by fr of
or ; ola day. Since ofthe hou.e rimain. its original value, the new price of stock
to be painted, it will take him ] ofa day to Ais
complere the !ob. [r l :
l4. (E) Ilra water rank drops From being iull i $l00 - |-: , $l00 | $l00 $la $Il0
ll0
i
ro lrull, then rhe level of the water tank
l

. If the price of stock B decreases byfrof


d.op, f
ofa full tank since
1_!:9 5 4
irs original vaIue. the neu price of stock
Bis
5315 15 15 (l \
tank drops f ofa full tank as a Sl00 |- $l00 l - $]00 $l0 : $90
The water l' l0 )
result of 12 gallons being poured out. Thus,
. The relationship beween the new price
411
-15
-J l2,so - --; 4 X
15
12 = J of stock l and the new price of stock B
can be found by íorming a fraction in
Answers to Chapter 3 Tune-IJp
Exerclses 117
which the numerator is the
new price of
l and the denominator is the new
stock . Since j of the male voters cast their
_,price
ofstock &
+ #: ballots for candidate l, this candidate
1 he new price of
stock l is ,' of rhe new received j X l00 or 50 votes
price of stock B.
rro- ,t.
males.
. \, .
jO) S;nnose ,z re_pre'ents rhe original num- ,Since 3 of the female voters cast their
\X/hen J ofrhese
u§r or peopIe tn rhe room. ballots fo_r candidate l, this candidate
.lpeople leave rhe room,
}r
people received j x 150 or 100 votes
remain. from the
Alter three people enrer rhe females.
same room,
lhere are,;z -l_ 3 people
in rhe room. Since
. A total oí 100 + 15O or250 ballots were
tne nu.mber ol people nou cast. Candidate l received
- oi rhe_,originJl number ofinpeople
rhe room is
150 votes. Since
50 ]- i00 or
in rhe
foom, lZ -r J : :n. Hence,
1,50'_3X26 _3
1

J-1]
- -n - -n )50 5\r4 i
34
rhe righr side of this equation candidare,4 received i
]:lll? fractjonr alre, changing
by
ot votes cast.
of rhe roraI number
],1o.'.1.,,ng them rá
l llfl r lowesl
common denominaror; Grid in as 3/5.

2_ 4 3
12 12.
Lesson 3-8
1
lVULTlPLE_CHolCE
12
3X12=n 1. (B)21o/oof ?of l5: ] ,2
36-n 3 ,X-x15 =2
You now know that the original 2. íC) To chan8e from a percent to a decimal,
number of
people in the room was dtvide the percenr by l00 and
3Ó. The qu.r.ion drop rhe per-
lsks For, rhe number of people cent sign:
who'lelr the
room:-X36:27_
2^, 1 =-:0,004
2
o.+
_.. D-lN i'"=]00
, 'I8l After a number is increased by , of
3. (C) If480 of500 seats were occupied, then
iL,
20 seats were not occupied. H.nce,
''alue. the new number i, + .r;,i the per-
oILhe increase"r;gi"ri
,alue. Since rhe retult cenr o1 rhe seat: thar were nol
ir-24, occupied is
ll 20l
!.,, r00 '
IO0oo = L ./ l00oo -_ 4oo
3-2Zl,roj_7\24 =rr
Since i of rhe original 4. (C). Plus .".y-,o-ur" numbers
number is 6, multi- into the
ptytng by 3; problem. Assume there are 20
members of
the, class who do nor belong
3 ro ,i. ;;;
-+3X6=i8 c]ub..Then 50o/o of 20
= 10 members of
the class belong ro rhe marh .lub.
gives l8
as rhe originaI number,
H.nc.,
rhe toral number of math club
members in
-, J/5) Pick a conuenient number.
fbr the number of male
say l00,
voters. you are
rhis clas, isffi \ l00oo = .1.1-joo orthe
,
rold that rhe number of Female entire class.
volers wJs
l : ílmes rhe number
of male vore rs. Then
there were 1 00 X l j o.
150 fem"l. uot"...
'l'l8 ArithmetiC Skills and Concepts

5. (A) 10. (C) Two students use the computer a total


Amounr oldecre:se
X of 28o/o i 52o/o or 80o/o oí the total time of
i00o/o
l00o/o Original amount 3 hours ol 180 minutes. The third student
_ |68 - 147
X 100o/o
uses the computer for 100% - 80a/o or 20o/cl
l68 of 1 80 minutes, which equals 0.2 X 180 -
2], 36 minutes. 1
X 100% 4l 1/
168 l l. (D} 20oo of 25no o] -55 -
: 0.125 X 100% Á5
: !
-25 -
or
1
12.5Vo 12 o/o
0.04

6. (C) Since * : : |, the original 3-cup i2. (C) 0.04 percent means 0.04 out oíevery 100
'
mixture containr
}cupt oF m ilk. Alre r l cup or, equivalently,4 out ofevery 10,000. Since
of milk is added io rhe 3-cup mixrure. rhe
4-cup mixture contains
4I l our ol'every 2,500 li3hr
T.;dod 2J00
]* r - : -:: 1_4.uo,ot-ilk bulbs manuFactured will, on the average, be
))15 defective. Maintaining the same ratio, on the
Since average 1 X 3 : 3 out of every 2,500 X 3
14 : 7,500 light bulbs manuFactured will be
5_ 4 .,1 14 delective.
5n4- )O: 13. (B) Since 4oo/o ,l 25o/o + 20o/o + |0o/o :
U,/U
4
70o/o of the 4-cup mixture is milk. 95%, the percent of the vehicles that are
jeeps is 100% 95o/o - 5o/o. Hence, 5 out
7. (B) If the result of increasing a by 30Oo/o
of a is b, the n a i 3a : 4a : á. Dividing ofevery 100 cars are jeeps. Ifthere are 5 X 4
borh sides of 4a b 6y b gives ! = |, - 20 jeeps, there must be a total of 100 X 4
.o; - +.Since } : 25o/o, ais 25o/o of b. : 400 vehicles.
8. (D) After a 20O/o príce increase, the new 14. (C) Suppose]ack's beginning weight was 100
pfice of a radio is $78.00. Hence, pounds. After a 207o increase, his new weight

$ 7 8.00
is 100 + 20 : l2O pounds. Since 25olo
oIlglnal Dflce of 120 : 30, after a 25o/o weight decrease.
l -f 2Oo/o
his final weight is 120 30 - 90 pounds.
_ $78.00
which is

X l00no - 90no of his begin-
1.2 ,n",
- $78.00 + 1.2
ning weight.
15. (D) Since 4oo/o is ! and ais 40o/o of á, tIre::
: $65.00 a : :b.Thus:
!. (C) After a discount of l5%, the price ola ]5J d:d-r
22
shirt is $51. Hence, /l

$5l
orlgl nal prlce Since ú exceeds aby }a and ] : 1.5
t- l59o 100% - 150%, then á exceeds aby 150'
_ s5l of a.
1 0.15
$51 + 0.85
$60.00
Answers ta Chapter 3 Tune Up Exerclses 119

GRID-|N 2. (50) After three boys are dropped from the

. I. (87.5) The cash price of$84.00 rel]ecrs the R .l"r..25 students remain. Of rhe 25 studenls.
\ \ new amount alter a discount of40o. Hence, B 44oo are boys. so 5bozo are girls. Since 569o of
25 : 0.56 X 25 : 14, 14 girls are enrolled
in the class. Hence, 14 ofthe 28 students in
Original New amount after decrease of p7o the original class were girls. Thus, the num-
amount 1 - Po/o ber of girls in the original class comprised }
or 50olo of that class.
$84
1- 4o/o
$84
=1
- 0.04
: $84
0.96
= $87.5

The credit-card purchase Drlce of the same


item is $8.75.
Algebraic Methods
-&'
'r,
'&;.&*-

l--,,
!-\ ,a-\
-L
- -L
-
-

his chapter reviews equations and inequalities,


as well as some related algebraic
methods that you need to know for tÁe
SAT.

Lessons in This Chapter


Lesson 4- 1 Solving Equations
Lesson 4-2 Equations with More Than One Variable
Lesson 4-3 Polynomials and AJgebraic Fractions
Lesson 4-4 Factoring
Lesson 4-5
Quadratic Equations
Lesson 4,6 Systems of Equations
Lesson 4-7 Algebraic Inequalities

121
Solving Equations

Overview
A linear or first-degree equation is an equation suchas2y * 1 : 13 in which
the exponent ofthe variable is l. To so]ve a linear equation, do the same things
to both sides of the equation until the resulting equation has the form

letter : number

Solving Linear Equations by Using lnverse Operations


To isolate the letter (variable) in a linear equation, you must perform the same
arithmetic operation on each side of the equaiion.

' In the equation x - 7 - -3, x can be isolated by adding 7 on eachside of the


equation;

x - 7 : -3
+7: +7
xi0:4
x:4

In the equation x i 5 : 3, x can be isolated by subtracting 5 on each side of


the equation:

x+J:3
-5: -5
xi0:-2
x- -2
In the equation 3x: 21, xcan be isolated by dividing each side ofthe
equation by 3:
3x:2l
3x 21

122
Solving Equations 123

In the equation ] x : 8, x can be isolated by multiplying each side of the


equation by the reciprocal of the fractional coefficient of r:

2
-x:
3
-8

i(1,):},},

So|ving Linear Equations by Using Two Operations


lo isolate a letter in a linear equation, it may be necessary to add or subtract and then
:o multiply or divide. For example, to solve 2y + 1 : 13, begin by subtracting 1 on
.:clr side of the equation to get 27 : t2. Dividing each side of this equation by
i sives7: (.

Sclving Equations with Parentheses


: err equation contains parenthesesj remove them by applying the distributive law.
':en
solve the resulting equation.

:'"AMPLE
So]Ve for b: 3(b + 2) + 2b = 21

: _ .]tion

3(b + 2) + 2b: 2l
' :.;move the parentheses by multiplying each
:.|nl inside the parentheses by 3: 3b+6+2b_2I
' lnlbine like terms: 5b+6:21
' '.:'rtract 6 from each side ofthe equation: 5b+6 6:21 -6
" .:lpliFy: 5b: 15

' _ ;ide each side ofthe equation by 5b _15


5:
55
b:3
'l24 Algebraic Methods

Solving Equations That Contain Fractions


If an equation contains fractions, eliminate them by multiplying each member of
both sides of the equation by the LCD of all the denominators. Then solve the
resulting equation.

EXA|VPLE
mm :2
solve form:
a- 4
Solution
Since the LCD of 3 and 4 is 12, multiply each member of the given equation by 12;
(m\ (m\
lz':J)- tz|4)- tz{zl
4m - 3m: 24
m: 24

Solving Equations by Cross-Multiplying


4I
Since
*: i,4 / 2:8 X 1. This operation is sometimes referred to as cfoss-
multiplying. If an equation sets one fraction equal to another fraction, then cross-
multiply and solve the equation that results.

EXAMPLE

í::L:l
ó5
, What iS the Value oí x?

SoIution
Cross-multiply by setting the products of opposite pairs of terms equal:

{ *--,,,-
t-\
x* 4
*-
]

5r: j(x + 4)
5x: 3x -l 12
2x: 12

,:!: e

Solving an Extended Equation


Ifan SÁT math equation involves an extended equation such as 27 - x: x1 6
10, break down the equation into two equations. Solve the equation that con
only a single variable and then use that result to help solve the other equation.
example, to solve the given extended equation for .1
. 'Write: 21 ,x:10 and x+6-10.
Solwng Equations .l25
. Solve the equation that contains only
a single variable:

lr -| 6 : 10
-6: -6
X:4
Substirure rhe result in rhe other equation:

)^,_.,_ 10
10
l0+4 T§F

z z
)lt I4 ln your regular
math class, solving
for x typically gives
7 you the final
answer to the
Using a Root of an Equation to Answer problem. Butx is
a Question not a|ways the final
To find rhe value oFan
d'l dlHcDralc
aleebrai expression that involves particular answer on the SAT.
,.J';; ;;;;; .;t
a lettet you may When reading a
question, circle or
underline the
EXAíVPLE quantity that the
question asks you
lt 2x + 5:11, to find. After you
What is the Value ot 2X
- 5?
solve the problem,
check that your
Solution answer matches
what you were
Instead ofsolving for x, solve for
2x. Then subtract 5 from both sides reqr_rired to find by
ofthatequation, Since2x+ 5 : 11, 2x :
6,so2x _ 5 : 6 - 5 _ t. |ooking back at
what you circled or
The value of 2x
- 5 is 1. underlined,

T|PS for Scoring High


1. If time permits, plug your solution ofan
equation back into the origina1
equation to make sure that it works.
2. stuck on. a multiple-choice quesdon
l|]:,:, *:' that asks for the value of a
you may.be able to 6nd rhe solurion by
:?:';:':.::r:,::"ation, tesring each
oi (rle numerlca] answer choices in the equarion
unril you 6nj th. on. thar
works_
3. Answer the question asked. For example,
ifthe question is ''If 31 2 = 4,
-
what is the value of y
- 2?'' make.*. yo..,, ;;;;';i. ,ar,
"r - ',
y
not the yalue ofl.
126 Alqebraic Methods

Lesson 4-1 Tune-Up Exercises


Multiple-Choice
1. If 5 + J : S.Z, what is the value of 6. If 3x - 6: 18, then x * 2:
> - \ln!
"
(A) 2
(A) 0 (B) 4
(B) 1.8 (C) 6
(C) 3.2 (D) 8
(D) 5 (E) 10
(E) 6.8
7. If2x 1- 1: l + 74, then x:
2. If (9 - 4)(x + 4): 30, then x: (a)
7
(^) 2 7
(B) 4 (B) 7
(C) 6 (C) 14
(D) 8 (D) 21
(E) 12 (E) It cannot be determined.

Á4,
3. If 2(I + 5):3(u- 4),thenw: jk:
(ö0
8. If
;h: 36, then

(B) 2 (A) 21
(C) 4 (B) 27
(D) 6 (C) 32
(E) 8 (D) 35
(E) 42
x
4. Il 3, 1:1-3,rhenx: 1l1*
Ií ,rx + : 14, then x --
9.
4x ,x
(A) -6 (A) 4
(B) -3 (B) 8
(C) -1 (C) 12
(D) 3 (D) 16
(E) 6 (E) 24

5. If 5a -
2b = b + I : 9,whatis the 10. If 2s + 3t = 12 and 4s : 36, then r:
valle of a?
(A) -:
(A) 2 (B) -2
(B) , (C) 2
(C) 10 (D) 3
(D) 1, (E) 9
(E) 25
Solving Equations 127
i
1. : :
1 If r
}*t,
*h^tis the value of lwhen
15. If+: 4, then 31
r= 54andx:12?
(A) l (^) 4
(B) 6 (B) 12
(C) 8 (C) 24
(D) 12 (D) 30
(E) 20 (E) ae

?-2,thenxl2:
16. Ir4U + k) - 5: 16 + (j + k),then
12. If j+ k:
(A) 8

@)1 (B) 7
(C) 5
5 (D) 4
(B) 1 (E) It cannot be determined from the
2
(C) information given.
3
(D) 4
(E) 6 17. V/hen the number / is multiplied by 5,
the result is the same as when 5 is added
,: to É.lX4rat is the value of É?
J
l-')
l3. If
2
7 -l 2,then7: l,
(A) -:
(A) -6 5
(B) (B)
-4 1

(C) -2
(D) 4 lC) +
4
(E) 6
(D) 1
2
I4. *? : +,then7: E)7
4

(A) 5 :
18. Ií - 27 and 1: j, whar is the
(B) 9
(C) 13 u"l,r. .,f f rl
(D) 17 2
(E) 21 (A) a
1
(B) a1
(C) 6
(D)
(E) 12
128 Algebraic Methods

rs. rr 1+ 1 :
3p 1, then p : 20, I[1_9:2
4x! ,thenxl7:

] (^) 4
6)1 (B) 8
(B) 2
(C) 12
(D) 16
(C) 1 (E) 20
2
o
(D) 1
2
(E) 6

Grid-ln
l. If 2x - |: ll andJy- 12, whar is rhe
,,ral,.e.,f I? 4. In the equation p: 5b
, what is a value
1 -c
of cwhen ?:9 and b: 20?
2. If 7(a + ll) - 4(a + b) : 24, what is the
value of a -| b?
5. If 600/o of ris equal to solo of45, what is
v"lu. of1?
3. Ifl-x 2x- 3x:6x - 1,whatis the
J
the value of .c
Equations with More
Than one variable .í&; &ril§
](*' :'{§§'

Overview
An equation that has more than one letter can have many difFerent
numerical solutions. For example, in the equation x J- : 8, ifx : 1, then y
7
: 7; if x: 2, then y : 6. Since the va]ue ofl depends on the value ofx,
and x may be any real number, the equation x * :
7 8 has infinitely many
solutions.
The SAT may include questions that ask you to:
. Find the value ofan expression that is a multiple ofone side ofan
equation. For example, íf x + 27: 9, then the value of 2x * 47is I8
srnce

2x-| 41 : 2(x + 21) : 2(9) : 18


. Solve for one letter in terms ol the other letter(s) of an equation. For
example, if x,f 7:8, thenxin terms oflis x:8
- !.

,Vorking with an Equation in Two Unknowns


:ome SAT questions can be answered by multiplying or dividing an equation by a
,:ritable number.

=XA|VPLE
|l 3X + 3y : 12, What iS the Value oí X + y - 62

: ] Ution
' :olve the given equation for x -l yby dividing each member by 3:

Jx 3y lZ
333
x+JJ:4
' i-,r,3|111. the required expression by replacing x i 1with 4:
x-l 7-6:4-6
.r,alueofr+1-6is-2.

129
'l3o Aloebraic Methods

EXAMPLE
t33s
:
lf
; o, what is the Value oí 4t ?
SoIution
t1
Since -s4: :. rhen Js : 4r J

EXA|VPLE

líX + 5 : í, lhen 2X + 9 =

(A) t 1

(B) t+ 1

(c) 2t
(D) 2t 1

(E) 2t+1

SoIution
Method 1: Use algebraic reasoning:
. Since the coefficient of xín2x,| 9 is 2, multiply the given equation by 2:
2(x+ 5):2t,so 2x+ l0_2t
. Comparing the left side of 2r t 10 : 2t with 2r i ! 5uggg515 that you
subtract 1 from both sides of the equation:
(2x+ 10)-I:2t 1

2x,|):2s 1

The correct choice is (D).

Method 2: Pick easy numbers for xand rthat satis$. x -I- 5 :


t. Then use these values
for xand / to compare 2x 9 with each i
of the answer choices, For example, when
j.: 1, 1 + 5 - t-
6. Usingx: 1,youknowthat2x+ 9:2(1) -]- 11. Hence.9:
the correct answer choice is the one that evaluates to 11 when r 6: -
Choice (A): t- I:6 1
:5 x
Choice (B): ti 1 :6 -l 1 x
Choice (C): 2t:2(6) : 12 x
Choice (D): 2t-í:2(6) 1 :11 ,/
Choice (E); 2t-t1-2(6)+l, .-13 X
Equations With More Than one Variable 131

Solving for One Letter in Terms of


Another
To solve an equation that has different letters, isolate lf an equation involving two or more variables appears
in the usual way the letter that is being solved for. early in a test section, try making a simple substitution.
For example, if it is given that 3(a - b)(a + b) : 6O and
a - b : 2, then you can find the value of a + b simply
EXAMPLE by replacinga - bwith2 in the equation:

lí u - z : y, What is x in terms oí y and z? j(2)(a + b) = 6),so a + b : §9 = 1g

SoIution
Tieat 7 and z as constants, and isolate ,r on one side of the equation.
. Add aon both sides of2x - z= ltogetZx:7-| z
. Divide both sides of 2x: 7 r zby2:

J+ z
2
'l32 Algebraic Methods

Lesson 4-2 Tune-Up Exercises


Multiple-Choice
1. If6 : 2x + 47, what is the value of 5. If xy }- z - 7, what is x in terms
of 7 and

xi2j
(A) 2 (A)
vl
4- z
(B) 3 }
(C) 6 y- z
(D)8
(B),z
(E) 12 v- z
(C)

2. "h (D) -1-z


|r
i -i = 3 *hr, is the value of
2a + 2b? z- 1|
(E)
y
(A) 6 -
(B) 8
6. It b(x + 4' :
60 and b! : 15, what is
(C) 12
the va|le of bx?
(D) 16
(E) 24 (ö 15
(B) 20
3. If 2s - 3t: 3t - s, what is s in terms of (C) 25
(D) 30
t?
(E) 45
,",, a2
tnt
(B) 2t 7. If 2x2 1- 37': 0, what is the value of
3x -f 21?
(C) t+2
(D) ++1
(A) 1

l (B) 0
(E) 3t (C) 1
(D) 3
4. lF a l b: 5 and { : 3, *har is the value (E) 5
2
of 2a * 2b + 2c?
í a--l' =
L,1
, *hr, of í
(§ 12
8,
b3'b
-l is rhe ua]ue

(B) 14
(C) l
16 (,\)
(D) 20 ;
(E) 22 ,) 9
(B)
]
O;
(D)
Í
(E) 2
Equations with More Than one Variable í33
9. Ils I l:
3sis 2 more than r-l- Jr, thenr- 13. If ux : a which of
the following
(A) -2 expressions is equal to xz?
(B) -_L
2 (A)
a
z,
(C) _L
2 (B) a2
z
(D) 1 (C) ll,z'
4
(E) 2 (D) u2z
(E) z2
''' 1
a
11
.1-
P+q
=r and p +-4. whar ispin
terms of r and
/ 14. If the value of z nickels plus /dimes
is r
cents, what is z in terms of d and,
t1,1 rQ - 1 e?

q (A) !- z,a
B) 1+,q
5

q (B) 5c, 2d
(C) (C\ C- d
-1_
,*r4 10
(D) 1- rq ,n
(D)É
r
(Dl-q ,-, c + lod
5
rq
' If 2b = 5c, then 4a is equal to which -- c a
of^a.: '' j- t =x.a _2c. and b-- Sd,
15.
the following expressions? =

I. c whatis the ,alue oF


) in rerms of "C
8

+ 10c
ix
I1. 4b
Á)
IIL 2b + l5c
(A) I, II, and III
(B) 3"
4"
(B) II and III only íC)+*
(C) II onty :)

(D) IIi only (D) *"


)
(E) None
,
G) Z,
2"
,, al
. ,, bl
--J-- c =
a-| b
;-Ihenr= l6. If bx - 4 : (h - 1)x, which of the
,, a - b following must be true?

a]
2 (A) x: -5
1B) ---
b (,B) X_ _4
2, (C) ,r: _3
C) 5a+ 5b (D) x: 4
D)n+b
(E) r:5
5
E) -a- b
,l34 Algebraic Methods

17, If c: b+ Iandp:4b+ 5, which of 18. Ifp and r are positive integers and
the following is an expression for p, in 2p-l r* I-211pi 1, which of the
terms oí r? following must be true?
(A) 4c l. p and r are consecutive integers.
(B) 4c 4 II. p is even.
III. ris odd.
(C)'''
4 (A) None
(D) 4c+ I (B) I only
(E) 4c+ 9 (C) II only
(D) III only
(E) I, II, and III

Grid-ln
1. If 16 X a'X 64: (l X b)'and a and, b 3. If a- 2x i3and b- 4x- 7, forwhat
are positive inte8ers, then Ú is how many value ofx is 3b : 5a?
times greater than a?
Xy 3l
4.
2. If 3a- c:5band3a+3b- c:40. 85 40
what is the value of á? In the equation above, if x and 1 are
positive integers, what is the value of
x-l 1?
Polynomials and Algebraic
Fractions

Overview
A polynomial is a single term or the
sum or difference of two or more
unlike
::'-'
F:.: example, the polynomial a +
2b i i...p.r*., .r'. sum of the
rltree unlike terms a, 2b, and
3c Since poly".rni"r, i"pr.r.l, ,."l nurnb.rr,
added, subtracted, multiptied,
,|fi;::.* diridil-;;;rg the laws of
""d
Whe never a letrer appean in the
,
rhat it cannot repre)enr a number
denominaror o[a í]r:ction, you may
assume
that make. ,rr. a."..i"r,.r. of rhe
equal to 0. lracrion

Jlassifying Polynomials
-^ polynomial can be classified according
to the number of terms it contains.
. Á polynomial with one rerm, as in
3l2, is cal]ed a monomial.
. A polynomial with two unlike terms,
as in 2x *
mlal. It, iscalled a bino-
. A polynomial with tlrree unlike terms,
trinomial.
as in x. + 3x - 5, is calle<J a

] oerations with Polynomials


,llnomials may be added,
subtracted, multiplied, and divided-
' po]yno-ials, write one polynomial
on top olthe other one so that like terms
]"]iened
"dd
in rhe same verricaI colLrmns.Then are
--i-.tii.,..-.. i..."r-ol",
zX - 7x+ /
+ x2 +5x-9
3x2 +2x-2
subtract polynomials, take the t
"o
..rbtracted. that is being
Then adcl the wo r",rj.Hi:'.:'."ij.;T:,ii:T,illiirial
(7x-37-9z)-(5x+7-4z)
a sum by adding the opposite
_:1 !:.*iod,rn,:
, ,]\,nomIal ro the hrsr polynqrniáj;
of each rerm olthe second

7x-3!-9z
| )x, )/-r +z
2x- 41 - 5z

135
_-

136 Algebraic Methods

' To multiPlY monomials, multiply their numerical


coefficients and mu]tiply lilee varia6|e
factors by addirug their exponent§, For example:

(-2db)(4dt;) : (-2)(4)(d . d)(b. b,)


: _8(i!+3)(bl+2)

:,8a'b^
' To divide monomials, divide their numerical coefficients
and divide /záe variable factots
by subtracting theil exponents. For example:

l4_1.t.r_/14)íL)i] )
2lx;.j lzt/\*'/\y'J
/?\
-l;ll] ,(l)

=?r,
3

' To multiply a polynomial by a monomial, multiply


each term of the polynomial by the
monomial and add the resulting products. For exan,ple,

5x(3x -! + 2) : 5x(3x) + 5x( )' + 5x(2)


: |5x2 - 5x1 i 10x
The expression -(a - b) can be interpreted as ''take the opposite ofwhatever
is insitle
the parentheses." The result is b a sínce
,(a- b) _ ,1(a_ b\

: (-1)a + (-1)( b)

:-á+borb-a
' To divide a polynomial by a monomial, <livide each
term of tbe polvnomial by rhe
monomial and add the resulting quotients. For example:

Ox+l)
_-_ 6x l5 :
J33 zx l i

Multiplying Binomials Using FoiI


To find the product of two binomials, write them next
to each other and then
the products of their first, Outer, 1nner, and ást
pairs of terms.

EXAN/PLE
N/ultiply (2X + 1) by k- 5).
Polynomials and AlgebraíC Fractions 1g7
Solution
For the product (2x + I)(x
-
5),2xand x are the first pair of terms; 2x and 5 are
rhe Outermost pairs of terms; 1 and x are the 1nnermost terms; 1 and
terms of the binomials. Thus:
-5 are the Zast
FoIL
(2x + l)(x - 5) : (2x)(x) + (2x)(- 5) + (1)(}) + (1)(- 5)

= 2t .l [-l0x- x] -5
:2t - 9r -5
froducts of Special Pairs
of Binomials
! \T problems may involve
:rese special products:
TIp
-
; b)(a + b), (a + b)2, and (a - b)'.
Time Saver
You can save some time if you memorize and learn to recognize
when these special multiplication rules can be applied.
_XA|VPLE
Express 3XX ' (a - b)(a + b\: d' bz
k + 3) as a binomial.
' (d + b)'= (a + b)(a + b): d' + 2ab + b,
. (a, t1' : @ - b)(a _ b) = az _ 2db + b,
3llution
(x+3)(x-3) (r)'- (3)'
t -9
_,ÁMPLE
Express (2y ,]
X2y + 1)as a binomial.

:: ltion
(27 - l)(21 + 1) _- (2st)'
- O)'
4],-I

:,.-rMPLE
: x- Y)' -8- yY : 28, what is the value of xy?

:_ ,tion
' \
iuare each binomial:
(x+1)'- (x- 1)2 - )a
(x2 -l 2x1 4- 72) - (x1 - 2r! + í) :28
138 Algebraic Methods

' 'Ü7rite the first squared binomial without the parentheses. Then remove the second
set of Parentheses by changing the sign ofeach term inside the parentheses to its
opposite.
x2 l2x7 l !'- r'-l 2x1 -f:29
. Combine like terms. Adding r'and -x2 gives 0, as does adding72 aná -f .

The sum of 2x7 and 2xl is 4x1, The result is


:
,:t:,
4x7 29

Combining Algebraic Fractions


Alge§raic fractions are combined in much the same way as fractions in arithmetic.

EXAMPLE
ww
Write
z- 3
as a single fraction.

SoIution
The LCD of 2 and,3 is 6. Change each fraction into an equivalent fraction that has
6 as its denominator_

a _a
23
:,,(r- 1g
_3w _2w
66
3u - 2tu

a
6

EXAMPLE
ft h=--Y .,whatish + 1intermsof xandy?
X-y

SoIution
. Add l on both .ides of the given equarion:

h-l: ! lt
x- JJ

' On the right side of the equation, replace l with {-1;


X- 1

h+t: t +'-!
I- tX- l
i Polynomíals and AlgebraíC Fractions 139
. \7rite the sum of the numerators orrer the common denominator:

h+7:!+x-!: X
x- ! X- J/

The Sum and Difference



of Two Reciprocals Tlp ////
l</
The formulas for the sum and the difference Reciprocal Rules
oF the reciprocals of two nonzero numbers, x and
1, lf x and y are not 0, then
are worth remembering: 1 l í+y l 1_7-x
XlXy and
X))Xl

=XAMPLE
lft: 11
+- , then 1
rs What is
t
in terms of r and s?

Solution
l l r ls 1 rs
Since r: hen
f s rs t rls
t

-,
140 Algebraic Methods

Lesson 4-3 Tune-Up Exercises


Multip!e-Choice
l.
2Ob' - 8b
( p : -! a * b, rhen, in rms
4b
5,
' a-b
and re

(A) 5b' 2b of a and b, 1 - p:


(B) 5b3 2 a
(C) 5b' 8b
(il b-a
(D) 5b' - z b
(E) 5b_2 (B) ,

(39), + 2(39)(6I) + (61), : a


(C)a-o ,

(A) 9,099
(B) 9,909 (D) b ,
(C) 9,990
(D) 10,000
(E) 10,001
(E)a+b
a-0,

3. (4a1- a-3)(4b-2b, b): - b)' + (a + b)' :


If (a 24, then
(A) 2a b d+bz:
(B) 2ab -2b (^) 4
(C) 5ab - b (B) 12
(D) 5ab - 3b (C) 16
(E) 3ab (D) 18
(E) It cannot be determined from the
4. If (x - !)1 : 50 and x7: 7, what is the information given.
value of 12 i 72?
(A) 8 7, a- b:
If pand al b: b, then
(B) 36 az-bz_
(C) 43 (A) ?h
(D)
(E)
57
64
(s) p, - k'
(C) p+ k
)
D\L
k'
(E) k'- ?'
8. Ifr: t* 2ands* 2: t,thenrs=
(A) r'
(B) 4
(C) t'-4
(D) r' 4t+4
(E) t2 + 4t+ 4
Polynomlals and Atgebraic Fractions 141
-w4lich
9. statement is true for all reai values
of x and 7?
14. For x, 7 } 0, which ,|
expression
is equal ro '
(A) (x+ y)2: * + 1l
?
7,
(B) x2 ] x2: x. __L

X!
(C):_:4
)x+2
(A) x+ 7
2,
(D) (3y7z = 6r. xl l
_-_,
(B)
(E) x5-x3:xz Xt

10. If (p - q'l' = 25 and pq = 14. what is the G) =u-


value ot (p -f q)r?
x+ 1
(A) 25 (D) " +l
(B) 36 1x
(C) 53
(D) 64
(E)]
x!
(E) 81
, .2

,,o[#-,)(*-,; : rs. rrlp+]l


\ h)
:16,then r*i:
(A) 4
bi - l, then h- (B) 8
(A) o.t (C) 12
(B) 0.01 (D) 14
(C) 0.001 (E) 18
(D) 0.0001
aY l"
(E) 0.000001 16. If;:1__ , then a
blt 4
l2.. : x2 -t 2x -t b, then
If (x + 5)(x J- p)
(A) )í
(A) p:Sandb:5 !-x
(B) p: -g and k - í5
(C)p:eandb:-15 G) l-r
(D) p: 3 and b: 15
(E) p: -l and k: ,n,
\\,l
t
-I5 X-1
]3. For what va]ue of p ls (x
- 2)(x + 2) : (D) {+t
x(x - p)? ]
(^) -4
(B) 0
(E) "/
)/- X
(c) 2

@)4
@) -4x
'l42 AlgebraíC Methods

Grid-ln
I. If (37 -I)(27 + k) = a!' + b7 _5 for all If 4x1 * 20x ,t r - (2x + s)'for all
values ofn what is the va]ue of a i h values ofx, what is the value of r - r?
t ᧠§§}d

Overview
&-&
Factofing feyers€s multiplication.

Operation Example
Multiplication 2(x+37):2x+67
Factoring 2x+67:2(x+J1)
There are three basic types of factoring
that you need to know for the SAT:
Factoring out a common monomial factor,
as in
-6x:2x(x- j and a7-b7:|ta.bl
_ .4r'
Factoring a quadratic trinomial u.ing
tÉ,.uá.. JrPOti, in
_ x2_x_6=(x-aÍll, +z) and ",
:' - 2r+ 1 : (Jí - 1)(x- 1) = (Jí- 1)'
Factoring the difference between tq,o
squares using the rule
a2- b2=(a+ b)(a- b)

:actoring
a Polynomial by Removing a Common Factor
.i all the terms of a polynomial have factors
in common, the polynomial can be
:,lctored by using the reverse of
:or example, in
the distributive l",-;;;;';h.se common factors.

+ 76x = 8Á3x2 + 2)
24x3
:ris the Greatest Common Factor (GCF)
of 24x3 and,i6rsince 8 is the GCF of 24
] j: x is the greatest power ofthatyariaSt
]d ,rr", i, ."r"i"ed in both 24xj and
O;*an.""9factor rhat.o...rponds to 8xcan
_

Ú;;;il Ü;ividing24xj .| 16x


24XJ +
:8"24x.*É=3x1
l6X _ 16x
+2
',
8"
ru^can check that the factorization
is correct by multiplying 3x' + 2 by 8r and
.ri$.ing that the product is 24x' + I6x.

_,sing Factoring to lsolate Variables


in Equations
-: may.be necessary to use áctoring to help isolate a variable
in an equation in which
:.;ms involving the variable .".r.roi
b..ombined lrr,o t..-.
".ingt.
r,GMPLE
l aX c = bX + d, What is x in terms oí a, b, c, and d?

143
144 AlgebraiC Methads

SoIution
Isolate terms involving x on the same side of the equation.
. On each side of the equation add c and subtract ár:
ax^ bx: cl d
. Factor out rc from the left side of the equation:

x(a-b):c-ld
. Divide both sides ofthe new equation by the coefficient ofx
t
x(é--'b) _cic!
4-="6 a- b
c*d
ab

Factoring a Quadratic Trinomia|


Á quadratic trinomial like r' - 7 x 1- 12 contains 12 as well as r. Quadratic trinomials
that appear on the SAI
can be factored as the product of two binomials by reversing
the FOIL multiplication process.
. Think: "Y/hat two integers when multiplied together give a 12 and when added
together give -7?"
. Recall that, since the product ofthese integers ts positiue 12, the two integers must havc
the same sign. Hence, the integers are limited to the lollowing pairs of factors of * 12:

1and 12; -1 and -12


2 and 6: 2 and -6
3 and 4: -3 and -4
. Choose -3 and -4 as the factors oF 12 since they add up to -7. Thus,

x2 * 12: (x
- 7x 3)(x - 4)
. Use FO]L to check that the product (x - 3)(x 4) k t - 7x,l 12.

EXAMPLE
Factor n' 5n 14.

SoIution
Find two integers that when multiplied together give - 14 and when added
give -5. The two iactors of - 14 must have different signs since their produ_-:
negative. Since (+2)(-/) : -I4 and 2 + (_7): 5, the factors of _14 r-c:
looking for are 1-2 and - 7. Thus;
r? 5n 14: (n+ 2)(n-7)
Factoring 145
Factoring the Difference Between Two Squares
Sínce (a +.b)(a, b) : d - F, any binomial of the form d
as (a -l b) times (a - b).
- F can be rewritten

é - F - (a+ b)(a- b)
This means that the difference between two squares can be factored as the product of
the sum and difference of the quantities that are being squared. Here are some
examples in which this factoring rule is used:

' !2- 76- (l + 4)(l 4)


. x2 - - (x + 0.5)(x 0.r)
0.25
. l00 -1;a - (10 xr)(10 * x')

Factoring Completely
To factor a polynomial into factors that cannot be further factored, it may be necessary
to use mof€ than one factoring technique. For example:
. 3f 75 :
- 25) : 3(t + 5)(t - 5)
3(t'
' | - 6f + 9t : í(t2 6t+ 91 _ 11, 3)(t _ 3)or t(t _ 3)2

Using Factoring to Simplify Algebraic Fractions


1b simpliS, an algebraic fraction, áctor the numerator and the denominator. Then
:ancel any áctor that is in both the numerator and the denominator ofthe fraction
,ince any nonzero quantiry divided by itselfis 1.

-XA|VPLE
^ 2b- 2a
5 mplity D .
a, _D,

::lution
' Filctof the numefator and the denominator: 2b 2a _ 2(b-a)
2
b' (a + b)(a - b)

' b- Z|-(a , b)l


Rervrite aas,(.a - b):
(a+b)(a-b)
' L_-ancel a , b since it is a factor of the numelator
1

: nd a factor of the denominaror: 2(;---á)


(a + b)(i,--b)
-2
- r+b
'l46 Algebraic Methods

Lesson 4-4 Tune-Up Exercises


Multiple-Choice
, 4x,f 4v :
12 12 4. If -:- 1 and x * *1 what is
LX - lJl x+ 1
x in terms of 7?
)
(A) yi1
X r] ,

(B) 2(x - si 11
ll1
(C) ,
Xy (B) l_
'2(x-l) !
(l \ (C)
y

íDt 2l , ]-I
,

\* + 1,)
I

E)2X- tol -l
] +1
1
(E) 1-- 1

2. The sum ofa",b


=! ,,
b
ls
1
^nd,
o'_b, 5. Ilx': 12 l2rs* s2,y2: rz-,z,
(Á)
1 x>O,and7>0.thenj=
a-b /+J
t
a (A)
r-s
-r,-r!
(B)
ab
]I
(C)
b (B) .i-
Y r-J
ah
a-tb r-s
(D) (C)
a-b
ab
- rrs2)+s
(E)
a-b (D)

3. If ax * x2 : !' - ay what is a in terms G) J;+J


of x and 1?
(A) r x 6. Uh:" ,'_ 1 ,.'-1',wharisxin
'+^
(B) r-"y x{1 x-l
(C) x+ y terms of /?
),2-ry b
x (A) -2
íD.)-
x+ y
(B) 2h+ 1

x2,1tv (C)2h-I
(E)-x-
J/ lh
(D) ./:

@
"Ei
Factoring 147
7. If ax2 - bx: aJ/2 + b7, then ! = 9, If rIj=i,th.n 8r*8r _
,l
X-l a I5x - 157
(Á) _..
x- J (A) €
(B)
' x +!l 45

X-t .
(B) L
(C) 15
,+ l
X+ y rc)L
(D)-X-! 16

{ \UZ)
,!
(E)

íF.) 1
If a* b and
a' - b'
8.
- a -r o, then 5
9
what is the value of a
- b? 10. Ifx': h + l.rh.n
' "n-l1 1

x2
(^) k
(A) +
J (B) h,
(B) 3
(C) k+2
(C) 9
(D) b-2
(D) 12
(E) 4h, + 2h
(E) It cannot be determined from the
information given,
i,i]§§ j}
&

Quadratic Equations
& "3*
q*a}

oVERVlEW
A quadratic equation is an equation in which the greatest exponent
of the variable is 2, as in l + 3, 10 : 0. A quadratic equation
has two roots, which can be found by breaking down the quadratic
equation into rwo first-degree equations.

Zero-Product Rule
If the product of two or more numbers is 0, at least one of these numbers is 0.

EXAMPLE
For What Values ofx is E 1XX + 3) : o?

SoIution
Since(x- 1)(J.+ 3) : 0,eitherx_ 1 : 0orr+ 3 - 0.
.Ifx- 1:0,thenx:1.
.Ifx+3:0,thenx:-3.
The possible values of xare 1 and -3.

Solving a Quadratic Equation by Factoring


The two roots ofa quadratic equation may oí may not be equal. For example, th:
equation (x 3)' - 0 has a double root of r: 3. The quadratic equation:r, = .
however, has two unequal roots, tr: 3 and x: 3. More complicated quadra:,:
equations on the SAT can be solved by áctoring the quadratic expression.

EXAMPLE
Solve X'+ 2x = olor x.

SoIution
. Factor the left side olthe quadratic equation: x2 l2x: (,

x(x -t 2) _
.
1l
Form two first-degree equations by setting each
áctor equa1 to 0: x:0 or x*2-
. Solve each first-degree equation; x:0 or x=
The two roots are 0 and -2.
148
Quadrati} Equations'l49

If a quadratic equation does not have all its nonzefo terms on the same side of the
equation, you must put the equation into this form before áctoring.

FXAMPLE
Solve x' + 3X = 10 for X.

SoIution
To rewrite the quadratic equation so that all the nonzero terms are on the same side;

' Subtract 10 from both sides of x' t 3x: 10: - 16 _


r' + 35 6

' Factor the quadratic polynomial: (x + 5)(x - 2) : 0

' Set each factor equal to 0: x+5:0 or x-2-0


' Solve each equation: x: -5 oí x_ 2

] he two roots are 5 and2.

:ou can check that x: -5 and, x: 2 are the roots by plugging each value
:ro :r2 ]- 3x : 10 and veri$zing that the left side then equals 10, the right side.

_KA|VPLE
]f 4 iS a root ofx' - X W : 0, What is the Value ol w2

: r Ution

' rce 4 is a root of the given equation, replacing x with 4 in that equarion gives an
.:..ration that can be used ,.
'.\i?' [_ . : o
16-4-w:0
12- u-0
Iz , 11)

,-adratic Equations with Equa| or No Boots


:uadratic equation may have equal roots. lf x2 - 2x ]- 1 : 0, then
(r- 1) (x- :0
rW?
1)
x-I:0 or x-1:0
x:1 Every quadratic
equation on the
-.:.. the equation x2
- 2x + 1 : 0 has a double root of 1. SAT has two real
roots, but the
: '.lugh a quadratic equation may have no real roots, as in 12 ] 1 : 0, on the roots may be
- . all quadratic equations have real roots. equal.
'l50 AlgebraiC Methads

Lesson 4-5 Tune-Up Exercises


Multip!e-Choice
1. The fraction is not defined 6, ll a and 7 are positive numberr. w2 - 2w
:
(x+ 7)(x- 3)
- 0.:nd 2y7 r 0, :
when x ;
(A) 2 "'' 2
(B) 7 or -3 (B) +
(C) -7 or 3 3
(D) (C)
-7 or -3 1

(E) 2or3 (D) 2


2

2. If L:2a. then aeouals (E) 4


)'
(A) 0 or -2 7. For which o[ rhe lollowing equarions is
(B) 0or2 x: -2aroot?
(C) 0 or ,4
I.?_ r.-O
(D) 0or4
(E) 2 or 22 IL *+4_0
III.-t'+4x+4-0
3, If (s - 3)' : 0, what is the value of (A) I only
(s+3)(s+5)? (B) II only
(A) 48 (C) III only
(B) 24 (D) I and ]II
(C) 15 (E) II and III
(D) 0
(E) It cannot be determined from the
inlormation given. 8. If|=^.rhenx:
3

4 If a(x - 1) : 0, then which oíthe (A) 0 ol -3


following statements is (are) always true? (B) 3or 3
(C) 3 only
I. a:0 (D) 0 only
II. 1: 6 (E) 0or3
II]. ,. - y
(A) I and II only c)
By how much does the sum ofthe roo:.
(B) I and III only the equation (x + 1)(x - 3) : 0 ex.e.:
(C) II and III only the product of its roots ?
(D) I, II, and III (A) 1
(E) None (B) 2
(C)
- 1) :
3
5. IF (x - 1)' - (" 0, then,
(D) 4
(A) ,- 0 or x:2 (E) 5
(B) ,: 0 orx:1
(C) -1 or x:2
(D) ": or x:2
(E) ":
1

í: 0 orx: -1
Quadratic Equations 151
'

10. If x' - 63x - 64 : O and ? and, n are 11. If r> 0 and r' : 6.25r'*2,then r:
integers such that 1r -x, which ofthe
following CANNOT be a value forp? (A) +
)
(ö -a
(B) -4 @)+
(C) -t (O;
(D) 4
(E) 64
@)t
@)1

Grid-ln
1. If (4p + 1)' : 91 and p > 0, what is a 3, By what amount does the sum of the roots
possible value ofp? exceed the product of the roots of the
equation (x - 5)(x + 2) : 6l
2. If (x - 1)(" - 3): - 1, what is a
possible value of r?
§§
Systems of Equations
-&_'l
,}jl9 laa
-_il-:§|
ti.;§§
'§i]
oVERVlEW
Á system of equations is a set of equations whose solution makes
each ofthe equations true at the same time. SAT questions involving
systems of two equations with two different letters can usually be
solved by:

. substituting the solution.ofone equation into the other equation to


eliminate one of the letters in that equation; or

' addinB or subtracting corresponding sides ofthe two equations so that an


equation with only one letter results.

Solving a System of Two Equations by Substitution


Ifone ofthe equations in a system oítwo equations has the form
letter - math expression

the same letter in the other equation can be replaced with that math expression, For
example, if
3x: 2 and 5J + 3x: 7

we can find the value of7 by replacing 3x w|th 2 in the second equation to obtain
51 + 2 - 7. Hence, 51 : 5, so 1: I.

EXAMPLE
líy:x - 3andx +y = 18, What is the Va|ue ol y X?

Solution
. Substitute 2x - 3 for lin the equation x 1- 7: 13. Then solve for x,

x+ 1, -- 18
}
x+2x-3=1,8
3x-3-18
)x, LI
2I
X
J. -/

152
syslems of Equations 153
. Find_the corresponding value of7 by substitutin g 7 for x ineither oF the original
equatrons.
xi 7:19
7 + l: l8
!: 1I
' The value of7 - xis lt , 7 : 4.
EXAMPLE
lf 8b = 40 and bc : 1, What is the Value of c?

SoIution
Use the first equation to help eliminate á in the
second equation.
. Since 8á : 411, 5 : '" = s
8-
. Substitute 5 for bin the second equation:
l
5c:1, so r=
,
rMMPLE
|íu - 3y = 6 andy - 5:3, y, What isthe Value ofx?

ScIution

' Since the second equation does not depend on x,


solve it for7:

7-5=3-7
2l: 8
]:r:* 8

' iubrritute 4 For yin the firsr equation:


2x- J1,: 6
2x - 3(4) : 6
2x- 12: 6
18

18
)-
:: .,ing a System of Equations by Combining
, :,responding Sides
. be
::.lr,
:.l\- possible to solve a system of two equations by writing
De posslb,le
one equation above
rher equation and then adding or subtracting
, .ne letter ,h. iik. ,..ri, in each column so
is eliminated.
'l54 Aloebraic Methods

EXAMPLE
lí x 2y:5 andX + 2y : 11, What is the Value oíX?

Solution
Since the numerical coefficients of;l in the two equations are opposites, adding the
corresponding sides of the equations will eliminate 7.
x 27- 5
1- xl2y_!|
2x+0-16
16
= ó
': 2
The value of x is 8.
Before the equations are combined in a system of equations, it may be necessary to
multiply one or both equations by a number that will eliminate one of the two letters
when the two equations are added together.

EXAMPLE
|t 2a : b + 7 and 5a :
,í5,
2b + What is the Value oí a?

SoIution

. Rewrite each equation so that all letters are on the same side:
2a:b+7 -+ 2a-b:7
5a:2b+15 J 5a-2b-15
' Multiply the first equation by -2, so that the coefficient of á becomes ] 2.
Then add the two equations to eliminate á:
2a- b:7 ) -4a + 2b: -14
5a-2b:75 -+ 5a - 2b: 15

The value of a ís 1 .

Solving Other Types of Systems of Equations


If a system of equations has more letters than equations, you may be asked to
for some combination oI letters.

EXAMPLE
lí 2r : s and 24t : 3s, What is r in terms oí í?
systerrs of Equations 155
Solutíon
Since the question asks for rin terms of l, work toward eliminating
. Substitute 2r for s in the second equation:
r.

24t: 3s = 3(2r) = 5,
. Solve for r in the equation : 24t = 6r
24t _ 6r
66
4t: r
Hence, r : 4t,

EXAMPLE
lf ab - 3 = 12 and 2bc = 5, What is the Value of 3?

Solution
\ince rhe question ask
Po, !, youmust eliminate
c' á.

' ,Find the value of ab in the fr,rstequation. Since ab : 12, then ab: 15.
- 3
' To eliminate á divide corresponding
sides of ab: ],5 and2bc: 5:
ab _15
lOC )
ab
2br
L=Z
2c
:olve the resulting equation for 4:
c

z(!\ : ,,^,
lz, )
a-
c
Lc value ot ls 6.
-C
_ .

at strategies 12 and 13 on pa8es


.-"r}rff- 35 and, 36, respectively, for more
156 Algebraic Methods

Lesson 4-6 Tune-Up Exercises


Multiple-Choice
I. If2x - 37: 11 znd3x l 15 : 0, what 6. If 5x+ y: 19and x- 37: 7,
is the value ofy? tnenx r_r,|_
(A) -7 (A) 4
(B) (B) -1
-5 (C) 3
(O+ (D)4
J
(D) 3 (E) It cannot be determined from the
(E) 10 information given.

2. If 2a:3bané4a+ b:2l,then b: 7. Ifx-9:27andx+3:5y


(A) what is the value of .d
1
(B) 3 (^) -2
(|C) 4 (B) 4
(D) 7 (C) 11
(E) 8 (D) 15
(E) 17
3. If 2p+ q:ttandp,l 2q:13,then
p+ q: tr1+ 1:].nd_L__l: ]
8.
x j 4""" x 1 4'
(A) 6 then r :
(B) 8
(C) 9 @)+
(D) 12
(E) 18 G)+
(C)
4. If m + ? + b: 70, p: 2m, and É:2p, (D) 2
1

then m : (E) 4
(A) 2
(B) 5 9, If5a+ 3b: 35 and a, -] , what is the
(C) 7 value of a?
(D) 10 1l
(E) 14 (A) +:
)
(B)i
,7

5. If x y: 3 and x 1- 1: 5, what is the


value of7? (C) 5
(^) -4 (D) 7
(B) 2
(C) -1 (E) 9
(D) 1
(E) 2
sysíems of Equations 157

10. IfX =6,1 = 4 and x: 36, what is ',-L." = 3and o ' ' =
la 14. If 5,whatis
tlre value of ez? r_t
the va]ue of ! ?
t
6)+ (A) 1
'2
@)+ (B) +
(C) 2 )
(C) 4
(D) 4 (D8
(E) 6
(E) 16

: 15. If 3xl7= candx + l: b, what is the


11. If 4r + 7s 23 and r - 2s = 17 then value of r in terms of r and /?
3r* 3s:
(A) 8 ,., c-
(Á) b

(B) 24 3

B) c-2
(C) 32 b
(D) 40
(E) 48
(C) b- c
- a
-^ b
n_=3andr?- 3
rq=12,
then r : (o1 b-c
2
(ö -1
(B) 1
@) !1
(C) 2
(D) 4 16. If a + b: 1l and a - b:7,
(E) It cannot be determined from the then ab :
information given.
(A) 6
If (a + b)': 9 and (a
(B) 8
13.
- b)z: 49,what (C) 10
is the value of az b2? l (D) 12
(A) 17 (E) 18
(B) 20 x* z= 7
(C) 29 x-f 1=3
(D) 58
(E) 116 '- != 6

17, For the above system of three


equations, x :
(A) ;
(B) 6
(C) 7
(D) 8
(E) 9
158 Algebraic Methods
l
a: 4c 19. ll r8 : 5 and r' - :, what is the va]ue
t
of r in terms of t?
a: 5e
(A) +/
3
18. For the system ofequations above, if
r * 0, what is the value of r? tB) ?r
)a
(C)5-..I
(A)+
! (D) 3+'t
,5
íB\
(E) +
t)
G)1
(D) 1
20. If ! : 9 ! : 2,whatisthe
(E) 20 b7c5 ^nd
valrc of 3b i cintermsof a?

(A\ 1za
(B) 9a
(C) 8a
(D) 6a
(E) 4a

Grid- ln
1. If5 sips i
4 gulps :
1 glass and 13 sips 3, John and Sara each bought the same rype
-l 7 gulps :
2 glasses. how many sips of pen and notebook in the school
equal a gulp? bookstore, which does not charge sales
tax. John paid $5.55 for two pens and
2. If2a:9 - b and three notebooks, and Sara paid $3.50 For
4a= 3b - 12,whatisthe one pen and two notebooks. How much
value of a? does the school bookstore charge for one
notebook?
i.§9 § §X

Overview
&*w
üneat inequalities such as 2x 3 < 7 are solved
- by isolatine
letter in much rhe same way thaL linear (fiil.*j'.;;;;;:
the
;;;
::P1:l"|,r'r'"g
or divitJing borh sides ," i"!q;"li;TY'.'".8"
tlve number reverses rhe direcrion of rhe "e
inequaliry r,gn. Éo,."".pT.,

5>J,so5 <l,-2l<3 < I_2)

Any,general property of inequalities that is


stated in this lesson
Iof tne ls less than' ({) relarion is also
true for each of the other
(<, ) and )) inequality relations.

Some Properties of lnequalities


,{n equivalent inequaliry results when

' The same numbe! positive or negative, is
added or subtracted on both
TlP ív
sides of the inequaliry Fo. .*amp'i., The direction ofthe inequaliry
sign also gers reversed when
6>4, so 6+2>4+2 comparing the reciprocals of
two positive numbe rs x and y:
6>4, so 6-2>4-2 lf , > a!.
y,then1
Xy
' Both sides of the inequality are multiplied
or divided by the same
positiue number, For example:

6>4, so 6X2>4X2
6>4, so Íri
multiplied or divided by rhe salne negatiue
Tl:j:::T*l:.q:,]i,y:l.:
n umDer and the direcrion
of the inequality is -
reversed. Fo, erampl.,
6>4, so 6X(-2)<4X(-2)
6> 4, so 6 _4

159
160 Aloebraic Methods

Solving Linear lnequalities


To solve a linear inequality, isolate the letter by performing the same arithmetic
operation on both sides of the inequaliry. Remember to reyerse the direction of the
inequality sign whenever multiplying or dividing the inequality by a negative number.

EXAMPLE
lí 3x - 2 < 10, find x.

Solution
3x-2<10
3x< 12

x{ -il2
,.;
The solution consists ofall real numbers less than 4.

EMMPLE ]

What is the greaíesíinteger value of x such that 1 - 2x > 6? i

So|ution
I

l
| -2x>6 l
_2x> 5
Reverse the inequality: ,a+ l
Since ris less than integer value of xis -3,
!,rhrEreatest l
Solving Combined lnequalities l

ff*:::--;fi:-'fi*-* |
' Add7 to each member of the inequalitY:
_, -:: ,:'_ r'*, ==r'-
rl

I
Algebraic lnequalíties 161

' Divide each member of the inequa]iry bv 3, 2 < 3x < 15


33j
:=
)
x <5

The solution consists ofall real numbers greater than or equa.l to l and less than or
equal to 5. 3

EXAMPLE
lí3 <x + 1 < 8and 2 <y <9, Which oí thé following best describes the range of values
oty X?

(A) -7 <y -X<5


(B\ -5<y - X<7
(c)O<y-X<2
(D|2<y-X<7
(E)2<y-x<9

SoIutjon
First find the upper and lower limits ofxand7.If3<x+ 1< 8, then3
( -8 1so 21 x 17. This means that the lower limitoí xis2 and the upper limit
- 1(y
1t < 9, the lower limit of 1is 2 and the upper limit is 9.
s 7, Since 2 <

' The up^per limit of1 - xis obtained by taking the difference between the upper
limit of7and the lower limit of xwhich is 9 : Z : l.

' fhe lower limit of7 - xis obtained by taking the difference between the lower
limit of,7 and the upper limit of x which is2_ 7: -5,

' Hence, -5<7- x<7.


l-he correct choice is (B),

3rdering Properties of lnequalities

'If a< bandbl c, then a { r. For example:

2<3and3<4, so 2<4
' IF a < b and x < 1, then a -t x 1 b-l 7. For example:
2<3and4<5, so 2+4<3+5
' If a< bandx) 7, then the relationship between a -| x and, b l7cannot be
determined until each of the letters is replaced by a specific number.
162 Algebraic Methods

Multiple-Choice
integer value of x that
1. lwihat is the largest integer value ofp that 5. 'w/hat is the smallest
satisfies the inequality 4 + 3p < p + I? satisfies the inequaliry 4 3r < 1?
'l
-
(A) -2 (A) -3
(B) -1 (B) -2
(C) 0 (C) -1
(D) 1 (D) 0
(E) 2 (E) 1

2. If -3 < 2x 1- 5 < 9, which of the If a> b ) c)


0, which of the following
statements must be true?
following CANNOT be a possible value
of x? ,l. .-a-c _ b-c
b-a b-a
(A) -2 II. ab} ac
(B) -1
(C) 0 III.! > !
(D) 1 ac
(E) 2 (A) I only
(B) II only
3. Iíthe sum ofa number and the original (C) III only
number increased by 5 is greater than 11, (D) I and II
which could be a possible value of the (E) II and III
number?
7. tl j1< t5ands: r* +. which oFthe
(§ -5
(B) following must be tfue?
-1
(C) 1 (A) ,< 5
(D) 3 (B) r< 18
(E) 4 (C) s< 9
(D) s< 20
4. IrO < a2 1
b, which of the following (E) s<49
statements is (are) always true?
must be
1.a1_
h 8. 'which of the following statements
a true when a2 1 b2 and, a and b are not 0?
II. aa 1 a?b ,,a--L) !7

]l1I.L<í
2
áa
l1
(§ I
b
only
II. ;a' ) h'
--=

(B) I and II Il1. (a + b)(a - b) <0


(C) II and III
(A) I only
(D) I and III
(B) II only
(E) I, Ii, and III
(C) III only
(D) I and II
(E) II and III
I
Algebraic lnequalities 163

9. For how many integer values of á is 10. If x1> Iand z10, which ofthe
b+3>0and1)2b-9? following statements must be true?
(A) Four I. x)z
(B) Five
(C) Six
lI. xyz < -I
(D) Seven ]lII.9-<]
(E) Eight (A) I only
(B) II only
(C) III oniy
(D) II and III
(E) None

Grid-ln
1, For what integer value of 1is y + J> 8 3, If2 < 2Ox - 13 ( 3, what is one
and,2y-3<7? possible value for x?

2. If 2 times an integer xis increased by 5, the ,I 1 1 I ,| 1,l 1

result is always greater than 16 and less 789108910n


'§7hat
than 29, is tlre least value of .rc? For the above inequality, what is the
greatest possible positive integer value
of n?
164 Algebrai) Methods

Answers to Chapter 4 Tune-Up Exercises


Lesson 4-1 Solution 2 The answer choices represent
possible values for x - 2. Add 2 to each
|VULTlPLE-CHoICE answer choice to obtain the possible values for
1. (B) if + 16 :8.2, then.r,6 : S,Z x. Then substitute each of these values into
: 3.2,so
5 ' \6 : 5_ 5 _ 3.2 : 1.8. the given equation until you find one that

2. (^) (9 4)(x + 4) :
works:
30
5(x+4)-30 3x-6:18
,30 3(6+2)-6:18
x,T4: -6 24-6:l8
,:i-_a
n
7. (B) If 2r+ l: y+ 14, subtracting 7 from
each side of the equation gives2x:14,so
3. (E) 2(1 + 5) : 3(u - 4) I4
2(6):3(w)+3(-4) X--
2
/

lZ - 3w 1),
12 :
-l 12 3ul (B) If jÉ: 36. then
/1

24: 3w 7
l1 : (36):')
24 -k
74
3
8:ul 1. :
Since -É 9. then
4. (B) --1 _I-3 7
,k:3(.9):27
1
3
7
- -z
3
(D) 111 l -x :
9, ,748-x ,t -x 14
-3 -_ -2 + 1

l, ,,
x_ jx l ax l
-|:x _ 14
888
3
x:-I(3): 3
Zr: t4
8
5. (B) Break down 5a - 2b: b -l I : 9 into gíz \ 8,
5a - 2b : l- -|-xl:
_

-(I4)
two equations; 9 and b + 7\8 ) 7
9. Solving the second equation for b gives b X -
:
1(]
8. Subsrirure 8 for bin the first equation:
5a-2(8):9 10. (B) If2i + 3t: 12and4s:36, the second
5a-16:9 equation can be used to eliminate s in the first
5a:9 + 16 : 2, =:
5a_25
equarion. S]nc.
f 18, replace

55 2s with 18 in the first equation. Then solve


a:5 for t
],8+3t:12
6, (C) Soktion 7: Solve 3x - 6: 18 for x - 2
3t:12-18
by diüding each member of the equation by 2:
3x618 3t:,6
-6
333 3
X-,Z.-lJ
Answers to Chapter 4 Tune-LJp Exerclses 165

(B)
ixtand r:54andx: l2,then
1t. |t r,: Add 5 to both sides;

: 3(j +k)-5+5=16+5
'4 i (12)t
3v+ h):21
.3 3UFb_21
>4 : 102t 33
A
j- k =7
54:9t 17. (C) According to the conditions of the
54 _2- problem,5k: k+ 5so5h_ le_ 5.
99' Since4h:5.k:5.
6:t 4
18. (D) Solyg { _ z/ íoíx a-ttel replaclng
12. (C) If
+ = 2, rhen x: l since
2 l
)
Hence, ^ l ,j7 wltn -:
yJ- .)_ 1L lI z-:)
. _ ,
x+)/:27
13. 6)Ir!=:7-t2,then7 _)_ )
3
2(7 l 2). Eliminare rhe parenrheses, and
then collecr all the Lerms involving y on the
1
-ax _ )7
),
same side of the equation.
!-2=2(1 +2)
:2l+4
!-2l:4+z
i (i,):,, ;,
-!:6,so!: x: i8
-
f*
6
)",
:L = !-:,set
-.L 1 .o j,,st = o
E)
^,
14. lF the cross-products
/4 Insread of first solving lor x you could also solve
equal and üen solve üe resuJting equarion.
direcrly for 1 i
": ,l]lL "l:
Since ]r. solve
2t _ t+ 3 ". .
74 for
)
by dividing both sides of the equarion
4(4t) :7 (t + 3) ix
87: 71l + 2I
,
Dy Ji
81 - 77: 21
J l,|
t: 2l \2 ):27
l5. (E) lf 1: 4. theny = 3(4) : 12, so
ő3
) !"=
3l: 3(12)
36 : 2
q

I6, (B) Tieat "j -| É'as a single variable and


solve for it in tJre usual way:
16+U+b)
-v+ k)
3(j+ k) - 5: ],6
166 AlgebraiC Methads

19. (D) Sincel ): , isolare rhe lerrer b;


(8) If 7 (a + b)
:
- 4(a + b) - 24, then
Jp
]
3(a + b) 24, so
ai
,)4
b- .
su bt,"., ing ] From each side olt he equa t ion: 8
a,
-

312 3. (2ll2) 1- x 2x- 3x:6x- 1,then


If
p-33 l - 6x:6x 1,so 1 * 1 :6r* 6;and
Eliminate the fractions in this equation by ) : l2x, fhut. x : ?. Crid in as 2l12.
12
cross-multiplying:
32 l. (l0/3) ln rhe e.luation p=Y I p: q

?3 í20)
andá- 20.rhenq c,
5
2p: 9 orq,,l = l00,

l-;
9
so a _- 1!.0
9 lJl(ln8 tne Posltlve square

20. (E) \{/rite the equation


369 root ofborh rides ofrhe equation gives
4x,y
two equations: /l00 l0
j 6 ,3 _9 ,,9 3
4x4 1 Grid irr as 10i3.
Solve the first equation for x:
5. (0.75) Since 60% of r is equal to of 45,
36 0.r;0 r : l,l(x) ' *5 0.q5s.
so/o

Z-; 0.601 0.45s


Hence:

JX-z+
0.60 0.60
x 24
:8 r:
), 0.75s

Solve the second equation for 7:


r 0.75s
JJ
3:9
41 !: 0.75
J
37:36 Grid-in0.75 or 3l4.
y
,3
'::12
76
Lesson 4-2
Hence,
xi7:3+12:20 MULTlPLE_CHoICE
1 . (B) Dividing each member of the gi.,e:
GBlD-lN equation, 6 :2 x -| 47, 6y 2 wtll Ieave x + ).,

on rhe righr side ofthe equarion:


1. (614) If 2x- l - 11, then 2x: 12, so 6 )x 4v
-r- _,o, Jlnce
X.,_I) ,. JJ/ : lz, lhen 222
3: xi 2]
- + .4. Hence. "-
|1
j'Jy4 " 6
-i. Grid in as The value of x -l 21 is 3.

614 or 1.5.
Answers ta Chapter 4 Tune Up Fxerclses 167

(D) If a,b
(C) Multiplying each member of the given 8. )
, then
ab| b 3
equation.
) j:3, by 4 wiII make rhe ab =?
left side of t}re equation equal to 2a -| 2h
j-j 3
a
-_1
b
=?3
^(:)-],r ^(*): ^,
r/_ _ 11 a2
The value of 2a -f 2bis 12.
b3
5
-). (B) For the given equation, 2s - 3t : 3
3 r -
finding s in terms of r means solving
s,
The value of ! ir 2.
the equation for s by treating , as a constant, b3
\7ork toward isolating s by first adding 3 ron 9. (C) Since it is given that s * 3s is 2 more
each side of the equation: than r+ 3t, s -t 3s : (t 1 3t) i 2 or,
2s: 3t -| 3t .l equivalently, 4s : 4t 1 2, so 4s - 4t : 2.
:6t-s Dividing each member of the equation by
Nexr, add ron each side ofthe equation: 4e4t2
4 qlves --j -j_ -:. which simplifies
2sls:6t 44-
3s:6t ,or-r:1 2
,=9=Zt 1"
3 l0.
. ?+q 1

(E) If a + b: 5, then2(a + b) : 2(5), fractions by cross-muItiplying:


so2a* 2b : 1O,
r(p+q):1(1)
. Ir +|rl:
:::,
2c - 1)
.r."" 4(3), ,o
rp :l-r4
Hence, 2a + 2b f 2c: 10 12 : 22. p- 1-rq
(C) If x7 1- r:1, thenr7- !- zso
y-z 11. (B) Determine -t ..rr.. each Roman
numera.I expression is equal to 4a.
1 . I. Since a: 2b: jc, then a: jc, so
(D) If b (x + 2) : 60, then bx + 2fu : 4a:4(5c):20c
60. Since it is also given that fu : 15, Expression I is nor equal ro 4a.
bx+2b:& . II. Rewrite 4a as 2a 2a. Since al : 2b -
bX+2(I5):60 5 c, then 2a 2(2 b): 4 b and 2a : : 2(5 c)
bx+30:60 : 10 r, so
bx:60 - 30:30 4a:2a -f 2a: 4b í 10c
Expression II is equal to 4a.
(B) Since;' and 72 are both greater than or . III. Rewrite 4a as a l 3a. Since a : 2b
and, a - 5a then 3a : 3(5 c) : 15c.
equal to 0, the only yalues of x2 and, f for
rvhich 2r' i 312 :0 are 112 - !' : g.If
Hence,
..' :,/' - 0, then r : 7 : 0, so
i 27: : 4a aL 3a- )b- |5r
3x 3(0) + 2(0) 0
Expression III is equal to 4a.
Only Roman numeral expressions II and III
are equal to 4a.
168 AlgebraiC Methads

lz. (B) Since a !_!:_! !:!, eliminate 17. (D) If c: b + t, then b: r- 1. Hence,
32
?:4b+ 5- 4(c 1) + 5: 4c- 4+ 5
the fractions by cross-multipll4ng: :4c-l 1,
2(a+ b+ c): j(a+ b)
18. (1t) If 2p * r -I : 2r + p -| 1, then, after
2a+2bi2c:3a]-3b like terms are collected on the same side of
2c: (3a- 2a) + (3b,2b) the equation, ?: r + 2 where p and rare
+b given as positive integers.
a,l h
. I. Since 1 is 2 more than p and rcannot
r,
be consecutive integers. Hence, Roman
13. (E) If ux: 6 then , : 1, ,o
.
numeral choice I is false.
:
lD II. Since p r -l 2, p can be either odd
xz- l/\)
zl z (if r is odd) or even (if r is even). Hence,
z,l - l= Roman numeral choice iI is false.
\a ) a .III. Roman numeral choice IiI is also
14. (A) The value in cents of z nickels plus / false since r can be either even or odd.
dimes is 5z + 10/ which you are told is since none ofthe Roman numeral choices
equal to c cents. Hence, 5n 1- 10d: c or must be true, the correct choice is (A).
5n: c- ljd, so
n= cIOdc
- "=--2d GRID-|N
555 1. (8) Since (4 x : 4'x b' : 16 x b,,
b)'
15. (D) Since á : 2c and b : 5d, replace a 16Xa'X64:16Xb'
,db ' - ! ,- x, wirh 2c and
in rhe equarion !Áxa2X64:y6Xb'
a2 X 64: b'
replace bwith5á
^WXG{: \F
c2c aX8: b
d5d Hence, ú is 8 times as gíeat as
5c 2c a.

5d 5d (5) Rearrange the terms of 3a -| 3b - c :


5c-2c 40 to get (3a - c1 + 3b : 40. Because
1.J 3a- c: 5b,5b + 3b- 40 so,8b:4Oand
3c ,40
5d ':8:\,
To find the valle of in terms of r, multi- (18) Ifa :2x 1- 3 and b: 4x 7, when
) 3.

: , 3b 5a, x musr saris$, the equat;on


3(4x - 7) : J(2x -F 3). Removing
ply borh sicles o[ the equation by
srtL
th. r..in.n.rl nf i, Pafentheses makes 12x - 21
: 10x + 15.
'5 Collecting like terms gives 12x - 10x : 15
5 r3.) 5 *21 or 2x: 36 so x -- z:tg
36
3|r7):3-
c5
a:3'
16. (D) If Éx-4 : íb - l)x, removing
parentheses makes ár- 4: hx- x so 4
: -xor x:4
Answers to Chapter 4 Tune-Up Exerclses 169

*; : +: # 5. (B) Solution 1; Do the algebra. If


4, (5) Multiplying both sides
p: n- then
by 40 produces the eqivalent a-b
5x 1 87: J l. Subsrirure consecutive"qrr"tio.,
positive -,
integer values for x until you find one that a-b
makes 7 have a positive integer value. ab
31 -5x a,b a -b
8 _a-b-a
31 -5 26 a-b
J88 b

-l
31 -10 2I ab
88 h
Since ---:; is not one ofthe answer choices,
J88
31 -15 16
Stop!
a,b
eliminate the negative sign in the numeratol
by multiplying the numerator and the
Hence,x+!:3+2:5. denominator by - 1:

l
|-l\ -l
Lesson 4-3 D - |_|_
' \ t)a b
MULT|PLE-CHolcE _ (1)(-b) _ b .b
(-l)(a - b) aIb b-a
1. (D).\írite each term of the polynomial
numerator separately over the monomial solution 2: substitute numbers for the
denominator. Then divide powers of the letters. Let a: 3 and b: 2; then
same base by subtracting their exponents. 3"
20b3 - 8b 20b3 8b a- b 3_2
4b 4b 4b so 1-7:1_ 3: _2. V/hen you plug
: 5b3,| _ 2 in 3 for a and 2 for b in each of the answer
:5b' - 2 choices, you find that only choice (B)
(D) The given expression Q9)' + 2(39)(6l) produces -2.
+ (61)' has the form (B) If (a b)' + (a + b)' : 24,then
x2 * : (xt 1)2
2x1 + 71 Qr -?Ű 1- b1) t (az + 2"ab + b,) : 24
where x :
39 and 7: 61. Hence, 2a2 -l 2b' : 24
(39), + 2(39)(61,) + (61), : G9 + 6I), )a' ,2b2 _24
: (100)' 222
:10,000 d2 + b2: 12
3. (D) BeFore multiplying, simplify each 7. (A) Iía- 6: panda1- b: k, rhen
expression inside the parentheses: a2 _ b2: (a- b)(a+ b)
(4a + a - 3)(4b - 2b - b) - - 3)(4b -
(Sr 3b) : @)@
: - 3)(b)
(5a :ph
-5ab,3b (C) Rewrite s -l 2: ras s: t- 2.Then
+. (E) If (x- Ji' - : 7, then
50 and x1 ,5 : (1* 2)(t 2) - t' - 4.
G-l):r'-2r!+!':50
x'- 2(7) t !':50
X'-14+!1=50
x2+j,'_50+14
_64
17o Algebraic Methods
1l x+ !
9. (C) Examine each choice in turn: 14. (C) Use rne rolmuta -Xt i- - - x!
. 14; (x+1)': rú2 +./2 is false since (x * 7)2
Since
_ -2 )- )^, )- ^,2
(B) x' ._ x2 : x4 is false since x2 l x2 :
: 11
IXz + lX2 2xz,
-+-
X!
^
(C)
_
-)x+2 : 4 ts truesrnce
2, . xl y
is l he reclDrocal ot^ rhen
L1r+
l
y+7j-,
'x]
^l
2, -. x!
=22 11 X+ !
=4 -+*
X!
' (D) (]x.)' - 6xz is false since (3x)2 :
22-^"2_(\^"2 15. (D) Use the formula for the square of a
. (E) ,i - x3 : x2 is false since only like binomial to expand the left side oFthe given
terms can be subtracted. equation:

10. (E) You are given that (p q)2 25 and - :


: íe*!.]':16
?q 14. Use the formula for the square of a \ k)
binomial to expand (p q)', -
(?-q)':?'-2pq+q':25 k +,@(r-)+|+)' - l()
p)-2ll4)+q,:25
?'-|q':25+28:53 k,+2++
k, =16
Now use the formula fot (p + q)2:
(p+q)':?2+2?q+q' h,++ :16-2:14
: 53 + 2(l4)
k,
:53+28:81 1,6. (E) Change the right side of the given
/ ,- \/ ,, \ equation. 1: t {, into a single
l1. (D) l_a_ r|l_a+ ll = kx' _ l tfactton:
hl
[100 ](100 )
(0.01x - 1)(0.01x + 1) = kx2 - I aX
0.0001x'-I=hx'-! -:1_-
b7
since the coefficients of x on both sides of :1_,
the equation must be the same, á: 0.0001. l1
X
12. (E) Use FOIL to multiply the left side oí :y
the given equation: t
(x+5)(x+p):i*2xl,b L^
**5xl-px+5p=l+2x-lh Since 1 is the reciprocal of 1,
ab
Since 5x -F 7x must be equal to 2x,
1
: -3. Hence, L: l
k:5p: -15 : a l- x

13.
'(-3)
(D) Multiply on each side of the given
equation, (r 2)(x + 2) : x(x - p).The
result is
x' 4:x2-x?
Á
so4-_ xtandt-!,X
Answers to Chapter 4 Tune-LJp Exercises 171
GRlD_lN
2. (A) Combine the fractions and then
l. (19) Since simpli§,:
(3y - |)l27 1 h) :
a! r b7- 7 a , b a +b
and the producr of ih. lrrt r.rrn. of
the two binomial Factors is equal to the a-b'a'-b'or-b
conshnt term, (-1)(,é) :
.
-5, so,á = 5.
Now multiply the wo binomiai. tog.,h.r, r*ql
3y- l)Qr+ 5) = (3)Ql) + Gl)6) + (l)b,) + (-1)(5) (>vX, - a
:6J? + l51 -21 -5 1

:6J} + l3r -5 a-b


Since 3. (A) To solve ax l x2 : 72 -
a7 for a in
3.y, 1)(27.+ 5),: 61,1 + l31 _ J : a12 + b7 _ 5 terms of x and
7, isolate a on the left side of
equating the coefficients makes a : 6 and the equation:
b= 13, so
a+b:6+13:19 ax+ x' - !r- a!
2. (2o) ax l -, x2
a7 = 1,2

+20x!7:()x*s)2 a(X+ i9+ r7


)t): (l
: (2x + s)(2r + s) a(xt y) _ v ,lg)-rl
: (r-rl
:4x1
(2x.)(2 x) + (2x)(s) + (J)(2r)
* 2sx * 2sx
+ (r(' lx + 7)
: 4x' l s2 a: !- §
-| 4sx -1 l2
.1

since the coefficients ofx on each side ofthe Q) If # 1, then multiplying both
equation must be the same, 20 : 4s,
so s :
!:
5. Comparing the last terms of the sides of rhe equarion 6y
7gives x7 = x xl
+ )l, so x? -
x !. Hence, x(7 : - li :
polynomials on the two sides ofthe equadon l,
makes z : s2 : 52 :25. Hence, so

r-s:25-5=20 .. !
^_ ]-1--
Lesson 4-4
5. (B) Since
,.1ULT|PLE-CHolCE
x2: r1 l )6! 52: (7 | 5)2
and
1. (F) Factor the numefator and
denominator. Then divide out any fáctor
the
!' : ,. - ,z : (r ll)(, - .i
then
that is common to both the .r.r-..áto.
(rl s)(r'i ír-t 9
the denominator: "nd "
t lr is) (r-.l) (r
.4x - 47 4(x * y)
You are told that ) '
2x2 - 272 2(r' _ 1.) :r and JLl 0, so
x
4v)n) !
2V-A,í)G - ),)
'l72 AlgebraiC l,,,4ethads

6. (A) Simplifr each fraction, then add: 9. (D) Factor out the GCF from both the

í
),), í
numelator and denom inaLor of
-I -| 8ri 8s
;ct1 r-1 15x r5l
(x+1)(x-1)
I
(r + i)(x 1)
8r -]- 8s 8(r + s)
xi1 xl 15x - ] 5_y 15(). l)
:(x-1)+(r+1) 8 ._ rt,,
l\ X- ]
h &.5
)
:]-5"\
2
_2
5

7. (A) Collect the terms involving a on one side of 5


the given equation and the terms involving á on 10. (A) Factor the numelatof of the given
the opposite side ofthe equation: fraction, ;a as the diíference of nvo
- 1,
tlx' bx - a1,' * b1 squares:
ax'-ayz_bx+b1
l

Factor each side of the equation:


ax'- a1): bx+ bt
a(x'- f): b(x+ 7)
Divide each side of the equation by b aná
,' - l',
a x+ !
b ,'- !' Lesson 4-5
X+ j |VULT|PLE-CHoICE
(x+/(x t
1 1. (C) A fraction is NOT defined when
x- ! denominatorhas a value oF0, Thus, the

fraction (x
x2
is NOT defined ::
+ 7)(x 3)
8. (C) Solve tl-re given equation, o' b' - (x -)(x J) 0. u hich occurs ifx : -
-^ -
l
9
orx- 3.
a b, for a2 -
b2. Then solve the equation
2. (D) Multiplying both sides of the gir::
that results íor a - b.
.a)
equation. ? : 2a, by 2 gives a2 - 4a. -:
a2-b}
9 - al, b z
apply the zero-product rule, one side of ::T
A2 :9(a+ b)
equation must be 0. After 4 a is subtracted irt n
b2
(a+ - :9(a+ b)
both sides ofthe equátion, 22 4 a : 0, v,F-
b)(a b)
- 9(a1b)
a-b:',---i-9 can be factored x a(a - 4) : 0. Thus, ei:-
dr0
a - 0 or a - 4 - 0. Hence, aequals 0 or =
3. (A) Since 0' : 0,(s 3)' : 0 -eans
,- J 0, so s: J. Hence.
(l+ 3)(s+ 5) (3 + 3)(3 + 5) :
: (6)(8)
:48
Answers ta Chapter 4 Tune-lJp Exercises 173
4. (E) If a(x -
!) = 0, then eíther a : 0 or .
x - ! : 0 ar both factors are 0. Determine
III. Ifr' 1- 4x i 4: 0, then
whether each Roman numera] statement
(-2). + 4(-2) + 4 : 4- 8+2: 0
is Hence, -2 is a root of equation III.
always true.
. Only equation III has -2 as one of its roots.
L It may be the case that Jí y : 0 and
-
a * 0, so statement I is not always 8. (E} Il! =r,h.nx/= Jxso;rl .3x- 0.
tfue. 3
. Factoring x'- 3x: 0 gives x(x - 3) : 0.
II. Since 7 can be any number, provided
Thus, x: 0 or r: 3.
that a : 0 oí x :
!, statement II is not
necessarily true. 9. (E) If(r+ 1)(x- 3):0, thenx-| 1-0or
. III. It may be the case that a x, 3 : 0. Hence, x: - 1 or y = 3. The sum
= 0 and
x - ! + 0, so statement III is not olthese roots is - 1 -t 3 or 2, andtheir product
always true. is (- 1) X (3) : _3. Since
None of the Roman numeral statements is 2-(-3):2+3:5
always true. the sum of the roots of the equation exceeds
5. (D) Solution 1: If (x 1)' - (x 1)- o, - : the product of its roots by 5.
tlren lactoring dre left side of the equation
gives
(x-
1) t(x-
1) 1]- :0 10. (B) Ifx' - 63x - 64 : 0, then
which simplifies to (r - 1)(x - 2) : (x-64)(x+1):9
so x: 64 or r:
O.
Hence, x may be equal to 1 or 2. -1. lf pand zare integers
such that p" : .T, then either
Solution 2: Substitute the pair of va]ues
in 7' : 64 or fr :
each of the answer choices into the given - 1. Examine each answer choice in turn until
equation, (x - 1;r
- (x - 1) : 0, until vou you find a number that cannot be the value of
:
find a pair, x: 1 or x: 2, that works. PÍn either p" 64 or p, _
-1.
6. (E) IF u' - 2tu: 0, then w(w 2) : ' (A) If p : -8, then (-8) '' : 64, so
u:Ooía=2. - 0, so
n:2.
' 7 - 0,rhen yl)1 - l) = 0. soJi:
. (B) If p: -4, then there is no integer value
0or2v-t:0
'^Frr^ of z for which (-4)' : 64 or (_41,, = r.
-
' If27-you t
:0, then 1 :
1l. q) Isolate variable r by dividing
1

Since ,oíd, Ú', at 7 are positive, both sides


^r,
w:2and r: j. Hence: of the equation by r':

r' : 6-25r'*2
,|:- 6.25r'*2
(C) For each Roman numeral equation, :6.25rll+,),l
check whether -2 is a root. : 6.25r)
. I. If ?_ x: O,th..,2: x= _)-' ffi Using a calculator, divide both sides of the
X y x,If
üen
@ equation by 6.25. Since
;| _ 0,76 : r' ,
2) j /: Jo]re : o,4or?
, - z: -] x 5
Hence. -2 is not a roor of equation
. II. Ifx'-l 4: 0. rhen
[.

(-2)'+4:4+4*0
Hence, -2 is not a root oFequation II.
174 Algebraic Methods

GRlD-lN 3. (B) Add corresponding sides oF the two


given equations:
1. (2) If (4p + 1)1 -
81 and p > 0, the
expression inside the parentheses is either 9 2p+ q:11
or -9. Since7 > O,Iet 4p 1 1:9; then4/ + p+2q-13
: 8 and 1 : 2. A possible value ofp is 2. 3p + 3q = 24
2. - 1)(x- 3) - ,1
(2) If (x 1, then
:
Dividing each member of 3p + 3q
x2 - 4x + 3 :
24 by
so x2 4x 1- 4 : 0. Factoring this equation
3givesp+q-8.
gives (x 2)(x, 2) : 0. Hence, a possible (D) Since p : - 2p, then
2m and k
value of x is 2. k: 4m2(2n) :
3. (13) The roots ofthe equation (x - 5)G + 2) Substituting forp and bin m + p + k: 70
: 0 are the values ofrthat make the equation
a tlue statement: X : 5 oí X : 2. The sum
gives

ofthe roots is 5 + (,2) or 3, and the product


ml2m+4m:70
of the roots is (r) ( 2) or -10. Hence, the
7m: 70
70
sum, 3, exceeds the product, -10, by 3 m:-:10
(- 10) or 13. 7

5. (D) Eliminate 7 by adding corresponding


Lesson 4-6 sides of the two equations:
MULT|PLE_CHolCF
'- ! :3
1. (A) First solve the equation that contains +x-|),:5
one variable. Since 3r * 15 : 0, then ",:4
8
3x: - 15, so
)r| 0:8. so x
2
Sincer: 4andx-,/:3,then 4 - l- l,
3 so;l - 1.

Substituting 5 for r in the other equation, 6. (C) Subtract corresponding sides ofthe two
2x - 37 : 11, gives 2(-5) - 37 = 17 or given equations:
- 10 - 31 : 1 1, Adding 10 to both sides of :19
5x,| 5x + 1:19
the equation makes 37: 21, so
1,
rx.-J7--) -:--r13I
4x+4y:12
Dividing each member of the equation 4r -
41 : 12 by 4 gives x i 7:3.
(B) If 2a: 3b, then 4a: 6b, Substituting
6bíor4ain4a+ b - 2I gives6b + b- 2I 7. (E) Subtract corresponding sides of the n,c
or7b-21,so given equations:
x+ : 51 3 : x .| 3 57
b:--3 21
1x-9:Z:l) -->+,x*9: 2!
7 0+12:3!
t2
--
a - loll
Since 7 : 4 and x *J , 5y. rhen x - 3 =

5(4) - 20, so
x- 20 - 3 - 17
AnsWerS to Chapter 4 Tune-tJp Exerclses í75
8.
1l] '':T'"":. 7 by adding corresponding 13. (C) If (a 1 b)' : 9, r[rn
ol the given
sldes equations:
111 a2t2ab+ br=9
If (a - l)7z: 49, then
X' !- 4 a2 -
114 2ab-| br_49
+--!--:
'x' Add corresponding sides ofthe two equations:
y- 4
24 a2 + 2ab -t b. = 9
-*0:-=t
X-4, + a2 *2ab+ b2 = 49
)
2a'+0 +2b'=58
Jlnce
- : 1, then .r : 2.
Dividing each member of 2a2 + 2b2 :
9. (Á) In the eq u^tion j:
!,..o.s--ultiplyi.,g
by 2 gives
58

gives 5a : 2b. Since 54 + 3b: 35 and a2-tb2:29


5a:2b,then l4, (D) Proceed as follows:
2b + 3s: . Find the value ofj. atj :
5b:
35
35
rr If 3, then

b:7 ir):Jorl,j=J
rrr
Since 5a = 2b: 2(7) : 14, J
I4 so -:2.
a:- r
-5
(B) Ifa=óand .
_0. ,- *=a(,.tnen 36 Find rhe ualue of 1. ' ' = 5. rhen
) -:6,
,_L _ ]orl +
'' _: r
50/ = b. Slnceav : 4 and = 6,
fI

7 r-i- 7-5so,,_,4,
6
. Multiply corresponding sides ofthe equations
-:l
l.D '
s],
,_-zana,a:4:
4w:6 lr
6] sr
D: -:1
42 -X':zx4
J
1. aaa corresponding sides of the given ;=8
!B)
equatlons: 15. (B) EIiminare7 by subrracring corresponding
4r+ /s: 23 sIdes of the given equations:
-| r- 2s:17
3x* 7:,
5r+ j5:49
Dividing each member of 5 r l 5s : 40
- X+ !: b
5 gives r * ., : 8. Since rl s
by 2x-| 0: c- b

3r í j5: 3(8) : 24
= 8. then
c- b
soX=-2-'
Q) If l--! : 3 and r? - rq : t2, then
?-q:2(3):6
and
r(?*q):12
so r(6) = 12 or 6r: 12. Hence,

,:!:, (-,
176 AlgebraiC Methods

I6. (E) If a+ b: Ilanda- b:7, then ,


adding colresponding sides of the two
rf
::!,th,nZ r: 5a, So

equations gives 2a : 18, so c- za


5

n- 18z -9 . Hence ,

If a+ b:11 anda:9,then9 i á:11,so 7 5 1)


b:11 -9:2
3b-r ío')r--ra=6a
Hence,
ab: (9)(2) : 18
GRlD-|N
1. (3) Since
17. (D) For the given system of three 5 sips -1- 4 gulps : 1 glass
equations, and
l3 sips i 7 gulps : 2 glasses
x4 7:3 then

'- !:6
add the equations two at time to eliminate
l glass

the variables 7 and z


a
13 sips i 7 gulps : 2(5 sips + 4 gulps)
. Elimirrate y by adding corresponding
: 10 sips + 8 gulps

sides of the second and third equations. 13-10sips:8-7gulps


The result ís x l z: 9. 3sips:lgulp
. Eliminate zby adding x l z:9tothe 2. (1.5) \7rite one eq].ration rurderneath the
first equation. The result is 2x : 16. other, aligning like terms in the same vertical
I6 column. Elimirrate á by multiplying the first
Hence. x- - 8.
eqr-ration by 3 and then adding the result to the
2
second equation:
l8- (C) If a :
4c, c : re, and a : !a, therr
3X
substituting the second equation into the -+
first equation gives a 4re. Since : 4re: a: 2a -9- b 6a- 27-3b
5e and e * 0, the coefficients of e must be 4a - 3b- 12 -1,2 + 3b
IOa : |5 +0
equal, so 4r' and r- Z-
5
l5
' I.5
l). (A) Divide corresponding sides of the given
equations:
]. (1.45) If p :
the cost of a pen and z : t-
r8 §l cost of a notebook then
3_

t 2x
-)
r:5x !3 lp )n 3.50
1- )p * 4n: - t,
5
2? + 3n: 5.55 - 2? + 3n: 5.<'

3
,/.
20. (D) ' If " 9,rhen bb:-a,so The charge for one notebook is $1.45.
b7
7
3b: 1a
Answers ta Chapter 4 Tune IJp Exerclses .l77
LEssoN 4-7 .
,.4ULTlPLE-CHolcE III. Since / > 0, dividing both sides
ol a- < b by ú gives . nE'
l. (A) Solution 1: Since 4 + 3? < ? -F l, then
,o \tJlement
'.
3P-p<1-4 of 2p<-3 IlI is always true.
Only Roman numeral statements 1l
< -] and III
so .P --, Hence, the largest integer value are always true.
forp is -2. 5. (B) Sohtion ]:If 4 - 3x<
9olution 2: Plug each of the *7-7
1l, then -3r{
answer choices .50 x > - -. Since -;
lof 1 lnto 4 -r, 3p < p J l unrilyou Ja i, berwcen - 2 and
find one
thar makes rhe inequallry J J.^rhe smallest inrcger value of
true statemenl. x rhat
5tnce choice (A) gives sattrhes this inequaliry is -2.
J ( -2) .' I- h L 5o/urion 2: PILrg each oI rhe answer
choices For
x
I
there no need to conrinuc. ,rartin8 with tAt. into + -
15
Jx < l] unril
you find one that makes the inequaliry
]. (El Solurion _/: Solve - 3< 2x | 5 <9bv statement. Choice (§ gives
a true
nrst. subrracring 5 from each member. Th. 4-3(-3)<11,
result is - 8 1 2x { 4. Now divide
me mber
each 13<1l
of rhis inequaliryby 2, obraining - which is not a tfue statement.
4
< x_< 2. Examjne each ofthe Choice (B) gives
answer choices
until you find one (E) that is
and 2. Since x is less than 2.
not between -4 4-3(-2)<11
2 is nor a possible 10 < 11
value ofr which is true, so there is no need
Solution 2: PIug ea..h of the to continue.
lor xInro -J<2xr
answer choices 6. (B) Determine whether each
Roman numera]
5<9until you Ánd
one (E) that does not make
the i*qr"ii|'"
sta_tement is always ttue when
. a} b} c} O.
true statement.
l. Since a} b, then a - c> b- c, and
ú- arepresents a negative number. Hetlcc,
3, of a number,
t-l hen inequaliry a - c )>
bt_rrh sides o1rrhe
l|!|r*..tncreased
""rn x an<J rhe original
/ -,rare divided by b a,a true inequality
number by 5, 5, is greater -
]l.rhenx- (x' 5] >"'ll. so 2r.l 5 >ihan results only ifthe direction oíthe
in.qualiti
|l.
l.nen 2x ) 6. so x > 3. The is reversed. Thus,
only answer
choice that is greater than a- c _ b- c
b- a ' b-,z
3 is (L,).

+, (C) Determine whether each Romal numeral so statement l is not always true.
statement is always true when
. L 0 { a2 1 b. . II. Since b } c alld a > 0, multiplying
l-rom the given ineriualiry. you
know borh sides.of the inequaliry b
,:_*_:' a á. AJrhough a'is posirive, - ;6y'o
or may nor be posirive. If a > 0,
a may results ln rhe rrue inequaiiV
,b , or,,o
rhcn statement II is always true,
a < :. Il a < 0. then dividing . l
each side IIi. Since a ) ., then
1
< -. Since á )
of a2 1 b by a reverses the inequaliry n 0,
sign, multiplying both sides of the inequality
so a > -./, Hence, statement I is not always 1]
by
true. ;- ; ó produces rhe rrue inequaliry
' II. Multiplying both sides of < 61r, oz t bb
>
gives aa 1 a2á, so statement II is always ; ;, r" statement III is not always true.
true. Hence, only Roman numeral statement
II is
always true.
178 Algebraic Methods

GRID-|N
7. (E) Since .
Ji rs. 7|r]
< 3l] 5]. so r <_ 45.
ló) 1, (4) If 2sl 3 1 7, then 27 < 10, so 7 < 5,
Since s: rl4andr145,s<45 1 4or Since
s<49,
8. (E) If a' 1 b2 and a aná b are not 0, then a
/i5>8 and 21 -3<7
üen 7 ) 3 and at the same time Jr { 5. The
and b may be eirher posirive or negarive integer for which dre question asls must be 4.
numbers, Determine whether each Roman 2. (6) Vhen 2 times an integer x is increased
numeral statement must be true. by 5, the result is always greater than 16 and
. 1.1l a ) 0. rhen no 1 bo Since dividing less than 29, so 16 1 2x i 5 < 29.
both sides of an inequality by a negative Subtracting 5 flom each member of this
number reverses the inequaliry sign, if inequaliry gives 11 ( 2x < 24.Then
a { 0, then + < + Hence, statement I 11 2x 24
-<-<
is not always true. 222 -
. Il. Since a' 1 b',their reciprocals haue the so 5] { x< 12. According to this inequality.
opposite size relarionship, ,o + > +. x is lreater than 5j, so the least integer value
Statemenr Il is always true.
u o'
of ris 6.
. III. Since a' < b2, then a2 - b' < 0.
Factoring the left side of this inequality 3. (.76) If2 < 20x - 13 < 3, adding 13 to each
gives (a -1- b)(a - b) ( 0, so statement membel of the combined inequality makes
IIi is always true. 15 < 20x ( 16 orf < x < f , whicl:
Only Roman numeral statements II and III can also be written as 0-75 < l { 0.80. Hence,
must be true. one possible va]ue for x is 0.76. Grid ir
9. (D) If b+3> 0, then á ) -3. Since ,s - / ()_
1 > 2b- 9, then l0 > 2b,so 5 ) áor
/ < 5, Since á is an integer, á may be equal 4. (6) Canceling identical terms on either sir].

to any of these seven integers: 2, 1, 0, 1,


of the given inequaliry, i +* -i + +'
2,3, or 4. i-i+fr-1-.], results in }{r c:
equivalently, n 17, Hence, the greatc.:
10. (C) Determine whether each Roman numeral
possible integer value íor n is 6.
statement is always true when .r7 > 1 and
z10.
. I. If r ) 0, then x ) z However, the
fact that x may be a negative number could
mean that r { 5 so statement I is not
always true.
. II. Multiplying an inequality by a negative
quantity (z { 0) reverses the direction of
the inequality, so (x1)z < (l)zorxyz < z.
Since z may of may not be greater than or
equal to 1, the inequaliry ryz ( 1 may
or may not be true. Hence, statement Ii is
not always true.
. III. Dividing x1 > l by a negative
quantity reverses the direction of the
inequality, .o + < 1. Statement III is
always true.
Only Roman numeral statement III is always
tIue.
l§wK

word problems
W§l
xx §§
l 1,;1.1'.'l,;1 @}
rlE -|E

-.,

"'.:.
his chapter uses algebraic methods and reasoning to help prepare
you to solve
. the different rypes ofword problems thrt m"y o,, th. SAT.
"ip.".

Lessons in This Chapter


Lesson 5-1 Tianslating from English to Algebra
Lesson 5-2 percent problems
Lesson )-_J Some Special Types of\7ord Problems
Lesson 5-4 Ratio and variation
Lesson )-) Rate problems

17s
Translating from
English to Algebra

Overview
Some SAI algebra questions require that you translate an English sentence
into an algebraic sentence.

Associating English Phrases with Mathematical Operations


\íhen translating an English phrase into a mathematical expression, Iook for key
words that tell you which arithmetic operation is needed. For example:

. 'A number n increased b1,3" is translated as n l ).

' "Three les than a number1," is translated as.7 3.

. "Two times the sum of a number x and 3" is translated as 2 (x + 3),


. "Twelve diuiderl b7 a ,/)is rranrlared as
numbel,p" $.
. 'A number 7 exceeds two times a number xby 5" is translated as 7: 2x -t 5.

. "Tl:'e sum of a number xand 3isatleax7" istranslatedasxí3>7.


. "'i7hen 2 is subtracted jion a number z, the difference is at most 5" is
translatedasn-2a5.

Translating an English Sentence into an Equation


To translate an English sentence into an equation, identify key phrases that can :,.
translated directly into mathematical terms. For example:
. Two times a number z increased bv 5 is 17 .
\_/---.!+q/!

2X n i5 :I7
The equation is 2n * J : 17.
. One-half of a number z diminished bv 5 is 17.

+xn-5
!'-----J\.------\/J

_ I/

: tl.
}., -
Th.-.q.,"tio,, S
^
. Five times a number z exceeds 2 times that number bv 21.
_ .É*
'._____,

5n ln +21
The equation is 5n : 2n -l 21,

-\_J,

180
Translating from English to Algebra 181
Representing One Quantity in Terms of Another
Sometimes it is necessary to compare two quantities by representing one quantiry in
terms of another. For example:

Tim weighs 13 pounds more than Sue. If r represents Sue's weight, then Tim's
weightisx* 13.

The number of dimes exceeds 3 times the number of pennies by 2. If x represents


the number ofpennies, then 3Jr + 2 represents the number of dimes.

Bill has 7 fewer dollars than twice the number Kim has. if x represents the
number of dollars Kim has, then the number of dollars Bill has is 2x
- 7,
The sum of two integers is 25. If r represents one of these integers, then 25 x
Iepresents the other integer.

Less, Less Than, and /s Less lhan


TlP
\ common mistake is to think of /ers and less than as interchangeable. Although both
. x lessy means
:mply subtraction, les than reverses the order in which ttre given numÜers are
Xy.
iubtracted: "5 less 3" is translated as 5 - 3, while "5 less than3'' becomes 3
- 5. . y fieans
The phrase is less than reíers to a comparison of two numbers rather than an x less thdn

rrithmetic operation. Thus, "3 is less than 5" is translated as 3 { 5.


means x </.
=XA|VPLE
lf 11 less than 7 times a certain number is 7 more than 4 times the number, what is the
number?

SoIution
\ssume that írepresents the unknown number. The expression "11 less than 7 times
_lcertain number" means 7x decreased by i 1. Also, remember the n:,le that less tban
:everses the order of subtraction, so 1 1 gets subtracted from 7 x as in 7x 1 1. The -
.rpression "7 more than 4 times the number" means 4rr increased by 7, which is
:ranslated as 4r -]- 7. Thus,

7 times the number decreased by 1 l ls 4 times the ntrmber increased by 7

7x -11 - 4x +7
lolve 7y lI : 4x * 7 by collecting like terms on the same side of the equation:

7x,4x:11 1-7
3x:18
3x _18
33
x:6
. he number is 6.
182 Ward Problems

Lesson 5-1 Tune-Up Exercises


Multiple-Choice
1. CarI has 7 fewer than twice the number ol 5. \Mhen 3 is subtracted from 5 times a
course credits that steve has. steve has 5 nrrmber z, the result is 27.
more coulse credits than Gary. If Gary has
Which oF rhe |.llo" ing equations
8 course credits, how many does Carl
fepfesents the statement above?
have?
(ö 5n-3:27
(A) 6 (B) 3n 5 :27
(B) 19 (C) 5(z 3) : 27
(D) 3(n - 5) : 27
(C) 20
(D) 27
(E) 33
(E) 3-5n-27
._ l ,
2. \X,/hich of rhe lollouing expressions 6. lt i ol a number ls 4 le55 lhan l
i ol
represents the phrase "3 less than the number, the number is
2 times .t''?
(A) 12
(A)3-2x (B) 18
2-3x
(B) (C) 24
(C)zr-z (D) 30
(D)2x-3 (E) 36
(E) 2(3 - ,)
7. Su,an weigh. P pound.. If Susan gain,
3. \When 4 times a number z is increased by 17 pounds, she will weigh as much as
9, the result is 21. Carol, who weighs 8 pounds less than
Which ol rhe following eqLrations Judy. IfJudy weighs x pounds, then
Srr.an s weighr. p. in rerm. of x is
represents the statement above?
(A) x- 25
(A) 4(n + 9) : 2l
(B)
(B) 4n:)+2l
(C) "
9

(C) n+4x9:2I x+ 9
(D) 17r 8
(D)4n+9-21
(E) x+ 25
(E) 4+9n-2l
4. If x - 4 is 2 greater than 7 -l- 1, then by 8. When 2 rimes a number n is decl,ease;
how much is r -F 6 greater than 1? by 5, the result is at least 11.
(^) 7 \\,4l ich ol rhe Following expressions
(B) 8 IepIesents the sentence above?
(C) 13
(D) 14
(A)2(n, 5) > rl
(B) 2n-5<11
(E) l5 (C) 2n,5> 11
(D) 2n-5-Il
(E) 2(n - 5) < 11
Translating fram English to Algebra 183
\ 4len 3 times a number z is increased by
7, 13. IFr + 7 is 4 more than x
y, which of
the result is at most 4 times the number the following statements must be true?
decreased by
x:2
1 .
I.
Vhich of rhe following expressions II. 1: 2
repfesents the sentence above? IIL x1 can have more than one value.
(A) 3n1-7<4n-l (A) I only
(B) 3n-|7>4n 1 (B) II only
(C) 3nl7,:4(n-1) (C) III only
(D) 3nl7:-4(n_1) (D) I and III only
(E) 3(n + 7) < 4(n - 1) (E) II and 1II only

10. If r multiplied by the sum of x and 27 is


I4. Jillt present weight is 14 pounds less than
divided by 4n the result is
lrer weight a year ago. If her weight at that
,n, !! f! .9
v l,/
z4
^
- time was * of her present weighr, whar is
her present weight in pounds?
p's4+]!
42 (A) 98
rcl4l+!
4y 2
(B) 104
(C) 112
tot ]t+l (D) 118
l] qJ
(E) j(+ (E) 120
zj
15. A number x is 3 less than 4 times the
1, If 4 less than r is 1 more than 1, what is number 7. Two times the sum of x and 7
x in terms of 7? is 9. Vhich of the following pairs of
(^) l-3
(B) ,y+ 1
equations could be used to find the
values of x and 7?
(C) 1+ 3 (A) x: 4! 3
(D) ]+ 4 2(x+):9
(E) l+5 (B) !-4x-3
2(x+):9
If the number a -
5 is 3 less than á,
(C) x-4(1 -3)
lvhich expression has the same value as 2(x+)-9
1 more than á?
(D) 7-4(x-3)
(A) a-
2xi 1:9
2 (E) X:q|-3
(B) a-
(C) a
1
2Gl:9
(D)a+1
(E) a+2
184 Ward Problems

16. Arthur has 3 times as many marbles as 19. There are 36 identical .".ro,,r,9 of which
Madimir,. If Arthur gives Vladimir must be taped. If e,ery 3 carto*
6 marbles, Arthur will be left with tequire 2 rolls of tape, what is the total
-ü/hat
4 more marbles than Vladimir. is number of rolls of tape that will be
the total number oF marbles that Arthur needed?
and vlaclimir have?
(A) l8
(A) 36 (B) 21
(B) 32 (C) 24
(C) 30 (D) 27
(D) 24 (E) :o
(E) 21

20. After a certain number ol people enter an


17. A video stofe fents each DVD at the rate empty room. -i oI the peopIe who entered
) ll

of x dollars for the first day and 7 dollars the room leave. After 2 more people leave,
for each additional day the DVD is out. ] of rhe original number of people *ho
\When Sara retulns a DVD to this store,
ántered the room remain. \íhat was the
she is charged c dollars. In terms of ,r, 7, original number ofpeople who enteled
and c what is the number of rental days the empry room?
for which Sara is charged?
(A) 9
(A) x+ c7 (B) 12
(B),y' x (C) 18

,^. ,i
(L) y (D) 24
a.- (E) 30
(D) r+7(c 1)

(E),y1+ry 21 ln a high school that has a total of 950


f
.

srudents, the number of senion is ol'


18, A computer plogfam is designed so that, the number oíjuniors, and the number :

when a number is entered, the computer juniors is j of the number of


output is obtained by multiplying the sophomores. If this school has the same
number by 3 and then subtracting 4 from number of freslrmen as sophomores, hc,.,
the product. If the output that results many students are seniors?
lrom entering a number x is then entered,
(A) 120
which expression represents, in terms of r,
(B) 150
the final output?
(C) 180
(A) 3x 8 (D) 200
(B) 3x - 12 (E) 300
(C) 9x B
(D) 9r - t6
(E) 6r+ 9
Translating from Engti§h to Algebra 185
Grid-ln
1. If o1 of x is 20, what number is x decreased
In a certain college class, each student
3.
bv 1?
received a grade ofA, B, C, or D or an
"lncomplere." ln this c|ass, ].ofrhe
,lr. dilference of rwo positive
'. "rr
numbers is 10. If the smaller of the two
o
students received As. j received Bs.
numbers is 3, what is the sum of the two ], received Cs, and
numbers? I received Ds, lf3
students received grades of "Incomplete,''
how many students were in the clais?
percent pióuems

Overview
Simple algebraic equations can be used to help solve different types of percent
problems.

The Three Types oí Percent Problems


You can solve each of the three basic types of percent problems by writing and
solving an equation.

. Type l: Finding a percent ofa given number.

EXAMPLE
What is 15% of 80?
-T, T T T:-
n : 0.'15 X 80
:12
15% of 80 is 12.

. \pe 2z Finding a number when a percent of it is given.

EXAMPLE
30% of What number is 12?

JJ J IJ
0.30 X n :12
o.3on 12:
10(0.3n) = 10(12)
3n : 120
120

:4O
3oo/. ol 40 is 12,

í86
Percent Problems 187
. Type 3: Finding what percent
one number is of another.

E(AMPLE
What percent oí 30 is 9?

l JJ tJ
L X30=9
100
p
100
x30:9

LDí =9
10
10.9
9 is 30% of 30.
=30
188 Ward Problems

Lesson 5-2 Tune-Up Exercises


Multiple-Choice
1. The sum of 25o/o of 32 and 40o/o of 15is 6. A soccer team has p|ayed 25 games and
what percent of 35? has won 607o of the games it has played.
-what is the minimum number of
(A) 20vo
addirional games the team must win in
(B) 25o/o
order to finish the season winning 80% of
(C) 30%
the games it has played?
(D) 4ook
(E) 50% (A) 28
(B) 25
(C) 2l
2. 30o/o of 150 equals 4.5o/o of
(D) 18
(A) 10 (E) 15
(B) 100
(C) 250
(D) 1,000 7. \7hat percent of 800 is 5?

(E) 10,000 (A)


1

l60%

3. lf Ais 725o/o of B, then B is what percent (B) +qo


ó
of A? (C) 1.60/o
(A) 60% (D) 62.50/o
(B) 7syo
(C) 80%
(E),)!9%
(D) 88%
(E) 900/o
8. ]2,)
i. *h", o....n, of J?
(A) 250o/o
4. By the end ofthe school year, Térry had
(B) 210%
passed 80% of his science tests. IfTérry
lailed 4 science tests. how mJny \cience
(C) I40o/o
(D) 40o/o
tests did Térry pass?
(E) 20o/o
(A) l2
(B) 15
q In an opinion poll of 50 men and
(C) l6
(D) 18 40 women, 70o/o of the men and 2ít :

(E) 20 of the women said that they preferrej


fiction to nonfiction books. 'ü4rat pe::
of the number ofpeople polled prett:
5. If 30olo of xis 21, what is 80% of x? to read fiction?
(A) 28 (A) 40%o
(B) 35 (B) 45%o
(C) 42 (C) 50%o
(D) 48 (D) 60%
(E) 56 (E) 75"/o
Percent Prablems 18g
10. If 25o/o of xis 12.5, what is 12.5o/o of 2x? 15. If ais 30o/o greater than l
and b is 20o/o
(A) 6.25 greater than B, then ab ís what percent
(B) 12.5 greater than AB?
(C) 25 (A) 25o/o
(D) 37.5 (B) 5oo/o
(E) 50 (C) s6yo
(D) 600/o
] (E) 75yo
11. Tf ol a number is 9. what is 75oo of the
i
same number?
16. A number a increased by 20%o ofa results
(A) 36 in a number /. -When & is decreased by
(B) 48
(C) Tlo/o of b, the result is c. The number
54
c is what percent of a?
(D) 72
(E) 81 (A) 40%o
(B) 60%
(C) 80%
I2. 300o/o of 6 is what percent of 24?
(D) l201/o
(A) 4oo/o (E) 1r0olo
(B) 5oolo
(C) 60%o
(D) 75o/o 17,
VOTING POLL
(E) 80% Candidate A 30olo
Candidate B 50%
l3. The regular price of soltware at a Undecided 20olo
computer supefstore is 120lo offthe retail
price. During an annual sale, the same The table above summarizes the results
software is 25o/o off the regular price. If of an election poll in which 4,000 vot_
rhe rerail price is 2, which expression ers participated, In the actual election,
represents the sale price? all 4,000 of these people voted and those
people who chose a candidate in the poll
(A) 0.34p
voted for that candidate. People who were
(B) 0,37?
undecided voted for candidate A in the
(C) 0.63p
sarne proportion as the people who cast
(D) 0.64p
votes for candidates in the poll. Of the
(E) 0.66p
people polled, how many voted for candi-
date Á in the actual election?
I4. The price ofa stock fal|s 25o/o, By what
(A) 1,420
percent of the new price must the stock (B) 1,500
price rise in order to reach its original
(C) 1,640
value?
(D) 1,680
(A) 25o/o (E) 1,800
(B) 30o/o
(C) 33].o/o
3
(D) 40o/o
(E) 75O/o
190 Ward Problems

Grid-ln
1. A high school tennis team is scheduled to 2. In a club of 35 boys and 28 girls, 80%
play 28 matches. If the team wins 60% of of the boys and 25o/o of the girls have
the first 15 matches, how many additional been members for more than 2 years. If
matches must the team win in ofder to ll percent of the club have been mem-
finish the season winning 75o/o of its bers for more than 2 years, what is the
scheduled matches? value of n?
Son]s Spe_cial |yp es of ,,* §§§}i

woril probléliis
ffi-§

lit ]:::;ttl^i:,1q.b*
problems and age problems.
translation skills to solving consecutive integer

Consecutive lnteger Problems


Consecutive integers differ by 1, and consecutive
even or odd integers differ by 2.
. A list ofconsecutive integers that
begins with x is
x,xlI,x*2,x+3,...
. A list ofconsecutive even or odd integers that
begins with ris
x,xl2,xl4,x+6,...

axAMPLE
ln a set oí 3 consecutive odd integers,
twice the sum of rhe second and the third
is 43 more than 3 times the first integer integers
i
What is thg sm"rr""t otin" int"g"r"z

SoIution

' Represent the unknown quantities in


mathematical terms:
x:
Let the first ofthe three consecutive odd integers.

Then r * 2 : the second ofthe three consecutive odd integers,


and x l 4: the third ofthe three consecutive odd integers.

' by directly translating the key words of the


Y:,:::r:L::ion
ln to mathematlcal telms:
English sentence

Twice the sum of the second and the


third is 43 more than 3 times the first
:
q! q_-J g-_f-
2|(x+2)+(x+4)] 43+ )X

191
192 Word Problems

. Solve the equation:

2|(x+2)+(x+4)]:43+3x
2L2x+6]_43+3x
4x*12:43+3x
4x- 3x- 43,12
x -- 31,

Hence, the smallest oF the three consecutiye odd integers is 31,

Age Problems
Ifxrepresents a person's píesent age, then subtracting a number ofyears from x gives
the person's age in the past. Similarly, adding a number ofyears to r gives the person's
age in the future. For example:

x - 5 represents a person's age 5 years ago.


x -]- 5 represents a persont age 5 years from now.

EXAMPLE
lf 7 years from now Susan Wi|l be 2 times aS old as she Was 3 years ago, What is susan's
present age?

SoIution
:
Let x Susan's present age.

Then x i 7 : Susan's age7 years from now


andx- 3: Susan's age 3 years ago.

7 years from now Susan will be 2 times as old as she was 3 years ago

xt7 2(x - 3)

x+7 :2(x- 3)

Xi / - LX o

13: x
Susan is now 13 years old.
some Special Types of Word Problems 1g3

Lesson 5-3 Tune-Up Exercises


Multiple-Choice
1. In an ordered set of4 consecutive odd 5. If the sum oftwo consecutive integers is
integers, the sum of3 times the second whar is the larger oI rhe.e ruo inregers
integer and the greatest integer is 104.
expressed in terms of É?
§7hich is the least integer in the set?
L
(A) 11
(A)
i- l
(B) 17
G) i+1
1"

(C) 19
(D) 23 (C) 2k- 1
(E) 27
plk-t
2
2. Four years ago Jim was one-half the age
he will be 7 years From now. g1 h+l
2
If r represents Jimt present age, which of
the following equations represents the 6. The greatest of 4 consecutive odd integers
above statement? is 1 1 more than twice the sum of the first

2(x- 4): X+7


t,{] and the second odd integers.

(B) x-4:!u*ll Ifx is the least ofthe 4 consecutive odd


1-
(C)i(J(+41 :*17 inregers. which of the following equation.
repfesents the above statement?
(D)i(Jr+4)=x-7
l
(A) x+4:2lk+ 1)+(x+2)] +11
(B) x+ 8 : 2LQ + 2) + (x+ 4)l + 11
(E) (x- 4) - 2(x+7)
(C) (x+ 6) + 11 : 2|x + (x+ 2)]
(D)x+5:2Lx+(x+1)]+11
3. The sum of z consecutive integers is 11. If (E) x+ 6:2lx+ (x+ 2)] + 1l
the least of these integers is
- i0, -hrt i.
the value of z?
7. If r is the least of 4 consecutive cvcn
(A) 11 integers whose sum ir 5. whar is x in rerms
(B) 12 ot Jl
(C) 20 c
(D) 2i ,4 +-3
íA)
(E) 22
,_,
(D/
3(s + 1)
--__4
+. How old is David if his age 6 years from
now will be twice his age 7 years ago? (C) s 1]
4
(A) 13 lc
(B) 15 (D) -:
)
(C) 17
(D) 20 (E) 3s- 4
(E) 23
194 word problems

8. If John's age is increased by Maryt age, the C)


If 3x years from today Reyna wiil be
result is 2 times Johnt age 3 years ago. If (37 + 4) times her present age, what is
Mary is now Myears old, what is John's Reynat present age in terms of x and jt
pfesent age in terms oí.l1?
3x
(4 M+3 {Al
' 4(1- l\
(B) M+6
(C) 2M-3 ' }i.
(B)
-3] +4
(D) 2M* 6
(E) 2M+ 3
(C) -+,
J-r I
(D)+
+l
(E) Í
4y - I
Ratio and variation i§§ §::§

Émm
ffi*tfr
Overuiew
A ratio is a comparison by division of two quantities that are measured in the
same units. For example, if Mary is 16 years old and her brother Gary is 8 years
old, then Mary is 2 times as old as Gary. The ratio of Maryt age to Garyt age
is 2 ; 1 (read as "2 to 1") since

Maryt age _ 16 vears 2


Gary's age 8 years 1

One quantiry may be related to another quantity so that eithef the ratio or
product of these quantities always remains the same.

Ratio of a to b
The ratio of a to b (b + 0) is the lraction
dl5 to 0 ), $, *trXt, can be written as a : b (read as

EXAMPLE
The ratio of the number oí girls to the number of boys jn a certain class is 3 : 5. lf there is
a total of 32 students in the class, how many girls are in the class?

SoIution
Since the number of girls is a multiple of 3 and the number of boys is the same
multiple of 5, let

3x : the number of girls in the class


and 5 x : the number ofbovs in the class
Then

3x-| jy:32
8x:32

:
,:2:4
1-he number of girls 3x : 3(4) : 12.
8

195
196 Word Problems

Batio of Á to B to C
Ifl : .B represents the ratio of A to B and -B : C represents the ratio oí .B to C then
the ratio ofl to Cis l : C provided that B stands for the same number in both
ratios. For example, iíthe ratio of Ato B is 3: 5 and the ratio of Bto Cis 5:7,
then the ratio of A to Cis 3 :7. In this case B represents the number 5 in both
ratios.

EXA|VPLE
lf the ratio of Á to 8 iS 3 : 5 and the ratio oí B to c is 2 : 7, what is the ratio oí Á to c?

SoIution
Change each ratio into an equivalent ratio in which the term that corresponds to .B

is the same number.

The ratio ofl to ,B is 3 : 5, so the term corresponding to ,B in this ratio is 5. The


Iatio olBto Cis 2 : 7,so the term corresponding to Bin this ratio is 2.

The least common multiple of 5 and 2 is 10. You need to change each ratio into
an equivalent ratio ir-r which the term corresponding to B is 10.

' Multiplying each term of the ratio 3 : 5 by 2 gives the equivalent ratio 6 : 10,
Multiplying each term of the ratio 2 :7 by 5 gives 10 : 35.
. Since the ratio of,4 to _B is equivalent to 6 : 10 and the ratio of B to Cis equir-;-
lent to 10 : 35, the ratio of A to C is 6 : 35.

Direct variation
If two quantities change in value so that their ratio always remains the same, r}:::
one quantity is said to vary directly with the other. \íhen one quantity l,.r:i:,
directly with anothel quantity, a change in one causes a change in the other in ::r
5266 di1661i6n-both increase or both decrease.

EXAMPLE
lf 28 pennies Weigh 42 grams, What is the Weighl in grams of 50 pennies?

SoIuiion
The number oF pennies and their weight vary directly since multiplying one ::
rwo quantities of pennies by a constant causes the other to be multiplied by th<
constant. If x represents the weight in grams of 50 pennies, then

Perrnies _ 28 50
Gra ms 42 x
Cross-multiply: 28x - 42(50)

X: 2100
)a - /)
The weight of 50 pennies is 75 grams.
Ratio and Variation 197
lnverse variation
If two quantities change in opposite directions,
so that their product always
remains the same, then one quantiry is said to r,ary inver§ely
*ith th. oth...
EXAMPLE
Four Workers can build a house in g days.
How many days Would it take 3 Workers to build
the same house?

SoIution
As,the, number of people working on the
house drcreases, the number of days needed
to build the house increases. sincá this is an inverse
*.i"iiorr, ,t. number of workers
times the number of days needed to build the
t.",. r,"y. ..n.,*,.
If /represents the number of days that
MATH
3 workers take to build the house. then REfEREHüf
FAGT
3Xd:4X9
lf rwo quantities,
3d: 36 say x and y, vary

d:4: tz
inversely, then xy =
k where k stands
Three people working together would ,"k. for a fixed number
iZ days to build the house. other than 0.
198 Ward Problems

Lesson 5-4 Tune-Up Exercises


Multiple-Choice
1. A recipe for 4 servings requires salt and 4. At the end of the season, the ratio of the
peppef to be added in the ratio of 2 : 3. number of games a team has won to the
If the recipe is adjusted from 4 to 8 number of games it lost is 4 : 3. If the team
servings, what is the ratio of the salt and won 12 games and each game played ended
pepper that must now be added? in either a win or a loss, how many games
did the team play during the season?
(A) 4 3
(B) 2 6 (A) 9
(C) 2 3 (B) 15
(D) 3 2 (C) 18
(E) 8 4 (D)21
3^ (E) 24
lfsand rare integers.8 ( r< 40.
2, On a certatn maP. ol an lnch represents
Á and
120 miles. How maiy miles does lf inches
5.
l - a, ho* many pos.ible values are
represent? t7
there for l?
(A) 300
(B) 360 (A) Two
(C) 400 (B) Three
(D) 480 (C) Four
(E) 560 (D) Five

3. The population of a bacteria culture (E) Six


doubles in number every 12 minutes, A school club includes only sophomores,
The ratio of the number ofbacteria at the juniors, and seniors, in the ratio ol1 : 3 : 2.
end of 1 hour to the number of bacteria If the club has 42 members, how many
at the beginning ofthat hour is seniors are in the club?
(A) 64 1 (A) 6
(B) 60 1 (B) 7
(Q ez 1 (C) 12
(D) 16 1 (D) 14
(E) 8 1 (E) 21
- a) )
7. II * 4z - |-, whar is rhe ratio of

crc d?
(A) 5 1
(B) 3 2
(C) 4 3
(D) 3 4
(E) 2 3
Ratio and Variation 199
ö. If4 pairs ofsocks costs $10.00, how l2. Tn an ordered list offive consecurive
many pairs of socks can be purchased
positive even integers, the ratio of the
for $22.50?
greatest integer to the least integer is 2 to 1.
(A) \W4rich of the
5 following is the middle
(B) 7 integer in the list?
(C) 8 (A) 10
(D) 9 (B) 12
(E) 10 (C) 14
Two boys can paint a fence in 5 hours. (D) 16
How many hours would it take 3 boys (E) 18
to paint the same fence? 13. If the íaíloof ? to q is 3 : 2, what is the
,o, l ratio of 2p to q?

(B) 3" (A) 1 3


(B) 2 3
(C) 3+
3 (C) 3 3
(D) 3
@) 4: (E)
1

3 4
E) 7+ x:1
14.
z3
]0. A car moving at a constant rate travels
96 miles in 2 hours. If rhe car maintains If in the equation above x and z are inte-
gers. which are possible u"lues oF
this rate, how many miles will the car f?
travel in 5 hoursi I. () 1

(A) 480
(B) 240
x. 1
3
(C) 210 III. 3
(D) 1,92
(A) II only
(E) 144
(B) III only
11. The number of kilograms of corn needed (C) I and III only
to feed 5000 chickens is 30 less than twice (D) II and III only
the number of kilograms needed to feed (E) None
2800 chickens. How many kilograms of
corn are needed to feed 2800 chickens?
l5. If a - 3b: 9b - 7a, then the ratio of
ato bis
(A) 70
(B) 110 (öa 2
(B) 2 3
(C) 140
(C) 3 4
(D) 190
(D) 4 3
(E) 250
(E) 1 2
200 word problems

I6. The ratio of A to B is a : 8, and the ratio 20. At a college basketball game, the ratio of
of B to C is 12 : c. If the fatio ofl to C is the number of freshmen who attended to
2 : 1, what is the ratio of a to c? the number ofjuniors who attended is
(A) 2 3 3 : 4. The ratio of the number of juniors
(B) 3 2 who attended to the number ofseniors
(C) 4 3 who attended is 7 : 6. li7hat is the ratio of
(D) 3 4 the number of freshmen to the number of
(E) 1 3 seniors who attended the basketball game?

17. If 8' : 4', the ratio of r to r is (A) 7 8


(B) 3 4
(A) 2 3 (C) 2 3
(B) 3 2 (D) 1 2
(C) 4 3 (E) 1 3
(D) 3 4
(E) 1 2 21. Given that 1varies inversely as x and
18. If :4 x varies directly as z, if z is doubled,
+
ratio of crc b?
^r4ZÍl: 3, what is the
then 7 is

(A) divided by 4
(ö 2 3 (B) divided by 2
(B) 3 2 (C) multiplied by 2
(C) 2 1
(D) multiplied by 4
(D) 3 1 (E) unchanged
(E) 6 1

19. In a certain college, the ratio of mathe- It took 12 men 5 hours to build an
matics majors to English majors is 3 : 8. If airstrip. \(orking at the same rate, how
in the following school year the number of many additional men could have been
mathematics majors increases 20o/o and the hired in order for the job to have taken
number of English majors decreases 157o, 1 hour less?
whar is the new rario of marhemarics (Á) T-o
majors to English majors? (B) Three
(A) 4:9 (C) Four
(B) 1:2 (D) Six
(C) 9:17 (E) Eight
(D) 17 :32 23. The price per person to rent a limousine
(E) 7:12 for a prom varies inversely with the
number of passengers. If nine people ren:
the limousine, the cost is $70 each. Hor.
many people are renring the limousine
when the cost per cou?le is $105?
(A) 4
(B) 6
(C) 8
(D) l2
(E) 16
Ratio and Varíation 2o.|

Grid-ln
1. A string is cut into 2 pieces that have
4. For integer values ofa and b, ba : 8.The
lengths in the ratio of2 : 9. If the
ratio of a to á is equivalent to the ratio of
difference between the lengths of the 2
r to /. where c and d are inregers. \X,/har is
pieces of string is 42 inches, what is the
rhe value of r when /: l0?
length in inches of the shorter piece?
2. 5. If 6a - 8b: 0 and t : 12b, the ratio of
The ratio of ato bis 5 : 9, and the ratio of
a to r is equivalent to the ratio of 1 to
xto7is 10 : 3. The ratio of ay to bxis
what number?
equivalent ro the ratio of l to whar number?

3. The ratio of dimes to pennies in a purse is


6. Jars A, B, and Ceach contain 8 marbles.
lü/hat is the minimum
3 : 4. If 3 pennies are taken out of number of marbles
that must be transferred among the
the purse. rhe rario of dimes to pennies .iars so
that the ratio of the number of marbles in
becomes ] : l. How many dimes are in
the purse? 1ar A to the number in jar B to the
number in jar C is ), : 2 : 3?
Rate problems
ir§§ § ó§

ffi-§§
Overview
A ratio of two quantities that have different units of measurement is called a
íate. For example, if a car travels a tota.l distance of 150 miles in 3 hours, the
average rate of speed is the distance traveled divided by the amount of time
required to travel that distance:
Distance l50 miles :
Rate - ...::....::]::::.]
Time .} hours
50 miles per hour

Rate problems are usually solved using the general relationship

Rate (of l per unit B) X B : A


You are using the cofíect rate relationship if the units check, as in
Rate X Time : Distance
J J J

H x Hóíő - Miles

unit cost problems


Unit cost problems require you to figure out a rate by calculating the cost per
item. Multiplying this rate by a given number of items gives the total cost of those
items.

EXAMPLE
lí 5 cans of soup cost s1.95, how much do 3 cans oí soup cost?

Solution
-1

Since 5 cans ofsoup cost $1.95, the cost oí 1 can is

$1,95 0..]9 dollar oer can


5 cans

To find the cost of3 cans, multiply the rate of0.39 dollar per can by 3:

Cost of3 cans : 3 cans x 0.39 $ : Sr.rZ

2o2
Rate Problems 2o3
SoIution 2
The number of cans of soup varies directly with
the cost of the cans. Form a
proportion in which .lr represents the cost of cans
3 ofsoup:

Cost __ x_ i.95
Number of cans 3
s, :
5
Eá.gs)
X : 5.85

*:5'!5:t,rl
)

The cost of3 cans ofsoup is $1.1/.

Motion problems
The solution of motion problems depends on
the relationship

RateXTime=Distance

=XAMPLE
John rode his bicycle to town at the rate of ,15
miles per hour. He left the bicycle in town
íor minor repairs and Walked home along the same
route at the rate of 3 miles per hour.
Excluding the time John Spent in taking the
bike into tt. l."pui, trip took 3 hours,
HoW many hours did John take to Walk back? "hop,lhe

Solution
.ince the trip took a toral
of3 hcrurs,
:
let x the number ofhours John took to ride his bicycle to
town,
and 3 - r: the number ofhours John took to walk back irom town.

Rate X Time Distance


To town 15 mph x hours 15r,
Return trip 3 mph 3 - xhours 3(3-'
:_nce John traveled over the same route, the two distances must be equal. Hence,

15x=3(3-x)
15x:9 - 3x
18r - 9
9
x:T-:rhour
,l
.

in took 3 - : Zjhou.. to walk back.


+
2o4 Ward Problems

Lesson 5-5 Tune-Up Exercises


Multiple-Choice
1. Iffour pens cost $1.96, what is the 5. A man drove to work at an average rate of
gleatest number of pens that can be speed of 60 miles per hour and returned
purchased for $7.68? over the same route driving at an avelage
rate of speed of 40 miles per hour. If his
(A) 12
total driving time was 1 hour, what was
(B) 14
the total number of miles in the round
(C) 15
trip?
(D) 16
(E) 17 (A) 12

2. If a car is traveling at a constant rate of (B) 24


45 miles per hour, how many miles does it (C) 30
travel from 10:40,r.1,I. to 1:00 p.u. of the
same day? (D) 48

(A) 165 (E) ,0


(B) 150 6. If x people working together at the same
(C) 120 rate can complete a job in /r hours, what
(D) 105 part ol the same job can one p€rson
(E) 90 working alone complete in Éhours?
3. If , centsJ what is the cost in
Épencils cost (^) +
cents ofp pencils?
171 !}
"-, h
B)4
Xk
L
íB);L" lC);+ h
LL
(C)+
l?p
(D)T
(D) c- kp
E)+
(E) --!
pR ,

7. An electrician can install 5 light fixtures i:


'§í'orking
4. A freight train left a station at 12 noon, 3 hours. at that rate, how long
going north at a rate of 50 miles per hour. will it take the electrician to install 8ligh:
At 1:00 p.Irl. a pássengeí train left the fixtures?
same station, going south at a íate oí
(A) 3Í ho.,.,
60 miles per hour. At what time were the
trains 380 miles apart? {Bt 4}hours
(A) 3:00 l,.rrl.
(B) 4:00 p.ti,t.
(C) ljno,.r.
(C) 4:30 p,v. .4,
(D) 45 nours
(D) 5:00 p.v.
(E) 5:30 l.u. (E) l}ho,,.,
Rate Prablems 2o5
8. A freiglrt train and a passenger train staft
1 1. Carmen went on a trip of 120 miles,
toward each other at the same time from
traveling at an average of rr miles per hour.
two towns that are 500 miles apart. After
Several days later she returned over the
3 hours the trains are still 80 Áiles apart.
same route at a fate that was 5 miles per
If the average rate of speed of the
hour faster than her previous rate. if ihe
passenger train is 20 miles per hour faster
time for the return trip was one-third of
than rhe average fate ofspeed of rhe
an hour less than the time for the
Freight rrain. whar is the of outgoing trip, which equation can be used
speed, in miles per hour, "ue.rg...r.
oí the freight to find the value of x?
train?
(A) 40 /A\ 120 1

(B) 45
X+5 3
(C) 50 (B)1:Jr+5_1
(D) 55 720 120 3
(E)
(C) 120 :
60
9, 1
One machine can seaI J60 packages x+(x+j) 3
per hour. and an older .r.i,in..rn .."l
(D) 120 _ 120
xl5 +]3 l
14_0 packages per hour. How many
MINUTES will the rwo machines x
working together take to seal a total of (E) 120(x + 5) - I20X: :
_)
700 packages?
(A) 48
(B) 72
(C) 84
(D) 90
(E) 108
,0, A moror boar traveling ar l8 miles per
hour rraveled rhe Jength ofa lake in one-
quarter of an hour less time than it took
when traveling at 12 miles per hour. W4rat
was the length in miles of the lake?
(46
(B) 9
(C) 12
(D) 15
(E) 21
206 Word Problems

Grid-ln
1. Fruit for a dessert costs $1.20 a pound, If A printing press produces 4,600 flyers per
5 pounds of fruit are needed to make a hour. At this rare, in how many minutes
dessert that serves 18 people, what is the can the same printing press produce
cost ofthe fruit needed to make enough 920 flyers?
of the same dessert to serve 24 people?
Answers to Chapter 5 Tune-lJp Exercises
2o7

Answers to Chapter 5 Tune-Up Exercises


Lesson 5-1 \i(4ren 3 times a number z is increased by
MULTlPLE_CHolCE \ - )n + }. rhe result is at mosr (=)
4 times the number decreased by (:
l (B) Since Steve has 5 more course 1 4n - 1),
credits Hence,3n -t 7 4n - 1
and Cary has 8 course credits, =
1han lav l 0. (C), The olr x and 27 rs x - 27, and, t
5teve has 5 . 8 or lJ course 11m
crecJirs. Carl mrrlriplied by rhis sum is lrx
has 7 fewer than rwice the number
of
- 27j. 19n.n
this expression is divided by 4n thá
course credits that Steve has, so result is
Carl has
(2 X 13) - 7 or 19 course credits. t(x + 2y) xt + )vt
2. (D) The phrase .'3
less than 2 times l'means 47 47
2.r minus 3 or 2x
- 3. xt
3.
lD) Vn." 4 times a number z (= 4n) is ?4t
increased 6y j t-- +, r- 9).
|-1ence, lhe equarion is 4n l q
the resu|t is 2l.
41
fu
2
= 2|. Xt t
4. (C) If x - 4 is 2 greater than y .]- +
l. then 41
X J- Iy r l) 1 2
2

- y-1- J l 1. (E) If 4 subtracted from r is 1


more than 1
X=]+7 thenx- 4 : l + 1,sor: y.t 1 * 4 or
Sincex * e = (l + 7) + 6 or x i 6 : X = ] + 5_
.5, /.r.li..x+ ói§greaterrhan7b1 iJ. 12. (B) If the number a
(A) \X-lhen J is sl§lrr.,.6 -
5 is 3 less than á, then
frorn S ,i..,
r.number n I= 5n - J]. rhe ,.rul,
a-5:b-3
Hence. the equarion isJn - J
i.27. a - 5 a 3= b

6. (C) Solution 1: Let x represent


= 27. a 2-- b
the unknown Since 1 more than / is b + 1, add,1 to each
number. j of x is 4 less rhan I of side of the eqrration a 2:
a- 2+ l - bl lor a_- 1 : á+ 1. Henci
x. ,,, - ',,Since
j 4. Mulrinl"i".."-l á obt"ini,rg
"; .- .4. Multiplying
ly each meÁber
-_jl^_
OI tn15 equalIon by 6. the LCD lCD of its a - 1h:s the same value as l more than
ir.
denominators, gives 2x :3x 13. (E) If x -f
á.
- 24, so 7 is 4 more than í - then
y'
-x: -24 or x= 24 x +,!: x - ! + 4 or i x,'x
Solution 2 Plug each oí the
answer choices
!:
so 2! : 4. Derermine whether each
1l 1 4,
inro the sutemenr of rhe problem Roman
un,il 1ou numeral statement is true or
httd onelC} that works:
Iol2l = 8; is l less
r . false
I. The va]ue oFr calnot be determined,
.

than; oí 24 (: 12). statement I is í;lse.


so
-, (A), _Since Carol weigh s .
§ p6L] nd5 |ess rhan udy
J
IL Statement II is true since 2;, : 4, so
and Judy.weighs x pounds. Carol
*.;gh. * - á
pourlds. lfSuran, who weighs p
pound.. gain, .iII. Sin.. the value of x is not íixed, r7 can
l
,
she will weigh
.P,ou,'lOs. r I- poúdr.
/ have more than one value for differen,,i.l,,.,
whlch ls a, much as Carol weighs. Hgncg. ofx. Hence. starement lII is rrue.
P' 1--- x- 8.so p= x- 25. Only Roman numetal statements tl and IIT
8. tC) ] he expression ..at ]easr.. in the are tfue.
given
senlence. means greater than l4. (C] Jill'§ weighr
or equal to. l yfáf ago was ] of her
ll rvr o times a number z is decreased by presenl wei3ht. so Jill losr j of heiweighr.
? !:
2" - 5), the result is at least (>) 1 1'. Since, her presenr weighr ir']l poundri.r.
Hence,2n - 5 > 11. rhan her weighr a year a8o. Irt pounrJs
9. (ö The expression 'bt most'' rep-
in the givell resents l of her present weight. Hence,
sentence means less tharr or 1ili's
.q.,ri ,o. present weight is 8 X 14 or 112 pounds.
2o8 Word Problems

15. (A) If the number ris 3 less than 4 times the IrZ - 9x2:18 rolls of tape will be
number 1 then x : 47 - 3, If 2 times the ,í..d.d.
sum oFrandlis 9, then 2(x + 7) - 9. 20. (D) lf
the original number of people in
The mo equations in choice (A) can be used the loom is represented by x, then, after
to find the values of x and 7. I ol rhe x people in rhe room leaue. {x
16. (B) Since Arthur has 3 times as many maf- people remain, Afier rwo more peop]e leá\ e.
bles as Vladimir, let x represent the number .x - 2 people remain. Since this number rep-
]
of marbles Madimir has, and let 3r represent ierents ofrh. who originally enrered
"people
the number of marbles Arthur has. If Arthur the room, I,, 2: }x. Multiplying each
gives Vladimir 6 marbles, Arthur now has 3r member o|this equation by 12 gives 4x - 24
- 6 marbles and Vladimir has x * 6 mar- :3x,so4x 3x- 24andx- 24.
bles. since Arthur is left with 4 more marbles 21. (B) Let í fepfesent the number of sopho-
than vladimir, mores. Then jr..p.....rt. the number of
3x_6:(x+6)+4 juniors, and } t]r1 jx represents the
:r-]-10 ",
number of seniors. since this school has
2x: 16 the same number of freshmen as sophomores,
í:8 the number of lreshmen is x. The total number
Since :r : 8, 3x : 24. The
total number of ofstudents is 950 so
marbles that Arthur and vladimir have is
x+ 3X:8 _f 24or32. x * )1x i -x :
-l- x 950
17. (E) Let lz fepfesent the number of rental 32
days. Since the video store charges x dollars 21
2xl-x+-r:950
for the first day and 7 dollars for each addi, 32
tional day that the DVD is out, for z days
6()x\ ,t 6I x -l 6lítxl) :
l) \
the charge is lx i (z - 1)7. Since Sara is
6(950)
[;- ) \2 )
charged rdollars, r : x -| (n - 1)7. Now 12x 1 4x -| 3x : 5700
solve this equation for z by multiplyir-rg each 19x -- 5700
term inside the parentheses by 1;
x _ 5700
c: xlny - j 19
]L ryl
: 300
yCí ql
!t! ! Since jx: j laOO; - 150, 150 students are
c- x senlofs.
1+_- n
1
GRID-|N
18. (D) If :r is the number entered, then mul- i. (l5) If ; of xis 20, then fx - 20, so
tiplying r by 3 gives 3;r and subtracting
x-=(20)=
/1
l6
4 from that product gives 3x - 4. Since )
3x - 4 is then entered, the final output Hence, r decreased by 1 is x - 1 or 15.
is obtained by multiplying 3x - 4 6y 3
2. (26) If half the difference of two positi,. .
and then subtracting 4. Thus, the final out-
numbers is 10, then the difference of ri.
put is 3(3x - 4) 4; which simplifies to
two positive numbers is 20. If the smaller ::
9x, 12 - 4 or 9x 16,
the rwo numbers is 3, then the other poi ,
19. (A) Since l le : }x 16 : 3x 9 : 27, tive number must be 23 since 23 - 3 : ) .
"r
27 of the 36 identical cartons must be taped. Hence, the sum of the two numbe rs -,
If every 3 cartons require 2 rolls of tape, then 3 ,f 23 or 26.
Answers to Chapter 5 Tune,lJp Exerclses 209
3. (36) You are given that in a certain college
,: or ofl, X
: 56,S
Since 7, x, is 8
class each student received a grade ofA, B, Ö, + 8Oo/o 7

or $ oF the ,,.,-b.. of ,,.,1í" í ,h: .i"* d !!'#:1ii:§ilr:T:,'i:,T:".3T "j.|;


received le rter grades.
I
of the students in ,h" Bo, Ú g"-.r.
class received grades oi-'.I nco- p|ete.'. You are
1,If x represents the minimum num_
told that 3 ,tá.rrt, -*i".a ,r,i, *.ra.. s... loluti9n
ber oF addrt ional games the rea m musr win in
t hese J studen s lepresen r ;. ;il#;l;;,
r
;h;;
-'
were J x 12 or 3i r,ud.n#,n ,i;d; order to finish the season winning 80o/o of rhe
games it has played, then 15 -l 80% of Jr:
Lesson 5_2 t -
Q5 + x), so 15 y 0.80(25 -F x). Remove
the parentheses by multiplying
.- each term
IVULTIPLE-CHO|CE inside the parentheses by o.-ao:
l. (D) First find the sum of257o oF32 and,40o/o 15íx:20i0.8x
,. 1 oF l5: x - 0.8x: 20 - 15
\:_! 25oó ol 3) _- j x .1z = s 0,2x - 5
4
r 40oooft5:!-,,|5:6 .: L-r. z,\
x- oj =
...... -.....-...... .......
- --rn Solution 2: For each answer choice, form and
then evaluate the lracrion
Then answer the question ''14 is what per-
'§7ins Answer Choice f 1 5
of J5?" by solving
cent the equation Total games Answer Choice 25 f
14:# XJ5. where p is the unknown The correct answer choice is (B), which gives a
'1,4
percent. Since = 0.35p, fraction equal to 0.80:
1l
P=or=eo 15 25+15 40
=n art
so 14 ts 40O/o of 35. 25 25+25 50
0.80

7. (B) Answer the question "\ü/hat percent of800


2. (D) 3go7o of 150 : 0.30 X 150 : 45. Answer
is,5?" by.solving the equation X SOO i, fr :
; i ., solving
the question "4.5olo ofwhat number is 45?'' by
where p is the unknown p...eni. Si.r..
,, rhe equation 0.045 X z = 45. Since
é 0,045n: 45, D
X 800 Pu 8 -
i0 5

n: 0*- = ]000 then p : f%.


j. (C) If Ais 125o/o oi a th.., A : 1,25 x B. 8. (A) Answer the question is what percent" j
Since A: 1.258, or+?" by solving the equadon; :#
B:+íxA-o.80xA whére
*,
is the unknown fr.r...iii Sirr'..
7 '
j: #, th."
so B is 800/o of l.
i. (C) Since Térry passed 80% ofhis science tests,
?: +:25oo/o
he lailed 20oo or
9. (C) In the opinion poll, 70o/o of the 50 men
]ofrhe rests raken. Térry lailed
4 science lests. 50 4 represents ] ol rhe toral or 0.70 X 50 : 35 men preferred fiction to
number oIscience tests. Hence. trry rook 5 x
nonfiction bools. In the same pol|,25o/o of
the 40 women or 0,25 X 40 - 10 women
4 or 20 science tests. Since he failej 4 tests. he
preferred fiction to nonfiction books. Thus, 45
passed 20 - 4 or 16 science rests.
(: 35 + 10) ol the 90 (: :o + 40) people
<, íE) If JOoo ofxis 2l. rhen = 2l, polled preferred fiction to nonfiction toola.
|o x so
!--1/)l\_"
l0 " = t lzll
: 7 Since
S : -j : SOoo, 50qo oFrhe people polled
preterred to read 6ction.
21o Word Problems

I0. (B) I2,5|o of 2x : 12.5o/o X 2x Since


: (I2.5o/o X 2) X x 2,1.)d:o,8t)n
- 25o/o Xx '":Zh.
3" 3 ",'"'
Hence, 12.5o/o of 2x - 25o/o of x : I2,5, then r is 80o/o of a.
-Io
ll. (C) Ií-+ of a number is O. then or the whole
$
17. (B) find the number of people who voted
\g nu-b.? is 8 \ 9 or -2, Henc!. 75oo of the for candidate A in the actual election, add the
EL same number is 0.75 X 72 : 54. number of people who chose candidate A in
the poll to the number ofpeople who voted for
X 6 or 18. Now answer
12. (D) 3ggo7o of 6 is 3.00
candidate A but were undecided in the poll.
ffi rhe quesrion "]8 is whar percent of 24i"
B . 4,000 : ],,200,
:0.30 X
by .olving rhe equarion l8 : |Í0 Y 24, Since 30% of 4,000
: 1,200 people yoted for candidate A in the
where p is the unknown percent. Hence, 18
0.24p, so
poll as well as in the actual election.
. Since 20% of4,000 : 0.20 X 4,000 = 800,
18
?: 0ü:75o/, 800 people were rrndecided in üe poll.
13, (E) Since the regular price of software is 12olo
. It is given that in the actual election, the
undecided people voted for candidate A in
kÁ oFf rhe rerail price p. the regular price of the
the same plopoltion as the people who cast
\-"§ so|iware can be expressed as 0.88p. During
an annual sale, the same software is 25o/o off votes for candidates in the poll. Hence, of
the regular price, so the sale pfice is the 800 undecided people, -,!9% 1 366
0.75 X 0-88p: 0_66p. : x SOO : 300 voted for candidate A in
f
14. (C) Suppose the original price ofthe stock was the actual election.
$100. If the price falls 25o/o, the new price of
. Hence, a total of 1,200 + 300:1,500people
the stock is 0.75 X $100 : $75. To reach its voted for candidate A in the actual election.
original value, the price oF the stock must rise
$25, which is { or 33{% ofits $75 price. GRID-|N
15. (C) If a is 30o/o greater than ,4, then
1. (l2) Since the teám wins 60% ofthe first lj
d:A+30o/oofA matches, it wins 0.60 X 15 or 9 of its first
: A+ 0.30A 15 matches. Let x represent the number oi
: I.3A additional matches it must win to finish the
28-match season winning 75% of its sched-
Similarly, if b is 20O/o greatef than 8, then
b: I.2B.'íhus, uled matches. Since 5o/o :
7
1 ,

ab: (1,3A)(I.2B) Total wins 9] x 3


: I.56(Aö Total games 28 4
: AB + 0,56A8 Solve the equation by cross-multiplying:
Since 0.56 : 560/o, ab is 56%o greater than 4(9+x):3(28)
AB- 36 + 4x- 84

16. (C) If a increased by 20o/o of n results in á,


4x- 48
48
then ,:í
a -t (20o/o of a) : 5
a1 0.2a: b
l. ()).)) slnce
1.2a - b R . J5
R\eJ 80qo ofjs _ 0.80 28
§7hen / is decreased 6y 33!o/o of á the result
rs ,, so and
, l, 2,
b-jO:jo: r 25o/o oF 28 : 0.25 x 28 : 7
Answers to Chapter 5 Tune-lJp Exercises 211
the1 3J(:28+7)of the63'35+ 28)boys Since the question asks for the larger of
and girls have been club memberc for
mo.. th- the two integers, you need to find r-+ 1 in
2 years. Since,]i : O.:SS: . . ., 55.59o of rhe terms of É:
club have been*members for more than 2
vears.
h_1
x+l=-+1
Lesson 5-3 2

MULT|PLE_CHolCE
h_l 2
22
l (Dl llx is an odd inreger, then x L 2, x l 4,
_k +l
and x + 6 lepresent the next three consecu-
2
tive odd .integers. Since the sum of 3 times
the 6. (E) If x is the least of the 4 consecutiye odcl
second integer, x l 2, and the largest integer,
x -| 6, ís l04, integers, then the next 3 consecutive odd inte-
gers are í -f 2, x i
4, and x -| 6. The greatest
3(x+2)+(x+6)=10! of these 4 integers is x -| 6, and 2 times Áe sum
3x,| 6)- x+ 6:104 of the first and the second of these integers is
4x+12:104 2|x + (x + 2)]. Hence, the required equatlon is
4x:92 x-| 6:2|x1- (x-| 2) + 11
x-.f- c),
z3 7. (A) If x is the least of 4 consecutive even inte-
gers whose sum is § then
2. (B) If x is Jimt present ageJ then x
represents Jim's age 4 years ago and x l
- 4 x*(x* 2)+(x+ 4)+(rí+6) :S
represenrs his age 7 years from now.
7 4x+12:S
4 years agoJim was one-halFthe age he
Since
4x:S 12
wilI be
7 years from now,
s _ó 5
.§_12:i--T=i-3
, 1. t-,)
X- 4:;(x "- 4 :___::-

3. (E) If the sum of z .onr.-.uriu. integers


start-
ing with - 10 is 11, then the consecutive
inte-
gers_must range from
-10 to -I- l1, inclusive, 8. (B) Let ,/ represent John's age. If John's
so that number-pair opposites from age is increased by Mary's age, the result
-10 to
* 10 cancel each other. Including 0, there are is J + M, which equals 2 times
John's age
22 integers in this interval. 3 years ago or 2(J - 3). Thus, + M':
]
4, (D) If x is David's present age, then
x -l- 6 is )(J- lloíJ+ M:2J _6,so
his age 6 years from now and x
7 years ago. Since 6 years From
- 7 was his age M+6:2J-J and J:M+6
now David wll]
be rwice his age 7 years ago, 9. (C) Suppose R represents Reyna's present
x-t6=2(x-7) age. In 3x years, Reyna will be R + 3r, which
:2x-14 "-quals
(37 + 4) times her present age R. Hence,
X=20 R+ 3x: (3l + 4)R Solve this equation for R
i. (E) ll rhe two con5ecutive inregers
represented by x and x * 1, then
are R+ 3jr:37R+ 4R
3x= 3lR+ 4R- R
x+ (x+ 1): rt = 3!R+ 3R
)v+ 1 : L : R(3)(/ + 1)
2x- h- l 3x _D
x: k- 1 3( y +1)
Z X _D
y+1
212 Ward Problems

Lesson 5-4 x,|3xi2x:42


|VULTlPLE-CHolCE 6x:42
1. (C) \X4-ren the recipe is adjusted from 4 to 8
,_4
^6
servings, the amounts ofsalt and p epper are eac/l The number of seniors in the club is
doubled, so the ratio of 2 : 3 remains the same. 2x: 2(7) : 14.
2. (E) Solution 1: If r represents the unknown 7, (N If rP : !, then
number of miles, then 2(, 3d):4d
2t - 6d: 4d
Inches_Í:É 2c: l0d
Miles l20 x Since j : 5, the ratio of cto dís 5:I.
8. (D) ]f í represents the number of pairs of
Cross-multiplying gives
socks that can be purchased for $22.50, then
f,x. ljtl)0\ : - )l0 ,
ö.1
';(l2O)
+ _ 4:
Pairs ofsocks
Then Cost l0 22,50
, . 9tz tot 5b0 miles
So
3, l)x: 4(22,50) - 90
Solurion 2:.Since f ol an inch represents qn
y: _:: :
l 20 miles. j- of an inch re pre\enl5 1, l I20 "l0 O

40 miles. Since lJ : l j inches represent 9. (C) The number of boys working and the time
v 40 #.
14 or 560 miles. needed to complete the job are inversely relat-
3. (C) Let 1 replesent the initial population ol ed as one of these quantiries increase.,
"ince.
bacteria. After 12 minutes the population is the other decreases, Let x represent the time
2p, after 24 minutes it is 4p, aftet 36 minutes three boys mke to paint a fence; then
it is 82, after 48 minutes it is 16p, and after 60 3x- 2(J):10
minutes or 1 hour the population is 32p. Since
32P 32
,- 9::]
)J
p1 10. (B) If x represents the number of miles the c:_-
the ratio of the number olbacteria at the end of travels in 5 hours, then
1 hour to the number of bacteria at the begin- Distance 96x
ning ofthat hour is 32 : 1.
Time 2 5
4. (D) IFx represents the number oflosses, then so
'§7ins
_ 4_12 2x- 5(96):48o
Losses 3 x 'i=:
4x: 36 X' 480 Zqj
x:9 1 1. (C) If r repre sents-the r-rumber of kilogr::
The total number of games played was of corn needed to feed 2,800 chickens, i:.:
12 -t 9 or 21. 2x - 30 is the number of kilograms neede : :
feed 5,000 chickens. Ifthe amount ofFeed n.=:*
5. (C) If ; : }, thrn, =
f Si.,.. s is an inte_ for each chicken is assumed to be constant.
x _2x-30
8er, / must be divisible by 7. In the interval
8 < , < 40, there are four integers that are 2,800 5,000
so
divisible by 7: 14,21,28, and 35,
5,000r : 2,800(2r - 30)
6. (D) If x represents the numbel of sophomores
in the school club, then 3x represents the num-
50x:
:
28(2x -
30)
56x -
840
ber ofjuniors, and 2x represents the number of
6x: 840
seniors. Since the club has 42 members,
840
X:
6 - I4U
Answers to Chapter 5 Tune-IJp ExerOises 213
i], (B) If x is rhe leasr integer in an ordered
ftve consecutive po5ilive even integerr,
lisr of 17. (A) IF 8' = 4', then 23' :
22' so 3r : 2t_
then rhe Dividing both sides of this equation by 3r
other integers are x 2, x i )
4, x -| 6, and x ]- 8. grves
Since the rario ofthe grearesI integer to
tnteger ts Z: 1,
the least 312tr o' 2
3r- 3, ;:
! 3
x+ 8 _ The ratio of r to tís 2 : 3.
x1 18. (B) Rewrite each fraction by dividing each
2x: xf 8
term of the numefator by the denomina"tor:
].:8 o-b tt_l tl_.
The middle integer in the list is x -t 4
8+4:12.
: b b b |-1,b-)
b =_n*h_o
--L
c - v -f I : J, - =
^

]3. (D) Iftheratioof ptoqis3:2,thent: j,so ( C 2

c1
ora:'
z(ll
\q,- \z/ ^nd 22
j_
zr-!\
q =
Multiply corresponding sides of the two pro-
I

Hence. the raúo of Zp ro


4ls 3 : l.
:4. (D) Since j :
portlons:
}, then a,, t
x'_ i,;:Jn, ^.-
1

7 - "(z]:,$) lX\ lir


!=3
Determine whether each Roman numeral
value is b2
a possible va|ue of
{' *hen The ratio of to b is 3 : 2,
. |[ - z are in regers. c
*.í l) :
"and i9. (C) Since the ratio of mathematics majors to
j.. rhen x = ,.|. Sin..
,{
mu5t be/an integer. is no, " English majors is given as 3 : 8, supposá there
o|
"'possible are.30 mathematics majors and 8Ó English
Value oi
a.
. € : r(J : il then x: 1. Hence, } is
II.If ma,ors.
. . If the number of mathematics majors
a Possrble válue of- +.
--)
increases 20o/o, then the new number of
' III. If { : J.(,r) : 3, then x : 9. Hence, 3 is mathematics majors is 30 + (20o/o X 30)
a possible value offl. :30+6:36,
Only Roman numeral values Il and IlI are
. If the number of English majors decreases
possible values of
f. I50o. rhen the new number of English ma jors
is 80 - x :
80 _ 12:ó8.
,5. (A) If a - 3b: 9b - 7a, then
115oo 80)
. The new ratio of mathematics majors to
a+7a:9b+3b
: 12b 8a English majors is
ffi = } or 9 : 17.
a_12_3 20. (A) The ratio of freshme n to iuniors who
b- 8 - attended the game is 3 : 4, anÍ, the ratio oF
Hence, the ratio of a to b is 3 : 2. juniors to seniors is 7: 6. Since the least
.6. (C)
' mon multiple of 4 and 7 is 28, multiply the
com-
Since the lowest common muldple of 8 and
12 is 24, multiply each term ofthe ratio
a: 8 by terms of the first ratio by 7 and multiply the
3 and multiply each term of the ratio 12, cby terms of lhe second ratio by 4. Since 2l : 28 i:
2. The ratio of A to B is ú.rcn 3a: 24, and, the then rhe ratio oF lreshmen ro juniorl and 28 :
ratio oF8to Cís 2rl: 2c, so rhe rario oll ro Cis 24 is the ratio ofjuniors to seniofs, 21 : 24 is the
3a : 2r. Since you are given üat d-re rat io of ,4 ro ratio of freshmen to seniors.
Cis 2: 1. rhen Since
zJa
T-r,.o ,, -,,, _ )l 7
248
a
,
_2Q) :! the correct choice is (A).
3(1) 3
Hence, the ratio of ,z to c ls 4 : 3.
214 Word Problems

21. (B) It is given that x varies directly as z. If so :


3. Hence, the number of dimes in the
.lr
z is doubled, then x is also doubled. since purse is 3r: 3(3) : 9.
7 varies inversely as x, their product does not 4. (15) Since23 : 2 X 2X 2 : 8, then a - 3 and
change. Hence, since x is doubled or multi- b : 2, so i : }. Vol are told that f, : j,
plied by 2, then 7 must be divided by 2. Substituting } for and 10 for / gives
f j : f.
22. (B) The number of men working and Since 10 is 5 times 2, rmust be 5 times 3 or 15.
the hours needed for the men to complete 5, (9) To find the value of the ratio of f, multi-
the job are inversely related, If x represents the p|y !.If 6a 8b: 0, then 6a 8b, so
tby - :
number of men needed to complete the job
in 5 - 1 : 4 hours, then 4x : 5 . 12 - 60.
!:8:4
60
b63 l
Hence.x ;:15. This means rhat Since r = 12b,then#- u,.o
15 - 12 : 3 additional men are needed.
_a.b:4. 1:1
]

23. (D) If í repfesents the number of people {1

renting the limousine when the cost per cbc3t*9


couple is .$]05. rhen !E ,.pr.r.nt, ,h.
3

Hence, the íatio of a to r is equivalent to the


ratio of 1 to 9.
cost per person. Hence. x|,,o, : OtSZOt - 6, (4) Since 1ars A, B, and C each contain
: .J
8 marbles, there are 24 marbles in the three
$630, so $105x - $630 X 2: $1,260 and jars. Let x, 2x, and 3x represent the new num-
$l260
bers of marbles in jars A, B, and C, respectively,
$105
Hence, x -l 2x -| Jy: 24 or 6x: 24, so
GFlD-|N
6
,-4-1
1. (12) Since the lengths of the rwo pieces of To achieve a fatio of 1 :2:3, 1ars A, B, and C
string are in the ratio 2 : 9,Iet 2x and 9x rep- must contain 4, 8, and 12 marbles, respectivelf.
fesent theiI lengths. Hence: Since jar ,B already contains 8 marbles, 4 of th.
9x - 2x: 42 8 marbles originally in jar l must be transferred
7x: 42 to jar C,
42
*: (l
7 Lesson 5-5
Since 2:r : 2(6) : 12, the length of the
shorter piece of string is 12 inches. MULT|PLE-CHolCE
2. (6) Since the ratio of ato b is 5 : 9 and the
(C) If 4 pens cost $1.96, the cost of 1 pen ,,
ratioofxtoyis l0:3.f, ; r"di- +, 1.
$}n6 : $O.al. The greatest number of pt:_.
sol-fr.Hence,
áY a Y 5 ] 15 l that can be purchased for $7.68 is the great=;
bx b X 9 l0 -----or1:()
90 6""'" integer that is equal to or less than ffi. Si...
Grid in 6. E* :
Y#
$0,49
S.el.
15.67, the sreatest
greatest of p.:-,
numÜ"éi'of oe:,-
that can be purchased is 15.
3. (9) Since the ratio of dimes to pennies in the
purse is 3 : 4,Let 3x and 4x represent the num- 2. (D) From 10:40,c.Na. to 1:00 r,.v., 2 ho*:.
bers of dimes and pennies, fespectively. If 3 and 20 minutes elapse. Since the car travel. r
pennies are taken out ofthe purse, then 4x , 3 a constant rate of 45 miles per hour, it tra,.
=
pennies remain. 2 X 45 or 90 miles in 2 hours. Álso, si:-:
Since the rario of dimes ro pennies is now 20 minutes is } of an hour, the car tra,,
1 : 1, ] X 45 or 15 miles in 20 minutes. The :
ir cravels is 90 * l5 or l05 mile.,
"di"rance
3xl
4;=T:-j or 4x-3:.3x 3. (A) If Épencils cost . cents, then 1 p;:
costs pelrcils cost 7 X j or !f.
f,, so 7
Answers ta Chapter 5 Tune-Up Exer)ises 215
4. (B) Since the lreight train leaves 1 hour
before 6. (A) If x people working together at the same
the passenger train, it travels 1 hour longer.
rate can complete a job in / hours, then each
Therefore. when rhe passenger rrain ha,
rrau- person working alone can complete the same
eled x hours. rhe freight riain has rraveled
job in xb hours, so the rate of work is
xl l hours. Makea table. the
reciprocal of xh or 1,. Hence. rhe parr of the
yh
Bate " Time - Distance job one person working alone can
complere in
Éhours is tr .
50 mph X+
,1
hours 50(x + ,])
xh
00 nrph x hours 60x 7. (D) Since the rate of work is constant, form a
proportion where :r stands for the time it takes
Since the two trains are traveling in opposite to install 8 light fixtures:
directions, the sum of their diitances ^must
equal 380 miles: 58
50(x + 1) + 60r: 380 time toinstall 3x
50x + 50 + 60y: 380 5x: 24
110x: 330 5x_24
x: 330 _ 1
55
1l0
x
,4,nours
: 4,
Since rhe passenger train leaves aL ]:00 t,.v..
tne two lralns are 380 miles apart J hours
8. (E) Let_r be the rate oF speed of the freight
later or at 4;00 p.u.
train. Make a table .
5. (D) Let ,í represent the distance driven
at
each speed. Since time : distance + rate
of Rate X Timé : Distance
speed, the time driving at 60 miles x mph
per hour 3 hours 3X
can be represented as and the rime driv-
,'6. x+20mph 3 hours 3(X + 2oJ
lng.at 40 miles per hűr can be represented
". á. The total distance the two trains travel in 3
. Since the total driving time was 1 hour, hours is 500 - 80 or 420 miles.
Hence,
XX 3x+3(x+20):420
60- ' 3x"t3x+60:420
. ^= 6x: 360
Solve For x by multiplying each term on
borh sides of rhe equation by 2,400, rhe ,: l!0
6
= 60
lowest common multiple of 60 and 40: 9. (C) Ifa machine can seal 360 packages per
40 60 hour. then 1, ,.1. Jé9 61 6 packages per minure.
Lrlo (:;\ - )Áoo (+\ = 2.a00(I) If another machine cal seal l40 packages per
\60l \4t) !j
hour. rien i, ,.r],
60 -- ]] r*,
or packages per
40x + 60x : 2,400
minute. lf dre rwo machInes worl<ing rogether
I00x _ 2,400 can seal a rota] o[ 700 plckages in .i"nur.r,
100 100 then "
x= 24 -7
tlx+ix:700
. Because the one-way trip disrance is 24 mi]es, 18x -| /y : 21gg
the round rrip disrance is 24 + 24 48 : 25x: 2100
miles.
,: u 2 l00
:on
216 Word Problems

10. (B) Make a table. GBlD-lN


Bate X Time : Distance 1 . (8.0 or 8) If 5 pounds of fruit serve 18 peo-
ple, then
f pound serves one person, so
'l8 mph
-} nou." lalx 1) 24y2--4<1 )o
" \ 4) lE ., J- Pounds
12 mph x hours 12x serve 24 people. Since the fruit costs $1.20 a
pound, the cost of the fruit needed to serve
24 people is
Since the length of the lake remains constant, 2! x st.zo:20 \ $0.40 or $8,00
/ l\
18| x- , |: 121*;
3
\
T/
2. (12) Since the rate of work is constant, form
rcx_ !-lz,
4
a proportion where r stands for the time, in
minutes, it takes to produce 920 flyers;
-9
o,: , 4600 920
9C)J (:
^ 61,2) 1) 4
Time to produce 60 min 1 6our;
2
Hence,
RateXTime:Distance 460x 6 x,9zÓ

12X]:9-11"
460 46ő
4 rc : 12 minutes
11. (D) Let.,,r be the time For the outgoing trip,

Make a table-
Rate X Time = Distance

x
.
mpn 12o
nours
- ,l20

x+5mpn -115
120
hour5 120

Since the time for the return trip was one-


third of an hour less than the time for the
outgoing trip,
120 l20 J_ 1

x x*5 3
Geometric Concepts
and Heasoning
t|-. \-. !-.

-, -r
-. -.

his chapter reviews the geometric relationships and properties of figures


that
are tested on the SAT. Keep in mind that you will .roi b. ..quir.d
to-.o-pl"t.
:lrmal geometric proofs or to memorize key formulas.
Unless otherwise stated, figures on the SAT are drawn to scale. Ifyou
find a note
:rar tells y,ou. the accompanying 69 ure is not d,ra,wn to scale,
try redraivi"g .rr. ig"..
, l that
it looks in scale. You may then be able to arrive at the answer by"estimaiing
. b1. eliminating answer choices that appear to contradict the diagram.

Lessons in This Chapter


Lesson 6- 1 Angle Relationships
l-esson 6-2 Special Tiiangles
Lesson 6-3 Tiiangle Inequality Relationships
Lesson 6-4 Polygons and Parallelograms
Lesson 6-5 perimeter and Are a
Lesson 6-6 Circles
Lesson b-l Solid Figures
Lesson 6-8 Coordinate Geometry

217
Angle Relationships
a!r,§r- -, {d

Overview
This lesson reviews the relationships among
. angles l'ormed when rwo ]ines int.rs..,
. angles formed when two parallel lines are cut by a third line
. the angles ofa triangle

Classifying Angles
Angles can be classified according to their degree measures (see Figure 6.1).

. An acute angle is an angle that measules less than 90'.


' A right angle is an angle that measltres 90".
. An ábtuse angle is an angle that measules more than 90" and less than 180",
' A strai8ht angle is an angle that measures 180', so its sides lie on the same
straight line.

Acute angle
stíaight angle

la'\
l/
0<a<90 a=90 90<a<180 a= 180

Figure 6.1 Classifring Angles

Pairs ofangles whose measures add up to 90' or 180" are given special names,

. Tivo angles are complementary iíthe sum ofthe numbers ofdegrees in the
angles is 90".
' Two angles are supplementary if the sum ofthe numbers ofdegrees in the angla
is 180'.

21a
Angle Relationships 219
Adlacent Angles
In Figure 6.2, angles 1 and 2 are adjacent ang|es
because they have the same vertex,
share a common side, and do not overlap.

Figure 6.2 Adjacent Ángles Figure 6.3 Adding Arlgles About a Point

The sum of the measures of the adjacent angles about a point is 360.
ln Figure 6.3,

L1 +L2+L3+L4=360.
figufe at the right, the measure of angle a is
j60'
|1^t|re
- 70" or 290".

Angles Formed by lntersecting Lines


\\/hen nvo lines intersect, vertical or opposite
angles have equal mexures and
rdjacent angles are supplementary. In th. hg.,r.
b.loi,,

' L| : L3 and L2: L4


' Ll + L2: 180. and L1 + L4 : 180.
' L3 + L4: 180. and L2 + L3 - l80.
22o Geometric Concepts and Reasoning

Bisecting Ang|es and Line Segments


A line segment or an angle is bisected when it is divided into two paf$ that have
equal measures.

. In the figure below left, line segment,4,B is bisected at point tllsince line
segments AM and MB have the same length. A

' In the 6gure above right, LABCis bisected by ray BD slnce it is divided into two
angles that have the same number oí degrees.

Angles Formed by Parallel Lines

rr? z
When two parallel
Parallel lines are always the same distance apart and, as a result, do not intersect. In
Figure 6.4, lines l and rz are parallel. The notation é|m is read as "éis parallel to m."

lines are cut by a


third line,
. angles that look
equal are equal
. angles that do
not look equal are
supplementary

Figure 6.4 Parallel Line Angle Relationships

In Figure 6.4,

. Lb : Ld: Lf : Lb, since all the acute angles Formed by parallel lines are
equai.
. La: Lc- Le: L& since all the obtuse angles formed by parallel lines are
equa1.
. La: Lb : I80, La + Ld: l80, La + Lb : 180, and so forth, since the
measures ofany acute angle and any obtuse angle formed by parallel lines alwa,vs
add up to 1 80.

If a line is perpendicular to one ol two parallel lines, it must also be perpendicular i_


the other parallel line, In the accompanying figure, line l is parallel to |ine m, If line :
is perpendicular to line l, then line p must also be perpendicular to line m.
Angle RelationshlpS 221

EXAMPLE
ln the figure below, if l m, What is lhe Value of x?
ll

So]ution
'fhe
angle marked x is acute, and the angle marked, -| 30 is
2x obtuse. Since
:hese angles are formed by parallel
tirr.r, th.lri.g... ;;;;;, add up to 180. Hence:
x* (2xi 30) : 180
3x : 150
: 150
" = 56
3
ing|es of a Triangle
. he sum ofthe measufes of the three angles
ofa triangle is 180. (see Figure 6.5).

a+b+c=,180
Figure 6.5 Sum oftheAngles of a Tiiangle

- exterior angle ofa triangle,


as shown in Figure 6.6, can be formed
at any veftex
:ne rrlangle by exrending one of its sides.
222 Geometric ConCepts and Reasoning

d+a_lBOandd=b+c 120'+100'+140"=360"
Figure 6.6 Exterior Angle ofa Tliarrgle Figure 6.7 Adding the Exterior Angles ofa Tiiangle

In any triangle,

an exterior angle and interior angle that have the same veítex afe supplementary
(see Figure 6.6),
an exterior angle is equal to the sum of the measures of the two remote
(nonadjacent) interior angles (see Figure 6.6).
. the sum ofthe measures of the exterior angles, one drawn at each vertex, is J60"
(see Figure 6.7).

If a triangle contains a right angle, then


the measures of the two remaining angles
of the triangle must add up to 90'. In
the figure at the right, since 1Cis a right
angle, it measures 90", so a + b: 90.
Angle Relationships 223
Lesson 6-1 Tune-Up Exercises
Multiple-Choice
1.
3.

In the figure above, x l 7:


(A) 270 Ifin the figure above ! : 5 and 1: 4,
(B) 230 what is the value ofx?
(C) 210
(D) 190
(A) 8

(E) 180
(B) 10
(C) 12
). (D) 15
(E) 18

4.

ln rhe 6gure above, if t,llt,. whar is the


In the figure above, what is the value of value of x?
1?
(A)20
(A) so
(B)30
(B) 85
(C)45
(C) 75
(D) 60 (D) zo
(E) It cannot be determined from the (E) 63
information given,
5. Ifthe measures ofthe angles ofa triangle are
in the ratio of 3 :4:5, what is the measure
of the smallest angle of the triangle?
(A) 25
(B) 30
(C) 45
(D) 60
(E) 75
224 Geometríc Concepts and Reasoning

6.

In the figure abore, if t,llér, what is the


value of1?
In the figure above, if AB l BC what is
(A) 90
the value of x?
(B) 100
(ö 18 (C) 120
(B) 22 (D) 135
(C) 25 (E) 150
(D) 27
(E) 32 10.

7.

(,, (.
In the figure above, if 1,1ll, and éo|(r, whar
is in terms oí x?
In the figure above, if l, ll lr, what isx*y .1L,

in terms of zl and z? (ö 9o+r


(B) 90 + 2r
(A) 180 - wl z
(B) 180 l w- z (C) qo-1
(C) 180 - u- z 2
(D) 180 l ul z (D) 90 - Zx
(E) w+ z (E) 180 - 2y

8. l1.

In the figure above, what is z in terms of x In the figure above, what is the value of .r:
and y?
(A) 12
(A) ,+./+ 180 (B) 15
(B) r+7 180 (C) 20
(C) 180 - (x + !) (D) 24
(D) x+ 1+ 360 (E) 30
(E) 360 - (x + 7)
AngleRelationshlps 225
l2. 15.

ln rhe figure above. For which value ofx is


t,llt,l In the figure above, what is 7 in terms
ofx?
(A) 37
(B) 43 (ö 3
(C) 45 -x
2
(D) 47 4
(E) 55
(B)
3
(C) x
13,
3
(D)
4

(E)
2
-X
3
In the figure above, if é || zn, what is the
16.
value o|l?
(A) 60
(B) ,0
rc) 45
(D) 30
(E) 25
ln the figure above. iFZ,||7,. the value
of x is

G,) zz!
2
(B) 30
(C) 45
(D) 60
In the figure above, what is the sum ofthe
(E) it cannot be determined from the
degree measures of all of the angles information given.
marked?
17.
(^) 54o
|B) 720
C) qoo
D) 1080
D l44o In the figure above, what is the value of y?
(A) 35
(B) 45
(C) 50
(D) 60
(E) 75
226 Geametric Concepts and Beasoning

18. 20.

In the figure above, if line segment lB is


parallel to line segment CD, what is the In the figure above, which of the following
value of1? statements must be tfue?
(A) I2 I. ai- b: d+ c
(B) 15 II. a-f c-f e:180
(C) 18 IIL b+ 7: ,1 ,
(D) 20
(E) 24
(A) I only
(B) II only
(C) III only
19.
(D) i and II only
(E) II and III only

Note: Figure is not drawn to scale,

In ARS7above, what is the value of x?


(A) 40
(B) 60
(C) 80
(D) 90
(E) 100
Angle Relationships 227
Grid-ln
1.
Questions 3 and 4.

In the figure above, if line segment lB is


parallel to line segment CD and BE L ED,
what is the value of7?
Line segments OP, OA, and OQ in the
figure above, coincide at the time r : 0
2.
second. At the same instant of time, OP
and OQ rotate in the plane about point O
in opposite directions while Ol remains
fixed. Vith fespect to OA, segment OP
rotates at a constant rate of 4" per second
A---:,D C
and segment OQ rotates at a constant rate
of 5' per second.
In the figure above, what is the value ofx?
3. rü7hat is the smallest positive value
of r, in
seconds, for which segments OP and OQ
will coincide?

4. ]W4len - 1 hour, how many more


'
revolutions does segment OQ complete
than segment OP?
la §!:]i,,i
Special Triangles
ffi*§
Overview
This lesson reviews angle and side relationships in special types of triangles.
The following right triangle reFerence formulas are supplied in the SAT test
booklet:

,N "r *

a lsx
c2=a2+b2 special Right Triangles

Angle and Side Relationships in an lsosceles Triangle


In an isosceles triangle, equal angles are opposite equal sides, as shown in Figure 6.8.

b=c b_c
Figure 6.8 Isosceles Tiiangle Relationships

Angle and Side Relationshlps in an Equilateral Triangle


In an equilateral triangle, all three sides are equal and each of the three ang.,s
measures 60o.
If the three sides of a triangle have the same length, you may conclude that
triangle contains three 60' angles (see Figure 6.9).

22a
Special Tiangles 229

a=b=c=60 LB=60"anda=b=c
Figure 6.9 Equilateral Tiiangles Are Equiangular Figure 6.10 Equiangular Tiiangles Ate Equilateral

If rwo angles ofa triangle measure 60", the third angle also measures 60" and, as a result,
rhe three sides have the same length (see Figure 6.10).

Side Relationships in a Right Triang|e


In,a right triangle, the greatest angle is a right angle, the side opposite the
ight angle is called the hypotenuse, and the oiher two sides are calleá legs (see
iigure 6.11).

Leg,

Figure 6.1l Right Tiiangle

_ arry right triangle:

(leg,)'i- 1legr)' : (hypotenuse)2

ris important relationship is called the Pythagorean theorem. The Pythagorean


' .olem can be used to find the length ofany side ofa right triangle when the lengths
:the other two sides are known. For example, if in the accompanying figure one'leg
.]. the other leg is 5, and xis the length of the hypotenuse, Áen

9+25_P
34:x)
l:.
- v:+
^._
^
23o GeometriC Concepts and Reasoning

Pythagorean Triples
Ifalib2:c2, then a, b, and r form a Pythagorean triple. For example, the set of
numbers (1, 2, 16) satisfies the pl,thagorean relationship since the square of the
largest number, 16 , equals the sum of the squares oí the two other numbers:
|1 l22 = 1+ 4:5 a"d (.16)'- 5

so

s*eI:@
a2 b) C2

A Pythagorean triple is any set of positive integers that satisíy the relationship
a'+b2:cz.
If the lengths of the sides of a triangle form a Pythagorean triple, then the triangle is
a right tiiangle in which the longest side is the hypotenuse of the triangle. For
.*"Áple, the triangle whose sides measure 1, 2, and ,]5 is a right triangle in which
rhe lengrh oFrhe hyporenure is ,F.
Some Pl,thagorean triples occur so frequently that you
rwg ,§
should memorize them. For example:

Time Saver
The ordered set of numbels (3, 4, 5) forms a
Recognizing a Pythagorean triple can save you time. Pyrhagorean rriple since J] -L -]2 5'.:
lfthe length and width ofa rectangle are 5 and12,
The ordered set of numbers (5, 12, 13) forms a
respectively, then you immediately know that since
5-12 13 is a Pythagorean triple and the diagonal Pythagorean triple sirrce 51 * 7T !32. :
of a rectangle is the hypotenuse of a right triangle, The ordered set ofnumbefs (8, 15, 17) forms a
the diagonal must be 13. Pvthagorean triple. lince 8' + 15' :
'r'.
'i7hen each member of a Pythagofean triple is multipliec
M§trX R§r§a§N§§ rlE§Y bythe same positive integer, the result is also a Pythagorear
triple. Because 3 4-5 is a Pythagorean triple and
Other basic Pythagorean triples that you may
encounter are 7-24-25 and 9-40-41. Don't be
3X 2 : 6, 4 x 2 : 8, and j x 2 : l0,
alatmed ifyou don't remember a particular
Pythagorean triple, since you can always find the
6-8-10 is a Pythagorean triple, as are 9-12-1<
unknown side ofa right triangle by substituting into
the Pythagorean theorem and then solving for it. 12-16-20, I5'20-25, and so forth. Similarl,
multiplying each member of the Pythagorean tripl;,
5 12-13 and 8- l 5- 17 by 2 also produces Pythagore;:
triples: 1 0-24-2 6 and 16-30-34, respectively.

EXAMPLE
ln A/BC below, What iS the lenqth oí line Segment BD?

AD
_12+
special Triangles 231
Solution
If rwo sides of a triangle are eoual,
-DC a perpendicular drawn to the
side. Hence. AD : = á r. .igh, ,.i"di;-;;;, '# i."r,',. .r the sidesthat
third side bisects
(6. x l0) Plthagorean are
:*;'j:1 i,pl'.*nJ,, )Ő'! i,irj.*,*.,<r : 10, and

Side Relationships in a 45o_45.


Right Triangle
In an isosceles right triangle
the following are true:

Each acute angle measures 45..


The lengrh ofrhe hyporenuse is
J) times rhe |ength ofa leg.

the length oFrhe hyporenuse.


|: , )rimes

_-or example,

x:3, l,=3rl) ,=1:5nE ,=+Ji


l:"5,xlJi:a
232 Geometric Concepts and Reasoníng

Side Relationships in a 30"-60' Right Triangle


Ifthe acute angles ofa right triangle measure 30'and 60', then

. The leg opposite the 30" angle is one-halí the length of the hypotenuse.
Equivalently, the length ofthe hypotenuse is 2 times the length ofthe leg
opposite the 30' angle.
. The leg opposite the 60" angle is .6 times the length of the other leg.

For example:

y y
l
=4 2(5) : I0
z(sl
!: :
l: : 4J' 5J'
2(7)
special Triang|es 233

Lesson 6-2 Tune-Up Exercises


Multiple-Choice
1. c
4,

In the figure above, what is the length


In the figure above, r :
of R§?
(A) 4
(A) (B)
^lF - J3 6
(B) 2 G) 4J'
(C) 4
rD) z + r,6 @) 4J'
(E) 10 (E) 8\,5
5.

In LÁBC, L C - 90" , CD : BD, and, the


ratio of the measure of LA to thc measure In equilateral tríangle ABC above,
of LBis 2 to 3. IVhat is the value of x?
7 -
(A) 40
(A) 18 (B) 60
(B) 36 (C) 70
lC) 40 (D) 75
(,D) 54 (E) 80
(E) 72
6.

In the figure above, tf AC - BCwhat is 7


in terms of x?
In A/,(Z above, what is the value ofx? (Á)
1) z "
B) 3
(S) ?,
2
C)4 (C) 2x
D) 6 (D) 3x
E) It cannot be determined from the (E) 5x
information given.
234 GeometríC Concepts and Reasoning

7, B

Er
|-5
In the figure above, AB : CB, DAbisects In the figure above, if BC l ,4D, what is the
L BAC, and DC bisects L BCA. What is length of lin e segment CD?
the value of x? @) 6\E
(A) 18 (B) 8\6
(B) 30 (C) 15
(C) 36 @) \2J'
(D) -7) (E) 1,7
(E) 108
10.
8.

Note: F gure is not drawn to sca|e,


|.- 10-----'*1
In the figure above, what is the ratio of Rü-
In the figure above, what is the length of to I7S?
line segment lB?
(A) ótot
(A)
(B) V3 to 1
(B) 2J3 (C) 2to 1
(C) (D) 3tot
(D) 3Ó (E) 3toz
(E) 6
11. Katie hikes 5 miles north, 7 miles east, anJ
then 3 miles north again. 'ü4lat number o:
miles, measured in a straight line, is Karie
from her starting point?
(A) n&5 ("ppro"i-ately 9.1 1)
(B) 10
(C) ]t l3 (rpp.o*i-ately 10,63)
(D) I3
(E) \5
2,

F
Note: Figure is not drawn to scale,
Note: Figure is not drawn to Scale,

in rhe figure above, iF Bá = BC


In the figure above, what is the length
of BC I AD. and ÁD bisecrs LADC, what
line segment lD?
is the value oíx?

3. Two hikers started at the same location.


one traveled 2 miles east and then 1 mile
north. The other traveled 1 mile west and
then 3 miles south. At the encl of their
hikes, how many miles apart were the rwo
hikers?
Triangle lnequality Relationships

Overview
In any triangle,
. the length ofeach side is less than the sum ofthe lengths ofthe other
two sides and greater than their difference;
. if side lengths or angle measures are unequal, then the longest side is
opposite the greatest angle, and the shortest side is opposite the smallest
angle,

Testing Three Numbers as Possible Triangle Side Lengths


To determine whether a set olthree positive numbels can represent the lengths ofthe
sides of a triangle, check that each of the three numbers is less than the sum of the
other rwo numbers.

EXAMPLE
Which oí the íollowing sets of numbers cANNoT represent the lengths of the sides of a

íi+,,
triangle?
(A) 9, 4o,., J

i=i l
ll
lvtrt l
DrEFDFtarF
-:"l;:lo:,;,
F&§Y
The length of any
ii rri t r,*;:l H,.iTl.",2i:;i,: §,,|li ;l, rrr I
side of a tíiangle is
greater than the

ffirui".i
difference and less
than the sum ofthe

I
lengths of the other
tr,vo sides.
i,, limited
In any triangle, each side length is l,n'e of valrres,r,,,,
ló a range ofvalues
limire,] to o*--,".o.,
*n***:;*,**!; ;n,§ln,* *,-
;*:**f

236
1
Triangle lnequa|ity Relationships 237
EXAMPLE
lí 3, 8, and X represent the lengths of the sides oí a tíiangle, how many integer Values for x
are possible?

Solution
. The value of rmustbe greater than 8 - 3: 5 andless than 8 + 3: 11.
' Since x ) 5 and x ( 1 1 , x must be between 5 and 1 1.
. Because it is given thatxis an integer, xcan be equal to 6,7,8,9, or I0,
. Hence, there are five possible integer values for r.

EXA|VPLE
lf the lengths oí iwo sides of an isosceles triangle are 3 and Z What is the length of the
third side?

SoLUT|oN
Since two sides of an isosceles triangle have the same length, the length of the third
side is either 3 or 7 . The length of the third side cannot be 3 since 7 is zal less than
3 + 3. Iíthe third side is 7, then 3 <7 + 7 and 7 < 3 l7.Hence, the length of
the third side must be 7.

Unequal Sides are Opposite Unequal Angles


In a triangle, unequal sides are opposite unequal angles. These relationships MATx
are
illustrated in the figures below.
xEFExEN[í
rAcT
lf two sides (or
angles) of a triangle
are unequal, then
the measr.tres of the
angles (or sides)
opposite them are
also unequal, with
the larger angle
lying opposite the
longer side.

b> c
b> c
238 Geametric Concepts and Reasoning

Lesson 6-3 Tune-Up Exercises


Multiple-Choice
1. In obtuse triangle RS7] R7:
75. \,X/hich 4.
of the following statements must be true?
I. LR: LS
II. Z 7is obtuse.
III. Rs> rs
(A) I only Note: Figure is not drawn to scale,
(B) I and II only :
In AABC, if AB BD, which of the
(C) I and III only
following statements must be true?
(D) II and III only
(E) I, II, and III I. x} z
II. 1> x
2, In a triangle in which the lengths oftwo III. AB> BC
sides are 5 and 9, the length ofthe third (§ II only
side is represented by x. \W4rich statement is (B) I and II only
always true? (C) I and III only
(A) x> 5 (D) II and III only
(B) X< 9 (E) I, II, and III
(C) 5,<
(D) 4< x114 5. How many different triangles are there for
(E) x<14 which the lengths ofthe sides are 3, 8, and
where z is an integer and 3 1 n1 8?
3. In LABC, BC> AB and AC < ,4B. \W4rich (A) T*o
statement is always true? (B) Three
(A) LA> LB> LC (C) Four
(B) LA> LC> LB (D) Five
(C) LB> LA> LC (E) Six
(D) LC> LB> LA
(E) LC> LA> LB 6. In AR§I RS -t ö
1Imeasures 40". ,-:
Wis a point on side R7such that /Ri\'_'
measures 100'. \W&ich of the following
segments has the shortest length?

(A) Rs
(B) s7
(C) R\Y/
(D) 71Y
(E) slí/
Triangle lnequality Relationships 2gg
Grid-ln
. If the lengths
1 of two sides of an isosceles 3. If the integer lengths of the three sides of a
triangle are 7 and 15, what is the perimeter
triangle are 4, x, and 9, what is the least
of the triangle?
possible perimeter of the triangle?

2. The perimerer of a triangle in which rhe


[f the producr of rhe lengrhs ofthe rhree
lengrhs ofaJl oF rhe sides are integers is 2l.
sides ofa rriangle is l05, whar is a possible
If.the length ofone side ofthe triangle is 8,
perimeter of the triangle?
what is the shortest possible length o'f
another side of the triangle?
*§,]§
Polygons and Parallelograms
a, ,]

:rb á
,IA*.&&
'-er &
Overview
A triangle is a three+ided polygon. This lesson considers some angle and side
relationships in polygons with three or more sides.

Térms Related to Polygons


The accompanying figure shows a polygon with five sides. The corner points ,4, .B, Q
D, and E are called vertices. Line segments,4C and AD are diagonals. A diagonal of
a polygon is a line segment that connects any two nonconsecutive vertices of the
polygon.

An equilateral polygon is a polygon in which all the sides have the same lengr:
An equiangular polygon is a polygon in which all the angles are equal.

Angles of a Quadrilateral
A quadrilateral is a polygon with four sides. Since a quadrilateral can be divided ;::.
two triangles, as illustrated in Figure 6.12, the sum oFthe four interior angles .,l.
quadrilateral is 2 X i80'or 360'.

a+b+c+d=360
Figure 6.12 Angles ofa Quadrilateral

240
Polygons and Parallelograms 241
Sum of the lnterior Angles of a Polygon
By drawing diagonals from the same
vertexJ a five*ided polygon can be
separated
j",",5. 2 = 3 rriangles as illustrated on page 240.
.
he divided into n - 2 nonoverlapping
O"r'o"Írg." with r sides can
,.i;d;; S'l".; h:;i,-n of rhe angles of a
the sum ,! of the'i;";",
;l1ittTj:l, "Ű;;'"f " p"iig." *,,r, z sides is given
S= (n- 2) X 180.
For example, to find the sum of the
five interior angles of a five-sided polygon,
substitute 5 for nin the above formula:

S=(n_2)Xl80"
:(5-2) X 180.
X l80.
: 540"
a+b+c+d+e=54o
Sum of the Exterior Angles of a Polygon
|he sum of the exterior
.i;i.l"rrr.")* jil:iJ..ffi:T],lills"",
ansles ,
with one angle drawn at each yertex

21 + 22+23+14=g60ó
Figure 6.13 Exterior Angles ofa Polygon

Scecial Polygons
,_. of
the sides or all of the angles of a polygon
may have the same measure.

' {n equilateral polygon is a polygon


in which all of the sides have the same
,-ngth.
' ,\r equiangular polygon is a polygon
in which all of the angles have the same
:Tleásufe_
' { regular polygon is a polygon
that is both equiangular and equilatera],
242 Geomelric Concepts and Peasoning

lnterior and Exterior Angle Relationships


At the vertex ofa polygon, the measures ofthe exterior and interior angles add up to
180". In an equiangular polygon with z sides:

. each exterior angle


. : 360,
'"" . For example, the measure of each exterior angle oF
lz
an equiangular penlap,on is
160.
""-" : -2".

. : ]60"
each interior angle 180' - | |. Fo..r"-pl., the measure ofeach
\, )
interior angle ofan equiangulaf pentagon is 180" - 72" : 708".

rE Ív Similar Polygons
Formulas worth remembering for l-sided polygons:
-ü/hen
: a photograph is enlarged, the origina1 and enlarged
l Sum ofinterior angles (n - 2) X 180o
figures are similar sínce they have the sam€ shape. IF two
l Sum ofexterior angles = 360o
polygons are similar, corresponding angles are equal and
l At each vertex, interior Z + exterior Z = 180o the ratios of the lengths of corresponding sides are equa1.
For equiangular r-sided polygons: Keep in mind that corresponding sides are opposite
l Each exrerior ans|e : 360" matching pairs of equal angles. For example, the
quadrilareral, below are similar.
l : 180'
[+)
Each inrerior angle

The ratio of the lengths of any pair of corresponding sides of the two
quadrilaterals is 2 to 1:

side of ABCD AB BC CD AD
corresponding side oí JIűM ]K IC LM ]M
9421:! :? 10
5
2:l
Polygons and Parallelograms 243
Similar Triangles
If tao,angles of one triangle are equal to two angles of the second triangle,
the two
triangles are similar. For example, the trianglei in Figure 6.14 are
similar since
LB : LD and the vertical angles at Care equal.

Figure 6.14 Similar Tiiangles

since the triangles in Figlre 6.14 are similar, the lengths of corresponding
sides are
in proportion, Length 8 corresponds to length 6 sincá these
sides are opposite equal
angles. AJso, length 12 corresponds to lengih x since these
sides are oppo.it. .quul
angles. Hence:
Side of I A 812
Side ofA II 6x
8x:72
72
8

rarallelograms
\ is a special q,pe of quadrilateral in which both pairs of opposite
,parallelograrn
,:des are parallel. A parallelogram h", these three additional properties:

' Opposite sides are equal. Thus:

AB: CDandAD- BC
' Opposite angles are equal. Thus:

a-candb:d
244 Geometric Concepts and Reasoning

. Consecutive angles are supplementary Thus:

a-t b- bl c: c+ d: d+ a:180

Special Types of Parallelograms


A rectangle is a parallelogram widr four right angles. A rhombus is a parallelogram
with four equal sides. A squate is a parallelogram with four right angles and four equal
sides.

Flectangle Flhombus square


c

,§,
A D Au "D
AB=CDandAD=BC AB: BC _ CD: DA AB=BC:CD=DA
AC=BD ACrBD AC=BDandAC lBD

Diagonal Properties of Special Types of Parallelograms


In any parallelogram, each diagonal cuts the other diagonal in half.

. In a rectangle, the diagonals have the same length.


. In a rhombus, the diagonals intersect at right angles.
. In a square, the diagonals have the same length and intersect at right angles.

Geometric Notation
The SAT uses standard geometric notation for length, segments, lines, rays, and
con8ruence.
. } ..p.....rts the line that passes through points l and &

AB
. / ..p.....,t, the ray beginning ai point,4 and passing through point B
AB
. n3 ,rprr"rnt the line segment with endpoints l and B:

,4ö without an overbar, fepresents the length of Á . ff thr length of .,,qa i, S,


then AB : 6
Polygons and Parallelograms 245

,1B = CD is read as "line se3ment


AB is congruent ra Iine segmenr CD." Two
llne segmenrs are congruent when they have the same
lengrh:

AB = 6:11 since ÁB : BC = 3
con8ruent if all pairs oF corresponding sides
}:"rrt _: t
all pJlrs ol corresponding angles are congruent,
are congruent and

:or example, if
= AD,ái then
^ABC
Ats-DE zA-zD
D/-
EF
- anl Zts = ZE
ÁC .= DF /r - ,/E
_ine Symmetry
.: a line divides a, figure into two parts that when folded
along the line exactly
,oincide, then the 6gure has line symmetry.
The line of .y-m.try m"y be
a horizontal
,rc. a verrica] line, or a slanted line, as
iliu.rrared in ,há 6gur.r.
".--prnying

oK
Horizontal line of symmetry
l
I

vertical line
of symmerry
Slanred line of symmetry
246 Geometríc Concepts and Reasoning

A figure may have more than one line of symmetry or no lines of symmetry:

one horizontal and one


vertical line of symmetry A square has four lines of symmetry No lines oisymmetry

Rotational Symmetry
After a clockwise íotation of60'about its center O a regular hexagon will coincide
with itself. Regular hexagon ABCDEFhas 60" rotational symmetfy:
F' A'
,'----------',
\r 60' ,' ',
,z

lo Rotate 60'
E,(
\
/
a
o'
",
)s'

D'
"r-________,"
C'
A figurehas rotational symmetfy if it coincides with itself after a rotation thror-rqi
some positive angle less than 360'. Every regulár polygon enjoys rotational symmetr,

about its center for an angle of rotation of 360', where z is the number of sides oi
the polygon. For an equilateral triangle, {u".., ..g.r1". pentagon, and fegu.J
octagon, the angles of rotation are 120" , 90", 72", and 45' , respectively.

Point Symmetry
Á figure has point §ymmetry with respect to a point when it has 180" rotaric.--i
symmetry about that point, as illustrated in the accompanying figure:
c'

/\
/\//\
B'1---*-- -'ro'
,. P'
Rotated 180'
,'

A'

If you are not sure whether a figure has point symmetry, turn the page on u,h:::
figure is drawn upside down. Ifthe rotated and original figures look exactly th.
the figure has point symmetry.
Polygons and Parallelagrams 247

Lesson 6-4 Tune-Up Exercises


Multiple-Choice
I. Mlar is the sum of rhe lengrhs of rhe two
5,
diagonals in a q by l2 recrangle?
(A) 20
(B) 25
(C) 30
(D) 35
(E) 40
Note: Figure ]s not drawn to sca]e,
]. \X{-rat is the
ofa diagonal ofa square
length
In_the figure above, what is the length of
with a side length of JZ ?
síde AB?
-l; (A) 5
|^/
(B)
(B) -
2 6
1 (C) 9
(C) 2 (D) 10
(D 2J' (E) 12
(E) 4
6.
_]. A.diagonal of_a rectangle forms a 30" angle
with each of the longer sides of the
rectangle. If the length ofthe shorter side
is 3, what is the length ofthe diagonal?

(,\) 3",]D. Il in the figure above, ÁC: 3, DB : 4,


(B) 3.6 and, AB : 14, then AE -
(C) 4 (A) 19
(D) 5 (B) i2
(E) 0 (C) l0.5
(D) 8
-, If the degree measures ofthe angles ofa (E) 6
quadrilateral are 4x,7x,9y, andlOr. what
is the sum of the measures of the smallest
7. \W&ich of the following CANNOT
angle and the largest angle? represent the degree measure of an
{A) ]40 equiangular polygon?
(B) 150 (A) 165
lC) 168 (B) 162
lD) 180 (C) i40
E) 192 (D) l25
(E) 90
24a Geometric Concepts and Reasoning

8. 11, V4rat is the number of sides of a polygon


in which the sum of the degree measures of
the ir-rterior angles is 4 times the sum of the
degree measures of the exterior anglesl
(A) 4
If Egure ABCD above is a parallelogram, (B) 6
what is the value of7?
(C) 8
(D) 10
(A) l08 (E) 12
(B) 72
(C) 54 12. In quadrilateral l BCD, LA 1- LC is
(D) 45 2 times LB + LD.If LA: 40, then
(E) 36 LB:
C) (ö 60
(B)
^4^ü.r^F;
80
(C) 120
''.'r' ''ar' (D) 240
l l
(E) It cannot be determined from the
information given.

Vhich oFthe figures above could represent


13.
the rotation of square IBCD.rf about
point Á?
(A) I only
(B) II only
(C) iII only For parallelogram ÁBCD above, if
(D) I and III only AB > BD, which of the following
(E) I, II, and III statements must be true?

x
10. §íhich figures, ifany, have BOTH point I. CD< BD
symmetry and a line of symmetry? lI. LADB> LC

V rX,
III. LCBD> LA
v7 (A) None
(B) I only
L\ ^
lI
(C) II and III only
(D) I and I1I only
(E) I, II, and III
T ITT

(A) I only
(B) III only
(C) I and II only
(D) I and III only
(E) None of the figures
Palygons and Paratlelograms
24g
14,
18.

In the figure above, what is


Il in the figure above, CD: 1, AB: degree measures ofthe marked
the sum ofthe
2,
and AD = 6, then BC: anglesi

(Á)
(A) l20
5 (B) 180
(B) 9 (C) 360
(q 2+J' (D) 540
(D) 3,!6 (E) It cannot be determined
from the
information given.
tpt sJ) + zyE
15, If the length ofeach Grid-ln
side ofa triangle is 4,
how many differerr, lin..
of ,y-rn.'.f .",,
be drawn?

(A) 1

(B) 2
(C) 3
(D) 4
(E) It cannot be determined from
the
information qi.,o,,
l6, \X/hich lerter I. In^rhe accompanying figure ofpentagon
has point sym..r.y bur no ABCDE, points A
ltne of symmetrv? A.aid Blie'on,h". rr."
line. Whar is rhe value of7?
(A) E
(B) S
(C) W accompanying diagram of triangle
'. ABL.
'ij. AC- BC. Disapointon
(D) l tre,AEi,
(E) X exrended ro Á, and DlÁ- is drawn
so that
+ADE -
AABC.If mLC = 30, what is
l7. Through which the value ofx?
set ofcharacters consisting
ofa. lerter berween rwo digits
can borh a
horlzontal and a vertical line
of
symmetry be drawn?
(A) 3D3
(B) BS8
(C) BX8
(D) BYO
(E) 10]

You might also like